You are on page 1of 131

Universidad La Salle. Facultad Mexicana de Medicina.

Curso de Extensin Universitaria para la Preparacin del Examen Nacional para Aspirantes a Residencias Mdicas.

Nombre: Examen Diagnstico. Nmero de intentos: 3. Vigencia: 6 de Febrero del 2013. Horario: 9:00 a.m. a 9:00 p.m. Programar aleatorio. (5 bloques con 20 preguntas cada uno).

1.- Femenino de 45 aos es atendida en consulta externa por presentar desde hace 6 meses flujo transvaginal, mucosanguinolento y sinusorragia importante, refiere dispareunia de 3 meses de evolucin. AGO. G-5 P-5. Oclusin tubrica bilateral hace 6 aos. E.F.: TA 130/80 MMHG, FC 80 LPM, TEMP. 36.7 C. Laboratorio: HB 9.7 G/DL, HTO 37 %, Se reporta Papanicolaou clase V. El siguiente paso para confirmar el diagnstico es:

a) b) c) d)

Histeroscopa. Ecosonografa. Biopsia dirigida. Colposcopa.

El examen citolgico de papanicolau, realizado como examen de tamizaje en ginecologa, permite establecer las caractersticas de las clulas que se descaman del cuello uterino. La tcnica de papanicolau ha permitido disminuir la mortalidad e incidencia de cncer invasor de cuello uterino. Se define examen de papanicolau anormal cuando se presentan alteraciones en el ncleo, citoplasma y/o la relacin ncleo/citoplasma de las clulas examinadas. El papanicolau anormal, de acuerdo a la clasificacin de bethesda abarca desde un diagnstico citolgico de ascus (siglas en ingls de clulas atpicas de significado no determinado), lesin escamosa intraepitelial (lei) de bajo grado (corresponde a displasia leve y cambios por papiloma virus), lesin escamosa intraepitelial (lei) de alto grado (displasia moderada, displasia severa o carcinoma in situ) y clulas de cncer invasor.

4.6. Colposcopa Ideada por Hinselmann en Alemania en 1924, lleg a Amrica por el cono sur. Consiste en la visualizacin y amplificacin del cuello uterino mediante un sistema binocular de lentes, entre 25 y 40 aumentos, lo cual permite la observacin de las estructuras del cuello uterino mediante la asociacin con imgenes preestablecidas. La colposcopa tiene una mayor sensibilidad que la citologa, pero su menor especificidad. De conducir a procedimientos diagnsticos invasivos (biopsias y conizaciones) innecesarios y su mayor costo, son sus principales limitaciones. Combinadas la citologa y la colposcopia brindan una seguridad diagnstica que excede EL 95%. 4.6.1. Indicaciones de la colposcopia La colposcopa est indicada en las siguientes circunstancias (20), (21): - Pacientes con citologa cervical (papanicolaou) clase III, IV o V, o sus equivalentes en los otros sistemas de clasificacin. - Pacientes con citologa clase II con atipia inflamatoria, escamosa o endocervical, o cuando se informe la presencia de coilocitos. - Pacientes con crvix macroscpicamente normal, pero quienes presentan Sinusorragia. - Pacientes con crvix macroscpicamente anormal, en ausencia de carcinoma evidente. - Pacientes con citologa clase II persistente, pese a tratamiento de posibles causas. (Ejemplo: trichomonas). Tamizaje en cncer ginecolgico autores de la gua. Dr. Miguel Bueno Montao Profesor asociado departamento de ginecologa universidad libre gineclogo centro mdico Imbanaco profesor titular de ginecologa y obstetricia universidad del valle Dr. Jaime Rubiano universidad del valle Dra. Derry Trujillo.

2.- Mujer de 25 aos de edad, refiere que desde hace un par de meses ha presentado hemorragia irregular o postcoital, actualmente presenta disuria y dolor abdominal usted debe sospechar en cervicitis por:

a) Micoplasma b) Chlamydia c) Gardnerella d) Candida

Segn los datos de la Organizacin Mundial de la Salud, anualmente se detectan 89 000 000 de nuevas infecciones por Chlamydia trachomatis en el mundo. Esta infeccin provoca uretritis y cervicitis, y las secuelas incluyen enfermedad inflamatoria plvica, embarazo ectpico, infertilidad por dao tubrico, epididimitis, proctitis y artritis reactiva. Se considera principalmente un problema de salud en la mujer, en ella las manifestaciones y consecuencias son ms dainas para la salud reproductiva. Los individuos infectados con Chlamydia trachomatis pueden portar el microorganismo por meses o aos y transmitir la enfermedad a sus parejas sexuales. Su diagnstico sigue siendo un reto, ya que quienes la padecen presentan sntomas muy leves o son portadores asintomticos.

Manifestaciones clnicas

La cervicitis es la manifestacin clnica ms frecuente de la infeccin por C. trachomatis en la mujer. Sin embargo, el 70% de las mujeres infectadas no tienen sntomas, mientras que en el tercio restante las evidencias clnicas son poco especficas de infeccin, como flujo genital, dolor abdominal o pelviano, sangrado y/o disuria.

La presencia de disuria puede indicar una uretritis acompaante, lo que sucede en el 35% de los casos. En otras oportunidades, solo la uretra est comprometida, y la infeccin uretral se manifiesta como piuria o disuria con cultivo negativo (23% de los casos). El diagnstico se realiza al examinar el hisopado endocervical, que muestra flujo amarillento o verdoso con ms de 10 PMN por campo de inmersin en el examen de Gram. Este resultado define la cervicitis mucopurulenta (CMP) la cual tambin puede ser producida en casos de infeccin por gonococo o mixta (C. trachomatis y gonococo). Por lo tanto, el diagnstico debe confirmarse mediante estudios de mayor especificidad, como las tcnicas moleculares (test de ligasa, PCR), que tienen una sensibilidad del 96% aproximadamente, o la deteccin del antgeno por tcnica de ELISA, con una sensibilidad del 75%. Tambin se ha demostrado que las tcnicas moleculares en el primer chorro de orina son especficas y altamente sensibles.

Cuatro de cada diez mujeres con cervicitis no tratada adquieren enfermedad inflamatoria pelviana (EPI), con mayor riesgo de sufrir embarazo ectpico, infertilidad y dolor crnico pelviano. El riesgo de infertilidad se eleva segn el nmero e intensidad de los episodios: alrededor del 10% despus de un episodio, del 30% despus de dos, y mayor del 50% si ha habido tres o ms episodios. Por otro lado, el embarazo ectpico es cinco a siete veces ms frecuente cuando se trata de pacientes con antecedentes de EPI.

Tratamiento

Azitromicina 1 g VO dosis nica Doxiciclina 100 mg VO cada 12 horas por 7 das Eritromicina 500 mg VO cada 6 horas por 7 das Ofloxacina 300 mg VO cada 12 horas por 7das Levofloxacina 500 mg VO cada 24 horas por 7 das

CLAMIDIA

TRACHOMATIS

Basado en Guas Clnicas para el manejo de las ITS, OMS, 2003 / Norma Oficial Mexicana NOM -039-SSA2-2002

Berek J. (2002) Ginecologa de NOVAK. Mxico. Ed. Mc Graw Hill Interamericana. Pag 293.

3.- Masculino de 10 aos acude a su pediatra con fiebre e inflamacin dolorosa de la rodilla izquierda. Hace 2 semanas tena la garganta irritada, y un cultivo fue positivo para estreptococo hemoltico. El nio se trat desde entonces con un ciclo de 3 das de penicilina oral. Cul de las siguientes ofrece la mejor evidencia, por si sola, del diagnstico de fiebre reumtica aguda? a) b) c) d) Velocidad de sedimentacin eritrocitaria elevada. Soplo sistlico apical. Alta valoracin de anticuerpos estreptoccicos. Intervalo PR prolongado visible en el electrocardiograma.

Carditis Poliartritis Corea (Sydenham) Ndulos subcutneos Eritema marginado

Manifestaciones Mayores

La carditis es el sntoma diferenciador de fiebre reumtica, y un soplo sistlico apical, es el hallazgo inicial ms comn. Los dems casos enumerados en la lista son propios de pacientes con fiebre reumtica, pero tambin se encuentran en otros trastornos.

Clnicas Artralgias Fiebre Antecedentes de brote reumtico Laboratorio y Gabinete Elevacin de reactantes de fase aguda

Manifestaciones Menores

Prolongacin del intervalo PR Evidencia de infeccin Estreptoccica (Grupo A): Antiestreptolisinas Exudado farngeo PAC. Libro 1, Parte AFiebre ReumticaPag. 3,7 Dr. Jos F. Guadalajara Boo Jefe de Servicio Clnico. Instituto Nacional de Cardiologa Ignacio Chvez. Miembro numerario de la Academia Nacional de Medicina. Gobernador por Mxico del american College of Cardiology.

Presidente de la Sociedad Mexicana de Cardiologa.

4.- Se trata de neonato prematuro quien cursa con parto difcil, episodios de arritmia y sospecha de hipoxia-isquemia. Despus del nacimiento, se encuentra letrgico y tiene periodos de apnea. Se sospecha hemorragia intracraneana. No se observa un traumatismo craneal obvio. El USG craneal identifica sangre en los ventrculos. La estructura ms probable que da origen a sta hemorragia es: a) b) c) d) Corteza cerebral Matriz germinal Tlamo Vasos del crculo de Willis

Debe diferenciarse entre la hemorragia del neonato pretrmino y la del de trmino. La del recin nacido pretrmino es la ms frecuente y se produce por sangrado de la matriz germinal subependimaria.

La del neonato de trmino es menos frecuente y se produce por traumatismos, malformaciones vasculares, infecciones, tumores u otras causas. Matriz germinal

Es un tejido subependimario adyacente a los ventrculos laterales, altamente vascularizado, cuyos vasos muy delicados sangran fcilmente ante distintos tipos de estrs perinatal. Es una estructura transitoria del cerebro fetal que involuciona normalmente hasta desaparecer hacia las 32-34 semanas de gesta. Previo a su desaparicin slo persiste en el surco caudotalmico.

Se denomina "matriz germinal" porque es el tejido que da origen a las neuronas, que posteriormente llegan a la corteza cerebral por el proceso llamado migracin. Clasificacin de H. de la matriz germinal

Grado I: Hemorragia localizada en la matriz germinal. Puede ser unilateral o bilateral. Grado II: Hemorragia que se extiende al ventrculo lateral sin producir dilatacin. Grado III: Hemorragia intraventricular acompaada de dilatacin ventricular aguda y concomitante. Conviene aclarar que la hidrocefalia secundaria que se produce luego de varios das no se considera como grado III, pues se debe a dificultad en la reabsorcin de LCR en las granulaciones de Paccioni, por la obstruccin producida por el sangrado. Grado IV: Compromete al parnquima cerebral vecino. Esta clasificacin en grados es importante y tiene valor pronstico. A mayor grado, ms frecuentes e importantes sern las secuelas neurolgicas. Cuadro Clnico La HIV tiene generalmente un inicio sbito, a las 24 - 48 horas del nacimiento, pudiendo aparecer en un prematuro con una clnica post-parto normal y lgicamente debe esperarse su aparicin en los que tienen cuadros respiratorios asociados, en los nacidos de madres muy jvenes, en los de peso muy bajo y cuando el perodo gestacional es ms corto, recordar que el 90% de los nacidos con una edad gestacional de menos 32 semanas hacen HIV (37,39). El cuadro habitual es de aparicin de convulsiones, paro cardaco o apnea prolongada, con toma del sensorio, fontanela abombada y cada del hematocrito; pero hay pacientes que pueden tener poca o ninguna sintomatologa (48,54) y por esto es importante descartar una HIV ante la presencia de una cada del hematocrito sin explicacin aparente. Se conoce que una fontanela normotensa no descarta un cuadro de HIV. Nosotros tenemos pacientes RN que llegaron a nuestro servicio para tratamiento de una hidrocefalia aparentemente congnita (sin antecedentes de ningn tipo que sugiriera que fuera secundaria) y detectamos que eran portadores de un lquido ventricular hemorrgico; por eso es nuestra prctica en la actualidad realizar una puncin ventricular para estudio del LCR(tanto del aspecto como citoqumico) a toda hidrocefalia que debuta en los primeros treinta das de la vida. Diagnstico Est bien establecido la vigilancia estrecha de los prematuros en general y en especial los que constituyen el grupo de "alto riesgo" (vase fisiopatologa y patogenia). A todo prematuro se le debe realizar estudios con ultrasonidos diarios durante la primera semana de nacido y posteriormente semanales hasta el alta. Mucho se ha discutido acerca de la investigacin ideal a realizar en estos pacientes. Algunos recomendaron la Tomografa Axial Computada (TAC) por los detalles que la misma puede aportar; sin embargo tiene el inconveniente de que al nio hay que trasladarlo hacia el Departamento de Imagenologa y al mismo tiempo va a recibir una cantidad enorme de radiaciones, en un paciente inmaduro, lo que puede ser perjudicial. La Ultrasonografa (US) es indudablemente el estudio ideal, pues permite realizar el diagnstico al lado de la cama y

repetirlo las veces que se crea necesario sin efectos deletreos sobre el paciente; debe tenerse en mente la necesidad de visualizar la fosa posterior. Para nosotros el mayor valor del US est dado por la evolutividad del mismo y claro est en los casos que ofrezca dudas se puede complementar con la TAC (11,12,37,39,40,41,48). Los estudios Ultrasonogrficos, para la mayora de los autores, permiten clasificar las HIV en cuatro grupos fundamentales (6,11,12,62): Grado I: Cuando el sangramiento est localizado en la matriz germinal, sin o mnima hemorragia intraventricular. Grado II: El sangramiento es intraventricular y ocupa entre el 10 y el 50 % del ventrculo. Grado III: El sangramiento intraventricular es mayor del 50 % y el ventrculo lateral est distendido. Grado IV: Cuando a lo anterior se asocia sangramiento intraparenquimatoso. Volpe(63) slo reconoce los tres primeros grupos. Una vez establecido el diagnstico de HIV se realiza seguimiento evolutivo con US, buscando la aparicin signos de Hidrocefalia. James y colaboradores (30) sugieren los siguientes criterios para su diagnstico, atrium ventricular mayor de 10 mm, Indice de Evans mayor de 0,35 y cuando el cuerpo del ventrculo lateral, en un plano sagital, a nivel del tlamo mide ms de 10 mm.

5.- Femenino de 36 aos. Acude a consulta por presentar cefalea, cansancio e irregularidades menstruales con ritmo de 36 a 50 x 2-3 das. No se ha podido embarazar despus de 18 meses de actividad sexual regular. No tiene antecedentes importantes. EF: Campos visuales normales, tiroides aumentada de tamao una vez y aumentada de consistencia, no tiene galactorrea. Resto normal. Laboratorio: qumica sangunea, Bh y electrolitos normales. Prolactina 47 ng/dL (< 25), perfil tiroideo: TSH 18 mUI/ml, T4t: 50 nmol/L (57.9 a 154.4), T4L: 7.7 pmol/L (9 a 24), T3T: 1.06 nmol/L (1.2 a 2.9), T3L: 1.96 pmol/L (3 a 6.31) El diagnstico es:

a) b) c) d)

Hiperprolactinemia Hipertiroidismo Hipotiroidismo primario Sndrome de ovarios poliqusticos

DIAGNOSTICO Inicialmente el hipotiroidismo se diagnosticaba mediante la cuantificacin por tcnicas de Radio Inmuno Anlisis (RIA) de las hormonas circulantes triyodotironina y tiroxina; el proceso era lento y sometido a muchos factores de error que hacan su sensibilidad y especificidad poco confiables. Posteriormente, se desarrollaron tcnicas para la medicin de la TSH hipofisiaria igualmente mediante el RIA lo que mejor en forma importante la sensibilidad para el diagnstico de esta enfermedad; sin embargo, los niveles de deteccin de la prueba se encontraban en el orden de 1 IU/ml lo que haca que la prueba no fuera sensible para valores menores de 1 IU/ml. Debido a esto se crearon tcnicas de segunda generacin mediante la cuantificacin de TSH por anticuerpos monoclonales y RIA, el IRMA (Immuno Radiometric with Monoclonal Antibodies) que permiti detectar valores de TSH en rangos de 0.1 IU/ml; posibilitando desde entonces diagnosticar pacientes con

hipertiroidismo primario; pero con la limitante de que para esta tcnica era imposible detectar valores de TSH menores de 0.1 IU/ml por lo que se creo la medicin de TSH mediante quimioluminiscencia o mtodos enzimticos, es decir las tcnicas de tercera generacin, las cuales pueden detectar valores de TSH de 0.01 IU/ml; con lo que se logra el espectro ideal para una prueba de laboratorio que tiene la capacidad de diagnosticar tanto la hipofuncin como la hiperfuncin(20). Adems el avance no slo fue en la medicin de TSH sino tambin en las hormonas tiroideas que han evolucionado simultneamente con la TSH y ya se miden incluso las fracciones libres de hormonas y las fracciones totales, lo que ha facilitado el manejo de estos pacientes. Gracias a esta evolucin en tcnicas de laboratorio, el diagnstico de hipotiroidismo primario es bastante sencillo. Niveles de TSH superiores al valor mximo de la tcnica seran diagnsticos de la disfuncin; pero no es tan fcil. Cuando tenemos un paciente con toda la sintomatologa del hipotiroidismo y la TSH se encuentra elevada el diagnstico es obvio; pero podemos tener pacientes con sntomas muy inespecficos como depresin y con examen fsico normal a quienes se les encuentran valores de TSH por encima del lmite superior y con hormonas tiroideas normales. Se trata de un hipotiroidismo o es un valor ligeramente elevado ocasional de una persona sana(21). Igualmente tenemos otra circunstancia que ha sido descrita con mayor frecuencia: pacientes con valores de TSH en el lmite superior normal y con dislipidemia a quienes se les da tratamiento con hormonas tiroideas y su dislipidemia se corrige manteniendo valores de TSH en rangos normales. Todas las circunstancias anteriores han hecho que aparezca en el hipotiroidismo primario la expresin de hipotiroidismo subclinico, que ha sido objeto de reuniones y congresos dedicados exclusivamente a este tema. La sociedad Europea de Tiroides hace algunas recomendaciones para el manejo de esta situacin que se consideran tiles como gua (Tabla).

Tabla. Enfoque del paciente con disfuncin tiroidea de acuerdo a los niveles de hormona estimulante de la tiroides (TSH). Si TSH 0.4 a 2.0 mU/L Si TSH 2.01 a 5.0mU/L

Si TSH < 0.4m U/L

Si TSH mU/L

>

5.0

Normal, Medir T3 y T4 totales o Repetir Dar tratamiento libres para diagnostico cada cinco Medir T4 libre y anticuerpos para de hipertiroidismo. aos antitiroideos hipotiroidismo 1. Si AAT (-) y T4 libre es normal repetir screening cada ao. Si TSH es > 4.0mU/l en dos ocasiones dar tratamiento 2. Si AAT (+) y/o T4 libre esta baja o normal baja tratar si

TSH es mayor de 3.0 mU/l y observar a los otros Tomado de Koutras DA. Subclinical hypothyroidism. En G. Hennemann, E.P. Krenning, Thyroid International Merck KGaA, Darmstadt 1999 (3), 6-9

6.- Femenino de 26 aos su padecimiento actual inici hace 4 aos, cuando sus ciclos menstruales empezaron a ser irregulares. Su ritmo actual es de 40-90 x 3-4. FUR: hace 3 meses. Inici vida sexual a los 23 aos y no ha podido embarazarse. En la exploracin fsica encontramos la piel ligeramente seca, hay salida de lquido blanquecino escaso a la expresin del pezn izquierdo y tiene giordano positivo derecho. El estudio que es de mayor utilidad para aclarar el diagnstico es: a) b) c) d) Prueba de embarazo LH, FSH y estrgenos Tomografa de crneo Prolactina

En toda paciente con galactorrea, trastornos menstruales, hirsutismo, disminucin de la libido o infertilidad, deben determinarse los niveles de PRL plasmtica, entre el tercer y quinto da del ciclo menstrual, si este es regular o en cualquier momento, si presenta amenorrea u oligomenorrea.2 Si en la primera determinacin se obtienen cifras elevadas debe repetirse y si se confirma nuevamente, se establece el diagnstico de hiperprolactinemia. Una historia clnica com-pleta, con un interrogatorio y examen fsico detallados, permitir orientarnos hacia la causa de la hiperprolactinemia. La primera causa que se debe descartar antes de emprender otros estudios, es el embarazo. El uso de estrgenos, anticonceptivos orales y drogas que aumentan la secrecin de PRL debe precisarse en el interrogatorio, as como la presencia de quemaduras u otras lesiones en trax que se buscan en el examen fsico.

Pocas mediciones hormonales tienen el significado clnico que se observa con la prolactina. La tcnica est bien estandarizada y presenta bastante confiabilidad; la muestra de sangre se puede obtener a cualquier hora del da y debido a las variaciones fisiolgicas (efecto del ejercicio, alimentos, irritacin de la pared costal) cuando se obtiene una cifra de prolactina por arriba de lo normal, es necesario repetir el anlisis. Varios frmacos pueden producir una moderada elevacin en los niveles de prolactina, sin alcanzar los valores que se encuentran en prolactinoma; las drogas ms comunes son metoclopramida, fenotiazinas, risperidona, verapamil, metildopa, reserpina y los inhibidores de MAO. Exmenes: prolactina plasmtica (prolactinemia), tirotropina (hormona estimulante de la glndula tiroides o TSH), test de embarazo. Prolactina menor de 100 ng/ml: baja sospecha de prolactinoma. Prolactina entre 100 y 300 ng/ml: mayor posibilidad de prolactinoma.

Prolactina superior a 300 ng/ml: alta sospecha de prolactinoma. Es precisa resonancia magntica nuclear (RNM) de hipfisis (detecta prolactinoma mayor de 3mm). En el 50% de los casos de hiperprolactinemia se detecta tumor: microadenomas o macroadenomas; la prolactina suele encontrarse entre 100 y 200ng/ml; los macroadenomas tienen efecto de masa y causan hipopituitarismo; los microadenomas no presentan efecto de masa y slo producen hipogonadismo. Referencias: 1. Schlechte JA. Prolactinoma. N Engl J Med 2003;349:2035-2041. 2. Zrate A, Canales ES, Jacobs LS, Soria J, Daughaday WH. Restoration of ovarian function in patients with the amenorrhea-galactorrhea syndrome after long-term therapy with L-Dopa. Fertil Steril 1973;24:340. 3. Tyson JE, Carter JN, Andreassen B, Huth J, Smith B. Nursing mediated prolactin and luteinizing hormone secretion during puerperal lactation. Fertil Steril 1978;30:154. 4. Schlechte JA, Sherman BM, Chapler FK, VanGilder J. Long-term followup of women with surgically treated prolactin-secreting pituitary tumors. J Clin Endocrinol Metab 1986;62:1296-301. 5. Losa M, Mortini P, Barzaghi R, Gioia L, Giovanelli M. Surgical treatment of prolactinsecreting pituitary adenomas: early results and long-term outcome. J Clin Endocrinol Metab 2002;87:3180-3186.

7.- Femenino de 25 aos, acude a urgencias por cuadro de 3 das de evolucin caracterizado por dolor en cuadrante superior derecho, de inicio insidioso, intermitente, opresivo, sin irradiacin y de intensidad 7/10. Ingiere paracetamol para cefalea ocasional y anticonceptivos orales desde hace varios aos. El ultrasonido demuestra una imagen bien delimitada, hiperecica de 2 cm de dimetro en el lbulo heptico derecho. El diagnstico ms probable en esta paciente es? a) b) c) d) Carcinoma hepatocelular. Adenoma heptico. Quiste heptico. Metstasis heptica.

Los adenomas hepticos (AH) son proliferaciones benignas de hepatocitos, que suelen presentarse en mujeres entre los 20 y los 40 aos de edad y con antecedentes de toma de anticonceptivos orales. Infrecuentemente se presentan en varones, siendo la proporcin de varones: mujeres de 1:11. El sntoma ms frecuente es el dolor abdominal vago crnico, que si se localiza en el cuadrante superior derecho suele ser por una hemorragia intratumoral. Actualmente, las tcnicas de imagen con mejor rendimiento para su diagnstico son la TC helicoidal multifsica y la resonancia magntica (RM).

Hugh TJ, Poston GJ. Benign liver tumors and masses. Surgery of the liver and biliary tract. 3 edicin. London, 2000; tomo 2 p.1397-1422

8.- Cuando los resultados en la manometra esofgica muestran un aumento de la presin basal del esfnter esofgico inferior (EEI) junto a una disminucin o ausencia de su relajacin con la deglucin, estamos frente a un cuadro de:

a) b) c) d)

Acalasia. Esclerodermia con afectacin esofgica. Enfermedad por reflujo gastroesofgico Espasmo esofgico difuso.

Diagnstico de acalasia
Aspecto radiogrfico Un esofagograma puede mostrar la disminucin de peristalsis, la dilatacin del esfago proximal y el estrechamiento del esfago en su parte inferior. El paciente se traga una solucin de bario, con fluoroscopia continua que son grabaciones de rayos X para observar el flujo del fluido a lo largo del esfago, sin que se observe el movimiento peristltico normal del esfago. Hay un agudo estrechamiento en el esfnter esofgico inferior y reduccin del dimetro en la unin gastro-esofgica. La imagen que proyecta se denomina clsicamente en pico de loro o en cola de ratn. Por encima de la reduccin, el esfago a menudo se observa con una dilatacin de diversos grados a medida que poco a poco se va estirando en el tiempo. Por la falta de movimientos peristlticos, se suele observar en la radiografa un margen entre aire y lquido. Manometra esofgica Debido a su sensibilidad, el diagnstico es confirmado por medio de una manometra esofgica, que mide las presiones del esfago mediante una sonda nasoesofgica y permite comparar las presiones en situacin basal y durante la deglucin.4 Se inserta un tubo delgado a travs de la nariz, y se le instruye al paciente a deglutir varias veces. La sonda mide las contracciones musculares en diferentes partes del esfago durante el acto de la deglucin. La manometra revela la falla del EEI para relajarse con cada deglucin y la falta de peristaltismo funcional del msculo liso en el esfago. Para descartar complicaciones se suele acudir a una endoscopia digestiva alta.

Esquema manomtrico de acalasia demostrando contracciones aperistlticas, un aumento de la presin intraesofgica y el fallo de la relajacin del esfnter esofgico inferior. Debido a la similitud en los sntomas, la acalasia se puede confundir con trastornos ms comunes, tales como la enfermedad de reflujo gastroesofgico, la hernia de hiato, e incluso trastornos psicosomticos.

REFERENCIAS BIBLIOGRFICAS 1. Garca Gutirrez A. Acalasia de esfago. http://www.sld.cu/galerias/pdf/uvs/cirured/acalasia..pdf Consultado Mayo 27, 2006. Disponible en:

Fareras Rozman, et al. Acalasia esofgica.Tratado de Medicina Interna. 15 Edicin. Espaa: Ediciones Harcourt; 2003. Seccin 2. Cap 20.p.354-9.

9.- Masculino que acude al servicio de consulta externa presenta prurito y lagrimeo en ambos ojos, hiperemia conjuntival, fotofobia, exudado ms o menos viscoso y formacin papilar en la conjuntiva tarsal. El diagnstico probable es?:

a) Queratoconjuntivitis seca. b) Conjuntivitis alrgica. c) Conjuntivitis bacteriana. d) Conjuntivitis vrica.

Conjuntivitis alrgica. Condicin inflamatoria ocular bilateral, crnica y recurrente, que se presenta predominantemente en la infancia y con mayor incidencia en el sexo masculino. Las exacerbaciones se asocian a ciertas estaciones del ao (donde el nombre de "primaveral") en las cuales se supone que existe una mayor cantidad de alergenos en el medio ambiente; por ejemplo, el polen de las plantas. Estos pacientes frecuentemente presentan historia personal o familiar de atopia. El sntoma principal es el prurito ocular intenso, adems de secrecin mucoacuosa matutina, fotofobia severa y lagrimeo. Clnicamente se observa

intensa hiperemia y quemosis conjuntival bulbar, presencia de papilas gigantes en la conjuntiva tarsal, secrecin mucoide abundante, ocasionalmente se observan infiltrados eosinfilos a nivel de limbo que se conocen como "puntos de Trantas" y en etapas crnicas se aprecia acmulo de pigmento en conjuntiva bulbar y opacificacin corneal secundaria a queratopata punteada. Es frecuente que estos pacientes presenten una sobreinfeccin debido al contacto de las manos con estructuras oculares por el prurito incontrolable. Se ha demostrado presencia de eosinfilos e inmunoglobulina IgE; sin embargo, el diagnstico es esencialmente clnico. El tratamiento se divide en medidas preventivas, paliativas y antiinflamatorias. Las medidas preventivas se orientan a evitar la exposicin directa al sol, utilizacin de visera y lentes oscuros; el tratamiento sintomtico se basa en la aplicacin de fomentos fros, lubricantes y vasoconstrictores tpicos y el uso de antihistamnicos orales. Como antiinflamatorios tpicos en etapas agudas se emplean cursos cortos de esteroides. Los estabilizadores de clulas cebadas (cromoglicato de sodio) y antihistamnicos tpicos tienen efecto a largo plazo. Generalmente estos pacientes presentan mejora importante en la intensidad y frecuencia de los episodios al alcanzar la adolescencia.

Referencias Bibliogrficas: Miller Stephen JH. Parsons Diseases of the Eye. 18th ed. Edinburgh, London, Melbourne and New York: Churchil Livigstone; 1990. Snchez Salorio M, Rodrguez Ares T, Alio y Sanz JL, Pita D, Fontanela JR, Grau M et al. Conjuntivitis, Edika-Med S.A. Barcelona: CIBA VISION, 1992. Foulks GN, Pavan-Langston D. Cornea and External Disease. In: Pavan-Langston D, editor. Manual of Ocular Diagnosis and Therapy. 2nd ed. Boston: Little, Brown & Co. 1991. p. 100102.

10.- 2 year old female, is taken by her mother to the emergency room complaining of intermittent abdominal pain. Medical history of 6 to 24 bloody evacuations in the last 24 hours. Vital signs: BP 110/65, CR 90 pm, RR 28 pm, tempetature 36.5 C. The child is irritable, crying loud and in the physical exploration there is a abdominal mass in the right lower quadrant. Whath would be the most probable diagnosis? a) Meckels diverticulum. b) Apendicular abscess. c) Intestinal intususception. d) Acute appendicitis.

Invaginacin intestinal: Ocurre cuando una porcin del tracto alimentario se pliega dentro del segmento adyacente. Es la causa ms frecuente de obstruccin intestinal entre los 3 meses y los 6 aos de edad. El cociente varn:mujer es 4:1. Se produce de forma sbita con dolor clico paroxstico intenso recurrente a intervalos frecuentes, el 60% de los lactantes expulsan heces que contienen sangre roja y moco, las deposiciones en confitura de grosella. Nelson, Tratado de Pediatria, 17 Edicin, Ed. Elsevier, Pg.1242-1243.

11.- Masculino de 52 aos de edad, jardinero, acude a consulta debido a que presenta unas lesiones en su mano y antebrazo. La lesin inicial fue una ppula pequea, no dolorosa que se expandi lentamente y desarroll un rea central necrtica. Posteriormente el paciente present numerosos ndulos subcutneos en su antebrazo. Los ndulos aumentaron en tamao progresivamente, refiere que uno de los ndulos se ulcer (aproximadamente 1 mes despus de la aparicin de las lesiones). No refiere sntomas sistmicos. Tambin not que los ndulos se encuentran acomodados como en cadena. Cul de los siguientes es el diagnstico ms probable?

a) b) c) d)

Aspergilosis Candidiasis Mucormicosis Esporotricosis

DEFINICIN

subcutnea ms frecuente en todo el mundo.1, 2. Micosis subcutnea, se localiza preferentemente en cara y extremidades, afecta piel y vasos linfticos, se caracteriza por ndulos o gomas que dan lugar a lesiones fijas verrugosas o linfangticas, de evolucin subaguda o crnica,. En raras ocasiones es extracutnea o sistmica afectando pulmn, huesos o articulaciones. EPIDEMIOLOGA Mxico: Sur del D. F.,Puebla, Guanajuato,San Luis Potos y Estado de Mxico. Jalisco y Nayarit. Se han comunicado epidemias familiares y en empacadores de loza. Clima templado y hmedo, promedio entre 20 y 25C y humedad relativa superior a 90%. La mayora de los casos se presenta a finales de otoo y principios de invierno.

Sporothrix schenckii. Es una enfermedad cosmopolita y probablemente la micosis

La esporotricosis es una infeccin subaguda o crnica causada por el hongo dimorfo

Se ha aislado del suelo, detritus vegetales, madera, hojas y ramas ya sean frescas o secas, paja y zacate. Campesinos, jardineros, floristas y carpinteros. Se presenta en igual proporcin en hombres que en mujeres. Grupos de edad ms afectados: nios entre 5 y 15 aos en un 30% de los casos y entre los 16 y los 35 aos de edad en un 50% de los casos. El perodo de incubacin en los casos cutneos es de una semana a un mes. En los raros casos pulmonares se desconoce. Va de ingreso por un traumatismo cutneo.

PATOGENIA Esporotricosis cutnea. La lesin inicial es un chancro esporotricsico, constituido por una lesin nodular o gomosa, ulcerada, dos semanas despus se presenta un complejo cutneo linfangtico, siguen los linfticos regionales, puede involucionar y presentar cura espontnea. Cuando el proceso continua se extiende por contigidad presentando una cadena de gomas eritematoviolceas,no dolorosas que siguen los vasos linfticos regionaleshasta el linftico mayor.

CUADRO CLNICO Linfangtica ascendente de las extremidades

Presencia de lesiones gomosas dstribuidas a lo largo del trayecto linftico, en especial de extremidades superiores, siguiendo de manera paralela la extremidad. CUADRO CLNICO CUTNEO FIJA (20 a 30%) Forma crnica No tiende a la diseminacin Lesin nica Verrugosa o vegetante Borde eritemato-violceo Bordes bien limitados Cubierta con escamas y costras melicricas Asintomtica Tendencia a la curacin

DIAGNSTICO DE LABORATORIO Examen directo no se emplea da resultados negativos Cultivo son el mejor mtodo. Sabouraud y micosel agar Incubacin 28C Crecimiento 5 a 8 das En medios de cultivo ricos (gelosa sangre, BH etc) se obtienen colonias levaduriformes. TRATAMIENTO

Yoduro de Potasio es la terapia de eleccin Dosis Nios 1 a 3 g. al da Dosisn adultos 3 a 6 g. al da 20 g. de KI en 300 cc de agua Frasco obscuro (15 cc de solucin (una cucharada sopera) da una concentracin de 1 g). Tiempo de Tx. 3 meses promedio y continuar 2 meses ms.

Rodrguez GH, Magaa RMC, Jurez L, Arenas R. Esporotricosis cutnea diseminada: comunicacin de un caso. Dermatol Rev Mex 2008;52(5):228-30. La versin completa de este artculo tambin est disponible en: www.revistasmedicasmexicanas.com.mx

12.- Masculino de 52 aos acude al servicio de urgencias por presentar fiebre de 39.0c, exantema mculo-papuloso generalizado, incluyendo palmas y plantas. El paciente labora en el campo ordeando vacas frecuentemente parasitadas por garrapatas. Seale la enfermedad a la que se refiere, el germen causante y el tratamiento adecuado:

a) Kala-azar, Leishmaniae Donovani: Antimoniales. b) Fiebre Q, Coxiella Burnetti, Doxiciclina. c) Fiebre botonosa, Ricckettsia Conori: Doxiciclina. d) Fiebre de Malta, Brucella Mellitensis: Cotrimoxazol.

Las rickettsias son organismos coco-bacilares, Gram negativos, de 2-3 micras de dimetro, intracelulares, incapaces de crecer en ausencia de clulas vivas del husped. El gnero Rickettsia es ubicado taxonmicamente en la familia Rickettsiaceae, junto a otros dos gneros: Coxiella, con la especie C.burnetii, responsable de la fiebre Q, y Ehrlichia con las especies E.chaffeensis, agente de la ehrlichiasis monoctica, y E.phagocytophila, productora de la ehrlichiasis granuloctica humana. Junto a la familia Rickettsiaceae est la familia Bartonellaceae con tres especies principales: Bartonella henselae, agente de la enfermedad por araazo de gato (cat scratch disease); B.quintana, responsable de la angiomatosis bacilar(1), y B.bacilliformis, productora de la bartonelosis o enfermedad de Carrin (verruga peruana). Las rickettsiosis son zoonosis transmitidas desde los huspedes o reservorios animales al hombre a travs de picaduras de artrpodos diversos, que varan con cada enfermedad (piojos, pulgas, garrapatas, otros caros, esencialmente). La rickettsiosis por R.conorii es conocida con el nombre de fiebre botonosa o manchada del Mediterrneo o fiebre de Marsella, siendo transmitida al hombre desde el perro que constituye su reservorio por garrapatas de los gneros Amblyomma y Riphicephalus principalmente. En ellas el germen cumple un ciclo que incluye el pasaje transovrico a los descendientes por lo cual representan tambin otro verdadero reservorio del parsito. Clnicamente la enfermedad se caracteriza por la aparicin en el sitio de la picadura de la garrapata de una lesin inicial indurada con centro necrtico muchas veces (mancha negra o tache noir) rodeada de aureola inflamatoria, seguida de adenopatas regionales de carcter inflamatorio en los das subsiguientes. Concomitantemente, fiebre frecuentemente alta de 39-40C, malestar general, cefaleas a veces intensas, dolores musculares y articulares. Es relativamente frecuente la observacin de un exantema mculo-papuloso que explica el nombre de fiebre botonosa y que puede afectar varios territorios. Es una afeccin endmica en Sudfrica, Europa del Sur y Medio Este (2). El diagnstico se confirma esencialmente por la tcnica de inmunofluorescencia indirecta (IFI) empleando lminas que contienen antgenos de R.conorii y utilizando, siempre que sea posible, sueros pareados obtenidos al inicio del cuadro y 20-30 das despus para investigar la seroconversin. La histopatologa de las lesiones iniciales o taches noires fue estudiada en detalle por Montenegro y colaboradores en 1983(3). En otro trabajo posterior, Montenegro y

colaboradores(4) demuestran en ratones inoculados con R.conorii la importancia crucial de la inmunidad celular con respecto a la humoral en el control de la infeccin experimental y reduccin del ndice de mortalidad. El ratamiento de eleccin es dicloxacilina, alternativas macrlidos y quinolonas (ciprofloxacino). Bibliografa: 1 . Sampaio SAP, Rivitti EA. Dermatologia. So Paulo: Artes Mdicas, 1998: 1155. 2 . Harris RL, Kaplan SL, Bradshaw MW, Williams Jr, Temple W. Boutonneuse fever in american travelers. J Infect Dis 1986; 153:126-8. 3 . Montenegro MR, Mansueto S, Hegarty BC, Walker DH. The histology of taches noires of boutonneuse fever and demonstration of Rickettsia conorii in them by immunofluorescence. Virchows Arch (Pathol Anat) 1983; 400:309-17. 4 . Montenegro MR, Walker DH, Hegarty BC. Infection of genetically immunodeficient mice with Rickettsia conorii . Acta Virol 1984; 28:508-14. 5. Conti Daz IA, Rubio I, Somma Moreira RE, Prez Bormida G. Rickettsiiosis cutneo-ganglionar por Rickettsiaconorii en el Uruguay. Rev Inst Med Trop (So Paulo), 1990;

13.- Masculino de 19 aos tacleado mientras jugaba football se presenta con dolor severo en rodilla izquierda. A la EF la rodilla se encuentra edematizada y el paciente tiene dolor a la palpacin directa del aspecto lateral de la rodilla. Cuando se flexiona la rodilla 30, la aduccin pasiva ocasiona dolor en la misma rea, y la pierna puede ser aducida ms que la pierna contralateral. Cajn anterior, cajn posterior y Lachman negativos. Cul es el sitio ms probable de lesin?

a) b) c) d)

Ligamento cruzado anterior Ligamento colateral lateral Menisco lateral Ligamento cruzado posterior

Se produce por un trauma en varo al golpearle en la parte externa de la pierna por debajo de la rodilla o al caerle otro jugador o contusin en la cara interna de la rodilla. Presenta dolor y generalmente sensacin de parestesia en el territorio del nervio citico poplteo externo por distensin, que puede ser una parestesia transitoria en traumas de menor energa o una lesin definitiva con pie cado en traumas de alta energa. El paciente presenta dolor, derrame leve a moderado y equimosis en cara lateral de la rodilla.

El diagnostico se hace al encontrar: 1. Bostezo en varo en 30 grados de flexin: para evaluar el ligamento colateral lateral, el dolor es en trayecto del ligamento. Puede ser de diferente magnitud: Grado I: muy discreta apertura de la interlinea comparado con lado contra lateral. Grado II: apertura de 5 mm de la interlinea articular. Grado III: apertura de 10 mm de la interlinea articular.

2. Bostezo en varo en extensin completa: para evaluar esquina postero lateral, capsula postero lateral y ligamento fibulo poplteo. Las estructuras laterales tienen un potencial de cicatrizacin menor que las estructuras mediales. En las lesiones grado I y II se debe inmovilizar y proteger el apoyo durante seis semanas. Se debe tener un alto ndice de sospecha y en los casos grado III y donde se presente compromiso de la capsula postero lateral, se debe realizar ciruga para reparo primario en las primeras dos semanas, pues el reparo primario funciona mejor que cualquier reconstruccin anatmica.

14.- Masculino de 25 aos, que 10 das despus de acudir a una despedida de soltero, comienza con inflamacin de rodilla derecha y de ambos tobillos, conjuntivitis bilateral, aftas orales y erosiones superficiales no dolorosas en el glande. El disgnstico ms probable es: a) b) c) d) Enfermedad de Reiter Enfermedad de Still. Infeccin gonoccica. Infeccin por Staphylococcus Aureus.

Proceso inflamatorio estril de la membrana sinovial, precedido o precipitado por una infeccin que ocurre fuera de la articulacin.

DIAGNOSTICO: HISTORIA SNTOMAS GENERALES MANIFESTACIONES MSCULO-ESQUELTICAS Artralgias, artritis aditiva o migratoria Monoartritis u oligoartritis asimtrica Articulaciones grandes que sostienen peso: rodillas, tobillos y caderas Dactilitis o dedos en salchicha Afeccin axial: articulaciones S-I y columna lumbar Entesopata, tenosinovitis MANIFESTACIONES GENITO-URINARIAS Uretritis, balanitis circinada (es importante sealar que las lesiones son indoloras) , prostatitis Cervicitis, cistitis, enfermedad plvica inflamatoria MANIFESTACIONES CUTNEAS Y DE MEMBRANAS MUCOSAS Queratodermia blenorrgica Eritema nodoso Distrofia ungueal lceras orales

MANIFESTACIONES OCULARES Conjuntivitis y uvetis. MANIFESTACIONES G-I

BIBLIOGRAFA RECOMENDADA:

Klippel JH, Stone JH, Crofford LJ, White PH, editors. Primer on the rheumatic diseases. 13th ed. New York: Springer-The Arthritis Foundation; 2008. Martnez-Elizondo P, editor. Introduccin a la Reumatologa. 4a ed. Mxico: Colegio Mexicano de Reumatologa A.C./Intersistemas S.A. de C.V.; 2008. Firestein GS, Budd RC, Harris ED Jr, McInnes IB, Ruddy S, Sergent JS, editors. Kelleys Textbook of Rheumatology. 8th ed. Philadelphia: Saunders Elsevier; 2009.

15.- Masculino de 56 aos que llega al servicio de urgencias con dolor torcico de ms de 12 hrs. de evolucin, que inicia en forma sbita, mejora al estar sentado, no tiene antecedentes de importancia, habr que descartar de primera instancia:

a) b) c) d)

TEP Enfermedad cido pptica Cardiopata isqumica Lesin de grandes vasos

El manejo de un paciente con este tipo de dolor, requiere una evaluacin de la severidad, localizacin y caractersticas peculiares de dicho dolor. Muy pocos sntomas suponen una urgencia tan obligatoria como lo es el dolor torcico. Tanto el mdico como el paciente saben que la isquemia miocrdica puede ser causa de muerte sbita, pudiendo generar ansiedad en ambos. La importancia y dificultad en la valoracin del dolor torcico radica en la multitud de causas posibles y en el diferente pronstico segn la patologa subyacente. Al problema diagnstico inherente a un sndrome esencialmente clnico, se aade la dificultad de etiquetar el dolor en poco tiempo (ayudados slo por la clnica, una Rx de trax y un ECG), dada la importancia de iniciar con prontitud el tratamiento ms adecuado en los pacientes con patologa potencialmente letal. Aunque el dolor o la molestia precordial constituye una de las manifestaciones principales de cardiopata, es muy importante recordar que puede originarse no slo en el corazn, sino tambin en: estructuras cardiacas intratorcicas como la aorta, la arteria pulmonar, rbol broncopulmonar, pleura, mediastino, esfago y diafragma; tejidos del cuello o la pared torcica, incluidos piel, msculos torcicos, regin cervicodorsal, uniones costocondrales, mamas, nervios sensoriales o mdula espinal y rganos abdominales como estmago, duodeno, pncreas o vescula biliar; adems de dolor artificial o funcional. Manifestaciones clnicas en cardiopata isqumica Estas se pueden agrupar en cuatro grandes grupos o categoras, las cuales son:

persistente como para causar muerte del tejido muscular cardaco; hay tres tipos que son la angina estable, la de prinzmetal y la inestable.

Angina de pecho: en donde la obstruccin del riego arterial no es lo suficientemente

Infarto del miocardio: en este caso la obstruccin del riego arterial es lo suficientemente
duradera o persistente como para causar necrosis tisular isqumica del miocardio.

Cardiopata isqumica crnica : son pacientes que generalmente han sufrido uno o ms ataques cardacos y han sobrevivido a ellos, pero continan teniendo problemas cardacos debido a que la parte del miocardio que no muere se hipertrofia para suplir las necesidades del cuerpo y esto a su vez causa un aumento de la demanda cardaca debido al aumento de los componentes estructurales de las clulas cardacas, trayendo ms problemas porque no

se podr suplir adecuadamente al corazn debido a la obstruccin coronaria. Estos pacientes constituyen el 50% de los que reciben trasplantes cardacos.

hora antes de la muerte, o no se presentaron nunca. Causas: aterosclerosis coronaria, estenosis artica, hipertensin sistmica, comnmente arritmias letales (asistlicas y fibrilacin ventricular). BIBLIOGRAFIA:

Muerte sbita cardaca : Es el paro cardaco en el cual se presentaron sntomas en una

1. Goldman L., Braunwald E. Molestias torcicas y Palpitaciones. En Isselbacher KJ., Braunwald E., Wilson JD., Fauci AS., Kasper DL., eds. Harrison, Principios de Medicina Interna. McGraw-Hill. Interamericana de Espaa. 1994. 2. Braunwald E. Tratado de Cardiologa. Interamericana. Mcgraw-Hill. 1993. 3. Harkins SW. Geriatric pain. Pain perceptions in the old age. Clin Geriatric Med 1996. 4. Coto lpez, A., Morales JM., Gutierrez Rodero, F., Gonzalez E., .Dolor Torcico. Manual de diagnstico y teraputica mdica. Gutierrez Rodero F y Garca Daz JD. 2 ed. Madrid, 1990; pag. 165-172. 5. Durn Serantes, M., Caldern de la Barca Gzquez, J.M., Romero Moreno M., Martinez Guilln, J., Montero Prez, FJ., Jimenez Murillo, L., Cardiopata Isqumica ( I ): Angor. Protocolos de actuacin en Medicina de Urgencias. Jimenez Murillo L y Montero Prez FJ. Mosby/Doyma Libros SA. Barcelona 1996; pg. 51-56. 6. James H. Chesebro. La clnica del dolor torcico en el Servicio de Urgencias: abordaje de los pacientes y relacin coste-eficacia. Grandes temas de la cardiologa: avances hacia el cambio de siglo. 1998, American College of Cardiology. 7. Tresch DD, Aronow Ws. Clinical manifestations and clinical diagnosis of coronary artery disease. Clin Geriatr Med. 1996. 8. Owens, G.M.: Chest pain. Primary Care, 1986. 13; pg: 55-61. 9. Rutherford, J.D.; Braunwald, E.: Diagnstico diferencial del dolor precordial. En: Braunwald E: Tratado de Cardiologa, 4 edicin. Interamericana McGraw-Hill, Madrid, 1993; pg: 1448-1449. 10. Williams, E.S.: Approach to the patient with chest pain. En: Kelly WN, ed. Textbook of Internal Medicine. Filadelfia, J.B. Lippincott Company, 1989; pg. 374-379.

16.- Femenino de 34 aos que inicia con debilidad muscular, episodios frecuentes de ptosis, diplopia y fatiga generalizada. Refiere dificultad a la masticacin y deglucin. La prueba ms especfica para corroborar su diagnstico es: a) Electromiograma de fibra muscular aislada. b) Determinacin de anticuerpos anti-receptor de acetilcolina. c) TAC torcico. d) Electromiograma con estimulacin repetitiva.

La miastenia gravis es una enfermedad autoinmune que se caracteriza por presentar debilidad muscular fluctuante y fatiga de distintos grupos musculares. La miastenia gravis afecta a individuos de todas las edades, con una predileccin por mujeres entre los 20 y 40 aos. Los msculos oculares, faciales y bulbares son los ms frecuentemente afectados por la enfermedad. Los pacientes con miastenia gravis manifiestan empeoramiento de la debilidad muscular, con infecciones intercurrentes, fiebre y agotamiento fsico o emocional. La infeccin respiratoria (bacteriana o vrica) es la causa ms frecuente de provocacin. La presencia de anticuerpos contra receptores de acetilcolina en un paciente con manifestaciones clnicas compatibles con la miastenia gravis confirma el diagnstico. El tratamiento de esta entidad es controvertido y debe ser individualizado, ya que no existe un rgimen teraputico uniforme para todos los pacientes. Entre los tratamientos disponibles testacan los frmacos anticolinestersicos, los corticosteroides, la plasmafresis, la inmunoglobulina, los inmunosupresores y la timectoma.

REFERENCIAS 1. Drachman DB. Myasthenia gravis. N Engl J Med 1994;330:1797-8102.2. Engel AG. Myasthenic syndromes. In Engel AG, Franzini-Armstrong C, eds. Myology. 2 ed. New York: McGraw-Hill;1994. P. 1798-835. 3. Engel AG. Ohno K. Milone M, Sine SM. Congenital myasthenic syndromes caused by mutations in acetylcholine receptor genes. Neurology 1997;48 (Suppl 5):S28-35. 4. Steinman L, Mantegazza R. Prospects for specific inmunotherapy in myasthenia gravis . FASEB J 1990;4:2726-31. 5. Massey JM. Treatment of acquired myasthenia gravis. Neurology 1997;48 (suppl 5):S4651.

17.- Masculino de 46 aos de edad que inicia con dolor intenso en fosa renal izquierda ante la sospecha de litiasis renoureteral se realizan estudios de laboratorio y gabinetem, los Rx demuestran clculos radio lcidos, los cuales estn relacionados a: a) b) c) d) Calcio Estruvita cido rico Cistina

-Bruce E. Jarrell, R. Anthony Carabasi, Nacional Medical Series for Independent Study. Wiliams & Wilkins, 3rd Edition: 451-475. Los clculos que sepueden presentar en la va urinaria son formados por calcio, cido rico y cistina, los nicos radiolucidos son los de cido rico que representan un reto diagnstico.

18.- Masculino de 24 aos presenta dolor y tumefaccin del testculo derecho. Su mdico solicita un ultrasonido que revela una masa testicular de 2 x 2.5 cm. Se realizan una exploracin inguinal y una orquiectoma. El estudio histopatolgico revel un seminoma puro. Una tomografa computadorizada de trax, abdomen y pelvis mostr dos ganglios retroperitoneales de 3 cm que estn aumentados de tamao. La biometra hemtica, la qumica sangunea y los marcadores tumorales estn todos dentro de los lmites normales. Cul de los siguientes seria la mejor conducta? a) b) c) d) Extirpacin quirrgica de toda la enfermedad Braquiterapia Observacin Radioterapia

Allen R. M. MMS Medicina Interna. 5. Edicin. National Medical Series. Mc. Graw Hill. 2006. (captulo 4IX G 2). El cncer testicular es el ms comn en varones adultos jvenes variedades ms frecuentes son seminomas y tumores de clulas germinales no seminomatosos; ambos sor rabies aun en etapas avanzadas. Los seminomas son muy sensibles a la radioterapia; por tanto, los pacientes enfermedad de etapa II (la limitada a testculo y a ganglios por abajo del diafragma) pueden tratarse con < bajas de radiacin. Debido a la toxicidad de la mdula sea producida por la radioterapia mediastnica, y est indicada la radiacin profilctica del mediastino. En este contexto, los ganglios linfticos retroperito aumentados de tamao denotan la existencia de enfermedad metastsica y est indicado el tratamiento.

19.- Femenino de 64 aos que acude al servicio de urgencias por referir dolor precordial, nausea y malestar general, ante un probable infarto agudo al miocardio, Cul de los siguientes marcadores tiene mayor sensibilidad para realizar ste diagnstico? a) CPK b) Transaminasa c) Troponina d) Deshidrogenada lctica

Es muy conocida la falta de especificidad de la CPK. La elevacin de la actividad de esta enzima se produce tanto en el infarto de miocardio (IAM) como en afecciones caracterizadas por un grado variable de necrosis muscular. Por ello en los ltimos aos se han realizado, y se siguen realizando, considerables esfuerzos para encontrar nuevos marcadores analticos que sean capaces de diferenciar estos cuadros. Entre ellos, la determinacin de la isoenzima miocrdica de la CPK de forma cuantificada (CPK-MB-Masa) resulta tambin poco especfica ya que se eleva tambin en caso de necrosis de msculo estriado; est descrita su elevacin en traumatismos, rabdomiolisis, convulsiones1, miopatas agudas y crnicas2, insuficiencia renal en dilisis3 e incluso en el ejercicio intenso4,5; y lo mismo podemos afirmar de la mioglobina6. Ms recientemente se han determinado las troponinas T e I en sus isoformas especficas de msculo cardiaco, que tienen una secuencia diferente de aminocidos a las de msculo estriado no cardiaco, lo que permite el desarrollo de inmunoensayos especficos. El complejo de las troponinas T, C, e I est estrechamente unido al filamento de tropomiosina. La T se encarga de la unin a tropomiosina; la C es iniciadora de la contraccin tras unirse al calcio, y la I se llama as por ser inhibidora de la contraccin en reposo7. Esta especificidad ha sido demostrada en varios trabajos que muestran valores normales de troponina-I en procesos que cursan con necrosis de msculo estriado2,8. Adems de ser muy especficas, las troponinas son altamente sensibles en el infarto de miocardio. Mair y col encuentran una sensibilidad del 100% para el diagnstico si se hace la determinacin de troponina-I pasadas 6 horas del comienzo del dolor9. La cintica de estos marcadores en pacientes con IAM puede resumirse de esta forma: 1. La CPK total comienza a elevarse a las 4-8 horas del comienzo de los sntomas, y permanece elevada hasta que se normaliza a las 48-72 horas10. 2. La isoenzima MB de la CPK (CPK-MB) aparece en el suero tres horas despus del comienzo del IAM, con un pico mximo a las 18-20 horas, alcanzando valores 16 veces superiores al normal. A partir de este punto desciende lentamente y persiste elevada al menos 2 das11. 3. La troponina-I se eleva a partir de las 2-3 horas del comienzo de los sntomas, con un valor mximo a las 16 horas. Desciende bruscamente hasta las 48 horas, y a partir de entonces se produce un lento descenso; puede detectarse todava el 7-8 da11. 4. La mioglobina es la primera que se eleva. Da las cifras ms altas de sensibilidad en el plazo de dos horas del comienzo del dolor10 con respecto a la troponina-I y la CPK-MB. Alcanza su pico a las 8 horas y desciende bruscamente a valores normales a las 18 horas11 Utilidad de la troponina-I, CPK-MB y mioglobina en el diagnstico del infarto de miocardio y de los procesos de necrosis muscular de origen no cardiaco

J.M. Olite3, I. Idoate3, I. Berrozpe1, E. Arina1, L. Metola1, J. Sesma1

Use of troponin-I, CPK-MB and myoglobin in the diagnosis of myocardial infarct and processes of muscular necrosis of non-cardiac origin J.I. Ibez1, R. Sobrado1, M. Rivero2,

1.

Unidad de Urgencias.

2. Servicio de Medicina Interna. 3. Servicio de Bioqumica Clnica. Hospital Virgen del Camino Pamplona.

20.- Paciente masculino de 42 aos, con antecedente de alcoholismO Y DM tipo 2, hospitalizado por TCE. Durante su estancia intrahospitalaria inicia con fiebre en picos (38.5-39 C), escalofros, aparicin de soplo, la biometra hemtica muestra leucocitosis (16,700) se descarta IVU y neumona. Se sospecha endocarditis infecciosa: La conducta inicial es:

a) b) c) d)

Tomar hemocultivos seriados Ecocardiograma transtorcico Hemocultivo a travs de catter Retiro de catter intravascular

La endocarditis infecciosa es una infeccin microbiana que se localiza sobre las vlvulas cardacas o sobre el endocardio mural. A pesar que la mayora de estas infecciones son causadas por bacterias, se considera ms apropiado denominarla en forma global como endocarditis infecciosa debido a que tambin puede ser producida por hongos, rickettsias o clamidias. La endocarditis bacteriana es la forma ms reconocida y se clasifica en aguda o subaguda segn su presentacin clnica. La endocarditis mictica se detecta casi exclusivamente en drogadictos o en pacientes con prtesis valvulares. Hemocultivos con antibiograma Se recomiendan tomar tres muestras a la llegada del paciente (preferiblemente sin tomar antibiticos), las muestras de sangre separado por un intervalo mnimo de 1 h, en un perodo de 24 horas; al da siguiente debe repetirse el mismo proceder. Debe realizarse una correcta antisepsia previa de la piel y cambiar de aguja cuando se vaya a verter la sangre en los frascos. El cultivo de sangre se realiza en un tubo especializado para hemocultivos automatizados. Se prefiere dejar al paciente, si el estado clnico as lo tolerara, hasta un mnimo de 72 horas sin antibiticos, antes de tomar la muestra, pero si el paciente por su situacin de sepsis avanzada y repercusin hemodinmica no se le puede retirar el antibitico entonces entonces se utiliza el frasco de hemocultivo automatizado con concentracin mnima inhibitoria del antibitico, precisando en la orden el los tipos de antibiticos que se estn empleando en la teraputica antimicrobiana. Si al realizar los hemocultivos automatizados estos todos vienen negativos a pesar de que la sintomatologa del paciente no mejora se pueden indicar los hemocultivos automatizados

con medios de cultivos especiales para grmenes de crecimiento lento, por ejemplo en medio de Sabureao. A estos frascos se le aade sangre venosa, entre 5-10 mL en adultos. Se llevar la muestra lo ms rpido posible al laboratorio de Microbiologa. La toma de muestra se realizar independientemente de la temperatura del paciente o cuando se presentan los escalofros y no en pico febril. Para confirmar criterio de curacin de la infeccin previo al egreso hospitalario se realizan 6 hemocultivos automatizados una semana despus de suspendidos los antibiticos.

La endocarditis infecciosa (EI) es una enfermedad en la que uno varios mi-croorganismos infectan el endocardio, las vlvulas las estructuras relaciona-das, generalmente sobre una lesin ante-rior: cardiopata reumtica en otros tiempos, cardiopatas congnitas en la actualidad. Puede ser sbita y aguda, pero ms frecuentemente es subaguda y larvada lo que retrasa el diagnstico. Tiene morbilidad y mortalidad impor-tante a pesar de los tratamientos antimi-crobianos y de la difusin de su profi-laxis entre los nios susceptibles. Nue-vos grupos de riesgo son los pacientes sometidos a ciruga cardiaca, los nios que precisan catteres intravasculares, especialmente neonatos, los inmunodeprimidos y los adictos a drogas por va intravenosa. La ecocardiografa- Dop-pler aporta actualmente criterios bsicos para el diagnstico y seguimiento. Pue-den ser tiles a nios nuevos recursos en el tratamiento y una profilaxis precisa.

21.- Femenino de 34 aos con diagnstico de hirsutismo asociado a ovario poliqustico, usted elige el siguiente frmaco para su tratamiento por ser el ms adecuado:

a) Clomifeno b) Estrgenos c) Acetato de ciproterona d) Corticoide

El acetato de ciproterona parece ser ms efectivo que otros frmacos para el hirsutismo en mujeres causado por la produccin ovrica excesiva de andrgenos Una de las causas de hirsutismo (crecimiento piloso excesivo) en mujeres es la hiperproduccin de andrgenos a partir del ovario. Varios frmacos pueden utilizarse para contrarrestar los efectos del andrgeno. El acetato de ciproterona es un frmaco antiandrognico. Los efectos adversos informados con su uso fueron aumento de peso, depresin, fatiga, sntomas mamarios y disfuncin sexual. La revisin de los ensayos encontr que el acetato de ciproterona parece ejercer un efecto en el hirsutismo similar a otros frmacos utilizados para el tratamiento del hirsutismo por exceso de andrgenos. No existen pruebas suficientes para comparar los efectos adversos de las opciones de tratamiento.

Van der Spuy ZM, le Roux PA. Acetato de ciproterona para el hirsutismo (Revisin Cochrane traducida). En: La Biblioteca Cochrane Plus, nmero 4, 2007. Oxford, Update Software Ltd. Disponible en: http://www.update-software.com. (Traducida de The Cochrane Library, 2007 Issue 4. Chichester, UK: John Wiley & Sons, Ltd.).

22.- Gestante de 9 semanas la cual tiene contacto con un menor que, 6 das ms tarde, desarrolla un cuadro de exantema y sndrome general infeccioso sugerente de infeccin por virus de rubola. En el primer control serolgico gestacional se detect la negatividad de la IgG especfica. De las siguientes afirmaciones la correcta es:?

a) La aparicin de IgM materna positiva constituye indicacin para la determinacin de la IgM fetal. b) No existe posibilidad de contagio dado que el nio ya no se hallaba en fase de eliminacin viral. c) En este perodo de la gestacin, el riesgo de la infeccin y de afectacin embrionaria es mnimo. d) La existencia de IgM fetal negativa excluye en este caso la posibilidad de transmisin transplacentaria.

INFECCIONES EN EMBARAZADA VACUNACIONES: CONTRAINDICADAS: - Parotiditis - Rubola - Sarampin - Fiebre amarilla - NO se RECOMIENDA: gripe, poliomielitis, hepatitis B. S SE PUEDEN DAR (cuando estn indicadas): - Fiebre tifoidea - Rabia - Ttanos - Tos ferina (ver pregunta mir, que pone que no) TOXOPLASMOSIS: 50% de transmisin. Si grave (en 1er T, pero es menos frecuente): aborto, parto pretrmino, muerte fetal intratero. Dx: lo de siempre. Calcificaciones cerebrales. TTO: - En casos de seroconversin ESPIRAMICINA (depresin medular, ac. folnico) ht final del embarazo.

RUBEOLA: Contagio >80% si la madre se contagia en 8 primeras semanas. SORDERA CONGNITA. Acs <1/16 susceptibilidad para infeccin. La aparicin de IgM materna positiva es indicacin de determinacin de IgM fetal. La madre no debe quedarse embarazada en los 3 meses siguientes a la vacunacin. Profilaxis de EXPOSICIN: gammaglobulina, solo eficaz en primeros 7-8d.

Si infeccin fetal: PIRIMETAMINA Y SULFADIAZINA en ciclos de 3 semanas alternando con la espiramicina ht final del embarazo.

CITOMEGALOVIRUS: Insospechada. Inclusiones citomeglicas en OJO DE BHO en clas de tejidos afectados. SFILIS: Prueba sexolgica a todas las gestantes. Si se infecta, en los 3 meses neonatales: lesiones CUTNEOMUCOSAS, OSTEOCONDRITIS Y HEPATOESPLENOMEGALIA. DX: screening VDRL o RPR (no treponmicas, pero el embarazo es la primera causa de falso positivo) certeza FTA-Abs o MHA-TP (treponmicas) VARICELA: Si en 1er T 2%; gravemente teratognica. En perodos ms avanzados o periparto enfermedad sistmica generalizada o SNC.

HEPATITIS B: Se infecta el hijo si: - Madre PORTADORA CRNICA - INFECCIN ACTIVA durante la gestacin - Madre con HEPATITIS CRNICA ACTIVA HBsAg positivo (slo) riesgo bajo de transmisin placentaria HBsAg + HBeAg 90% de transmisin *El riesgo de cronificacin ser muy alto si se adquiere en el perodo perinatal. *Especial vigilancia del crecimiento fetal. *Se realizar profilaxis ACTIVA y PASIVA *La gestacin NO aumenta el riesgo de curso clnico grave.

RUBOLA Y EMBARAZO Existen tres situaciones claramente diferenciadas que exigen planteamientos diagnsticos distintos: Determinacin de la inmunidad frente a rubola en la gestante, sin sospecha clnica ni epidemiolgica de padecer la enfermedad: El objetivo de este estudio es conocer si la gestante est protegida, de una posible infeccin por el virus de la rubola, durante el embarazo. Se recomienda la determinacin cualitativa de anticuerpos totales o de IgG especfica, en la primera consulta de control del embarazo. Se desaconseja expresamente la evaluacin cuantitativa de los resultados, ya que no proporciona ninguna informacin til. La presencia de anticuerpos refleja contacto previo con el virus, y por tanto inmunidad, haciendo innecesaria la realizacin de nuevos controles en embarazos sucesivos. A pesar de que se describe que la rubola puede cursar de manera asintomtica, F. de Ory et al estudian 185 sueros de 101 mujeres embarazadas con presencia de IgM y slo confirman la existencia de primoinfeccin en tres de las mismas, asocindose siempre a datos clnicos o epidemiolgicos compatibles, por lo que es desaconsejable la realizacin sistemtica de IgM a las embarazadas. Si la mujer embarazada es seronegativa, deber adoptar las precauciones necesarias para evitar la exposicin al virus y debe ser vacunada frente a la rubola en el post-parto inmediato. Sospecha clnica de infeccin aguda durante el embarazo: Este caso puede plantearse ante la existencia de una clnica compatible en la embarazada, o por exposicin a un sujeto con infeccin aguda por rubola. La presencia de IgG en ausencia de IgM indica que la mujer est protegida, por vacunacin o por infeccin antigua y por tanto no deben realizarse ms determinaciones. La demostracin de seroconversin, con ausencia de anticuerpos en el primer suero y presencia de stos en el segundo, obtenido 15-21 das despus, es la forma ms segura de diagnosticar una primoinfeccin por este agente. Sin embargo, si el primer suero de la enferma presenta anticuerpos, aunque se produzca un incremento del ttulo de estos en el segundo suero, puede ser debido a una reinfeccin. La presencia de IgG y de IgM especfica en una paciente, nos hace sospechar la presencia de primoinfeccin, sin embargo, debemos tener en cuenta varios aspectos: a.- La IgM puede tener reacciones heterlogas entre rubola y otros virus como EBV, CMV, Parvovirus B 19 y virus del sarampin (por reacciones cruzadas o por estimulacin policlonal de linfocitos de memoria); por tanto es necesario confirmar su presencia, siendo la tcnica de ELISA de captura la que presenta mejor especificidad y sensibilidad.

b.- La IgM puede aparecer durante las reinfecciones, pero a ttulos bajos y durante poco tiempo. c.- En un pequeo porcentaje de personas, la IgM puede mantenerse positiva en suero hasta 6 meses. Thomas et al, detectan la presencia de IgM en el 9% de los casos a los 3 4 meses de la infeccin aguda. El estudio de la avidez de la IgG diferencia si la IgG es de aparicin reciente (baja avidez se asocia a infeccin primaria aguda) o si hay ausencia de infeccin primaria (IgG de alta avidez), puede ser una tcnica que ayude a valorar la presencia de IgM y puede colaborar en la diferenciacin entre primoinfeccin y reinfeccin. F. de Ory et al. estudian mltiples patgenos y comunican que esta tcnica presenta una sensibilidad entre el 81 y el 100% y una especificidad del 100%. Tambin se est valorando la utilidad en el diagnstico de la IgA, aunque los datos no son an concluyentes. Todos estos datos serolgicos deben ser interpretados junto con los datos clnicos de la embarazada, en el caso de que los haya y junto con los datos que podamos obtener de la posible fuente de infeccin.

Referencias:
BOSMA TJ, CORBETT KM, OSHEA S, BANATVALA JE, BEST JM. PCR detection of rubella virus in clinical samples. J Clin Microbiol 1995; 33:1075-1079. BOSMA TJ, CORBETT KM, OSHEA S et al. Use of PCR for prenatal and postnatal diagnosis of congenital rubella. J Clin Microbiol 1995; 33:2881-2887. DE ORY F, CASAS I, DOMINGO CJ, ECHEVARRA JM. Application of fluoroimmunoassay to the identification of low avidity specific IgG against pathogenic human viruses and Toxoplasma gondii. Clin Diagn Virol 1995; 3:323-332. DE ORY F, DOMINGO CJ. Los anlisis de avidez de la IgG especfica en el diagnstico de la infeccin por el virus de la rubola. Med Clin (Barc) 1996; 107:118. DE ORY F, ECHEVERRA JM, DOMINGO CJ. Cribado rutinario de IgM especfica antirrubola en mujeres embarazadas: una prctica desaconsejable. Prog Obstr Ginec. Dic 1998; 41:574-578. ENGLUND J, GLEZEN WP, PIEDRA PA. Maternal immunization against viral disease. Vaccine 1998; 16:1456-1463. FREY TK, ABERNATHY ES, BOSMA TJ et al. Molecular analysis of rubella virus epidemiology across three continents: North America, Europe and Asia, 1961-1997. J Infect Dis 1998; 178:642-650.

23.- Recin nacido con sospecha de menigoencefalitis por clnica y un lquido cefalorraqudeo con aumento de leucocitos, aumento de protenas e hipoglucorraquia en el que no se tiene an un germen aislado. El tratamiento de eleccin es: a) b) c) d) Dicloxacilina y amikacina Penicilina sdica cristalina Ampicilina y amikacina Vancomicina

La meningoencefalitis es definida como la inflamacin de las meninges y el encfalo. Dentro de las causas de la meningoencefalitis purulenta la ms frecuente es la bacteriana. La etiologa vara dependiendo del grupo de edad, en menores de 2 meses el ms comn es Streptococcus del grupo b, despus de esta edad el ms frecuente es el S. pneumoniae. El cuadro clnico se caracteriza por 4 sndromes: infeccioso, de hipertensin endocraneana, menngeo y de dao neuronal. Las complicaciones pueden ser agudas, subagudas y crnicas. El diagnstico se hace con el estudio citoqumico y cultivo del LCR. Las medidas teraputicas irn encaminadas al manejo de los signos y sntomas componentes de los 4 sndromes. El tratamiento especfico se iniciar en forma emprica y se modificar de acuerdo a los resultados del cultivo. En la meningitis bacteriana el aspecto macroscpico LCR es turbio o incluso purulento debido al mayor contenido de clulas y protenas. En la mayora de los casos de meningoencefalitis bacteriana el nmero de leucocitos es superior a 500 x mm3; y el predominio de clulas polimorfonucleares es mayor a 50%. La presencia de hipoglucorraquia (< 50% de la glicemia central o glucorraquia < 40 mg/dL) e hiperproteinorraquia moderada (entre 200 y 500 mg/dL) sugiere etiologa bacteriana. Sin embargo, en casos con cuadros neurolgicos de evolucin ms prolongada, mayor de 10 das, el diagnstico diferencial debe establecerse con meningoencefalitis de etiologa tuberculosa.

Enfermedades infecciosas y microbiologa Nmero number 1 enero-marzo January-march 2002 volumen volume 22 Meningoencefalitis bacteriana Fortino solrzano santos,* mara guadalupe miranda novales,** Rita d daz ramos*

24.- RN 16 meses de edad e inici con tos seca, rinorrea hialina, temperatura axilar de 37.8C. Por la tarde muestra tos intensa y en accesos, dolor subesternal al toser, estridor inspiratorio; al explorarle se le observa plido, con aleteo nasal, hundimiento intercostal bilateral a la inspiracin y taquicardia. Faringe hipermica roja; en trax se escuchan algunos estertores gruesos diseminados en ambos hemitorax y disminucin del murmullo vesicular. Con estos datos usted piensa que se trata de: a) Bronquiolitis. c) Neumona bilateral. d) Laringotraqueobronquitis. e) Cuerpo extrao en vas areas.

La laringotraqueobronquitis es una de las causas ms frecuentes de obstruccin de la va area. El agente etiolgico ms comn es el virus parainfluenza tipo 1 y 2, aunque otros agentes virales se pueden encontrar (VRS, Influenza, rinovirus, etc). El edema y la inflamacin de la regin infragltica son los responsables del cuadro obstructivo. El croup espasmdico es una entidad similar a la laringotraquetis viral, pero sin la fase prodrmica de infeccin viral y el cuadro febril. Se ha postulado que esta entidad representa una reaccin de hipersensibilidad a una infeccin previa, en general al virus parainfluenza. Los nios afectados suelen tener antecedentes de atopia, pueden desarrollar sibilancias y signos de hiperreactividad bronquial. Los sntomas y signos se detallan en la Tabla 1, en la que se hace una comparacin con la presentacin clnica de la Epiglotitis aguda.

Tabla 1. Diagnstico diferencial de laringotraqueobronquitis (LTB) y epiglotitis. Epiglotitis causa edad obstruccin inicio fiebre disfagia hipersalivacin postura compromiso txico tos voz frec. respiratoria curso clnico > 2 aos supragltica brusco (horas) alta si si sentado si no nasal normal o aumentada corto Laringotraqueobronquitis 6m a 3 aos subgltica gradual baja no no decbito no si, perruna ronca aumentada largo

bacteriana (H.Influenzae) viral

Croup leve: se recomienda la ingesta de lquidos, antipirticos, humidificacin de secreciones, observacin domiciliaria de los signos y sntomas de empeoramiento.4 Queda a criterio mdico el uso de Dexametasona 0.15 mg/Kg dosis nica, va oral o Prednisolona 1 mg/Kg va oral durante 3 das. Si existiere intolerancia oral se puede utilizar la va intramuscular.4,6 Croup moderado: Se puede utilizar Budesonide (2 mg.) nebulizado, L-adrenalina 1/1000 nebulizada, ms dexametasona oral (0.5 mg cada 8 horas), segn la evolucin.4 Croup severo: lo importante es evitar la necesidad de intubar al nio, se puede aplicar dexametasona por va parenteral, sumada a adrenalina nebulizada (hasta 3 aerosoles seguidos) y Budesonide nebulizado de apoyo. En caso de no haber respuesta favorable se proceder a entubar al paciente.4 Complicaciones: Ocurren en una minora de los casos y son generalmente secundarias a la entubacin: neumotrax, neumomediastino, superinfeccin bacteriana, estenosis subgltica, syndrome txico producido principalmente por Staphilococo aureus.8

CUADRO CLNICO El cuadro inicial es el de una rinofaringitis e incluye irritacin nasal, coriza, fiebre generalmente menor de 39 C, tos seca y odinofagia, con poca o nula afeccin del estado general; 24 a 48 horas despus aparece disfona, la tos se hace "crupal" (traqueal, perruna o en ladrido) y se presentan en forma gradual estridor inspiratorio creciente (estridor larngeo) y signos de dificultad respiratoria de intensidad variable. En la exploracin fsica son evidentes la disfona, el estridor larngeo que en ocasiones es audible a distancia, la disminucin del murmullo vesicular y datos de dificultad respiratoria.1-6 Forbes describe la progresin del cuadro clnico de la siguiente manera: Etapa 1: Fiebre, ronquera, tos crupal y estridor inspiratorio al molestar al paciente. Etapa 2: Estridor respiratorio continuo, participacin de msculos accesorios de la respiracin con retraccin de costillas inferiores y de tejidos blandos del cuello. Etapa 3: Signos de hipoxia e hipercapnia, inquietud, ansiedad, palidez, diaforesis y taquipnea. Etapa 4: Cianosis intermitente, cianosis permanente, paro respiratorio. Por lo general la enfermedad es leve, en pocas ocasiones progresa ms all de la etapa 1, ms del 95% reciben tratamiento ambulatorio; del 5% que requiere hospitalizacin slo del 1 al 1.5% requieren medidas de apoyo ventilatorio (intubacin endotraqueal o traqueostoma). La duracin del padecimiento es muy variable, desde tres a siete das en casos leves, hasta siete a catorce das en casos graves.

BIBLIOGRAFIA 1- Behrman R, Kliegman R, Jasn H. Tratado de Pediatria Nelson. 17 Edicin. Espaa. Elsevier 2004:1508-1503 2- Behar RR. Prevencin de las infecciones respiratorias agudas. Presente y futuro. La Habana Cuba. Hospital Peditrico Universitario William Soler Septiembre 2002; Boletn N 2 3- Mendoza A, Mejas H, Schmidt G. Educacin Continua en Pediatra. Croup. Revista de Pediatra. 2002; 40(2):46-51 4- Basanta A. Croup. Anales de la Pediatria 2003; Monog. 1(1):55-61 5- Kimpen Jann L. Management of respiratory syncytial virus infection. Curs Opin Infect Dis 2001; 14:323-8.

6- Hall C. Respiratory Sincytial and Parainfluenza Virus. N Engl J Med 2004; 344:1917-28 7- Wright R, Pomerantz W, Curia J. New Approaches to respiratory infections in children. Bronchiolitis and croup. Emerg Med Clin North Am 2002;20:93-114 8- Muiz A. Croup. Emedicine [en lnea] 2004 [fecha de acceso 10 de marzo del 2006]; 11:287-9.

25.- Masculino de 44 aos acude a consulta con estudios de laboratorio que reportan hipercalcemia el paciente se refiere asintomtico. El resto de sus exmenes de laboratorio muestran elevacin de parathormona, baja en fsforo, elevacin de cloro,BUN y creatinina normales. El calcio urinario est elevado. La etiologa ms probable en ste paciente es? a) b) c) d) Mileoma mltiple Hiperparatiroidismo primario Hipervitaminosis Sarcoidosis

Al grupo de sndromes que tienen como caracterstica comn la secrecin excesiva y no controlada de hormona para-tohormona (HPT) por una o ms paratiroides funcionantes se le conoce como hiperparatiroidismo primario (HPP). La causa ms frecuente es el adenoma y le sigue en frecuencia la hiperplasia. El cuadro clnico del HPP afecta principalmente al sistema seo, al rin y al sistema gastrointestinal. El cuadro clnico se caracteriza por debilidad muscular, fatigabilidad fcil, manifestaciones gastrointestinales y depresin. Cuando hay afeccin renal el paciente presenta hipercalciuria (25%), litiasis renal recidivante (25%) y/o nefrocalcinosis (20%). Cuando hay resorcin subperistica, tumores pardos en huesos largos o en el maxilar inferior y a nivel del crneo, las lesiones osteolticas le dan el aspecto de sal y pimienta. En los exmenes de laboratorio hay hipercalcemia, hipofosfatemia, fosfatasa alcalina elevada (fraccin sea), elevacin de HPT (90%), osteocalcina y desoxipiridinolinas; as como calciuria e hiperfosfaturia. Cuando hay afeccin del sistema gastrointestinal hay lceras y hemorragias. Por lo tanto, como podemos observar, se trata de una paciente con HPP clsico con afeccin del sistema seo y del rin, sin manifestaciones gastrointestinales aparentemente, el cual al someterse a tratamiento quirrgico y mdico adecuados hubo mejora inmediata que ha continuado hasta la fecha actual. Hiperparatiroidismo primario Ma. Esther Gutirrez Daz Ceballos,1 Hctor A. Rodrguez Martnez,1 Evelyn M. Torres Acosta,1 Humberto Cruz Ortiz1 1 Unidad de Patologa y Servicio de Endocrinologa del Hospital General de Mxico y de la Facultad de Medicina, UNAM

26.- Masculino de 32 aos que inici con disfona y disfagia, es referido al servicio de endocrinologa donde es confirmado el diagnostico de ndulo tiroideo nico, El estudio recomendado es? a) Gammagrama tiroideo b) Tomografa computada de cuello c) Biopsia por aspiracin con aguja fina d) Repetir Pruebas de funcin tiroidea

El ndulo tiroideo solitario se define como el crecimiento localizado de la glndula tiroides, usualmente es benigno, la prevalencia es del 4 al 7% en la poblacin general. Aunque el cncer tiroideo es el tumor endocrino ms comn slo representa el 1% de todos los cnceres y 5% de todos los ndulos tiroideos. El estudio clnico diagnstico y teraputico debe iniciarse con historia clnica completa, exploracin fsica y exmenes de laboratorio que incluyan un perfil tiroideo para evaluar funcin de la glndula. La citologa tiroidea por aspiracin (CTA) es el principal procedimiento diagnstico en los pacientes con ndulo tiroideo solitario, por ser capaz de diferenciar lesiones benignas de las malignas, sus principales ventajas son: segura, reduce costos de atencin mdica, selecciona mejor los pacientes que sern sometidos a tratamiento quirrgico y se realiza en pacientes ambulatorios. Torres AP, Hernndez SE, Caracas PN, Serrano GI et al Diagnstico y tratamiento del ndulo tiroideo. Rev Edocrinol Nutr 2000; 8 (3): 87-93.

27.- Masculino de 47 aos con datos positivos de alcoholismo crnico, al cual se le diagnstica pancreatitis aguda Cul es la primera medida teraputica a adoptar? a) Iniciar antibiticos de amplio espectro. b) Administracin de inhibidores de la bomba de protones. c) Ayuno absoluto d) Aspiracin nasogstrica.

El 80 % de los pacientes con pancreatitis aguda se tratan mediante medidas de sostn, suspensin de la va oral, hidratacin intravenosa, analgsicos y alimentacin parenteral cuando la va oral se restringe ms all de la semana.

La indicacin de la suspensin de la va oral se basa en el dolor y la intolerancia digestiva. No debe ser prolongada y se debe restablecer secuencialmente luego de 48 horas sin dolor. La alimentacin parenteral no tiene ningn sentido si el restablecimiento de la va oral se realiza dentro de los primeros 7 das. De no ser as se sabe que esta patologa grave provoca una agresin severa que determina un estado hpercatablico por lo tanto debe implementarse soporte nutricional para evitar la desnutricin y las complicaciones que ella trae consigo (alteracin de la modulacin de la respuesta inflamatoria, translocacin bacteriana, inmuno supresin, etc.) El 60 % de esta enfermedad presenta hipermetabolismo (1,5 veces el metabolismo basal), el 40 % son normo o hipometablicos. Segn la Conferencia de Consenso de Nutricin de la Agresin (Francia 1998), los triglicridos no son contraindicados a menos que el paciente presente una hipertrigliceridemia importante. La necesidad de aporte debe ser de 0,25 a 0,30 g/Kg. La suplementacin de micronutrientes fundamentalmente antioxidantes (vitaminas A, C, E y selenio) y zinc estn indicadas. La suplementacin en base a glutamina, inmunomoduladores, nuevas emulsiones lipdicas en base a aceite de oliva necesitan aun ser confirmadas para la pancreatitis severa. La va enteral se debe privilegiar ya que no solo es mas fisiolgica sino que tambin presenta menor costo y complicaciones habiendo actualmente estudios que muestran un menor ndice de morbimortalidad en estos pacientes ya que la integridad de la barrera intestinal limitara la sobreinfeccin bacteriana pero fundamentalmente la fngica ( Kalfarentzos 97, Windsor 98, Pupelis 2000)

Referencias Banks PA, Freeman ML; Practice Parameters Committee of the American College of Gastroenterology. Practice guidelines in acute pancreatitis. Am J Gastroenterol. 2006 Oct;101(10):2379-400. Frossard JL, Steer ML, Pastor CM. Acute pancreatitis. Lancet. 2008;371:143-152.

28.- El diagnstico para un hombre de 44 aos que ha presentado un cuadro de diarrea con una duracin de ms de dos semanas, pero que no se extiende por ms all de cuatro, es: a) b) c) d) Diarrea aguda Diarrea persistente Diarrea crnica Diarrea acuosa

La clasificacin de la diarrea de acuerdo al tiempo de duracin de este sntoma la divide en: aguda, con duracin de menos de 14 das, crnica con duracin de ms de 4 semanas, y persistente ha la que tiene una duracin menor del mes, y mayor a las dos semanas.

Kasper DL, Braunwald E, Fauci AS, Hauser SL, Longo DL, Jameson JL. Harrisons Principles of Internal Medicine. McGraw Hill. 16 Ed. 225 p.

29.- Masculino de 2 aos y medio aos de edad, diagnosticado con sfilis congnita tarda, una de las manifestaciones de esta infeccin es: a) Hepatomegalia b) Rinorrea c) Queratitis e) Pseudoparlisis de Parrot

La sfilis congnita tarda se manifiesta despus de los 2 aos de vida, el sntoma ms frecuente es la queratitis, que es parte de la trada de Hutchinson (queratitis, hipocausia y dientes en tonel o Hutchinson), as como deformidades seas (tibias en sable, frente olmpica, engrosamiento clavicular), rodilla de clutton, paresia juvenil y tabes dorsal. La hepatomegalia, rinorrea, roseola sifiltica y la pseudoparlisis de parrot son manifestaciones de la sfilis precoz que se manifiesta en los primeros dos aos de vida. Evans HE, Frenkel LD: Congenital Syphilis. Clin Honeyman: Manual de Dermatologa, 2da ed. 1988; pg. 46. Perinatol 1994; 21(1)

Lukehart SA, Holmes KK: Sfilis. en: Principios de Medicina Interna. 12 ed. Ed. Harrison. Cap. 128, pg. 765.

30.- A 55-year-old female is brought to the emergency room with a sudden onset of severe chest, back, and abdominal pain that began 20 minutes ago. The pain is described as being very sharp with a tearing-like character. Her previous medical history is notable for hypertension treated with hydrochlorothiazide. On examination, her blood pressure is 145/95 mm Hg in the right arm and 119/75 mm Hg in the left arm; pulse, 105 beats/minute; temperature, 37.3C.; and respirations, 17 breaths/minute. A chest X-ray (CXR) demonstrates a widened mediastinum, and electrocardiography (ECG) reveals nonspecific ST and T wave changes. What is the most appropriate next step in the management of this patient?

a) b) c) d)

Emergent surgery Thoracic computed tomography Aortography Intravenous labetalol

DISECCIN DE AORTA: Los predictores clnicos de mayor sensibilidad para el diagnstico son: dolor torcico sbito e intenso, ausencia o diferencia de pulsos y ensanchamiento mediastinal. El manejo mdico est dirigido a disminuir la presin arterial media y de manera

muy importante, a disminuir el aumento de la presin pulstil que golpea directamente la aorta (dp/dt). Las medidas a tomar para este objetivo incluyen reposo absoluto, control de las

cifras de presin arterial, administracin de oxgeno y monitorizacin completa no invasiva. Los objetivos iniciales de reduccin de las cifras de tensin sistlica deben

alcanzar los 100-120 mm Hg o menos, manteniendo la perfusin a los rganos vitales. Los medicamentos antihipertensivos de eleccin son los beta bloqueadores IV,

combinados con nitroprusiato de sodio. Una terapia alternativa a esta combinacin, es el uso de labetalol que tiene efecto bloqueador alfa y beta. Khan IA, Chandra N. Clinical, Diagnostic, and management perspectives of aortic

dissection. Chest 2002; 122: 311-328.

31.- Masculino de 42 aos de edad con diagnstico de herpes genital activo y recurrente, presenta de manera sbita numerosas mculas eritematosas, ppulas, vesculas. Las manos, pies y cara estn extensamente afectadas. A la exploracin fsica minuciosa demuestra que muchas de las lesiones tienen una aparencia de tiro al blanco. Algunas afectan labios y mucosa oral. El diagnstico ms probable es: a) b) c) d) Eritema multiforme Granuloma anular Pnfigo Necrolisis txico epidrmica

El eritema multiforme es una enfermedad de la piel y las mucosas que se manifiesta con lesiones eritematosas y de tipo vesculo-ampollar. Las lesiones Vesculo-ampollares y erosivas a nivel de la cavidad oral y la piel pueden ser causadas por un amplio grupo de patologas. La etiologa de las mismas tambin, puede ser muy variable, desde una causa traumtica o qumica por contacto, hasta una causa autoinmune.

Las lesiones producidas por el eritema multiforme son circulares y pueden presentarse como anillos concntricos (se las conoce como lesiones de tiro al blanco). Estas lesiones tambin pueden asociarse con otras condiciones mdicas tales como infeccin por herpes, infeccin estreptoccica, tuberculosis (TB), o como una reaccin a sustancias qumicas o medicamentos.

Gavald-Esteve C, Murillo-Corts J, Poveda-Roda R. Eritema multiforme. Revisin y puesta el da RCOE, 2004, Vol 9, N4, 415-423

32.- Femenino de 43 aos portadora de DM tipo II, e HTAS, es ingresada al servicio de Medicina Interna por cetoacidosis. Posterior a su recuperacin metablica inicia con fiebre, cefalea, dolor facial, disminucin del nivel de conciencia y enrojecimiento nasal con lesin negruzca en fosa nasal derecha. El diagnstico ms probable en sta paciente es:

a) b) c) d)

Endocarditis por S. aureus. Carcinoma epidermoide. Infeccin por M. tuberculosis. Infeccin por Mucor.

Mucormicosis es el nombre comn dado a varias diferentes enfermedades causadas por hongos de la orden de los Mucorales. Muchas diferentes especies han sido implicadas como agentes de sndromes clnicos similares.

Los Mucoraceos son hongos que se encuentran en todas partes y son comunes habitantes de materia en descomposicin. Por ejemplo, Rhizopus sp. Frecuentemente puede ser recuperado de pan mohoso. Por su rpido crecimiento y prolfica capacidad de formar esporas, inhalacin de conidias debe ser una experiencia cotidiana. La presencia de esporas Mucorales en cinta adhesiva no estril se demostr fue la fuente de mucormicosis cutnea primaria. Aun cuando estos hongos crecen en muchos nichos ecolgicos, la infrecuencia de enfermedad debida a estos organismos da fe de su baja potencial virulencia en el husped humano. En contraste a la amplia distribucin de este hongo, la enfermedad en humanos esta limitada, en muchos casos a poblacin con severo inmunocompromiso, diabetes mellitus o trauma.

Manifestaciones clnicas En 1973 Meyer y Armstrong12 categorizaron las diferentes presentaciones clnicas de mucormicosis, considerando el rgano involucrado y haciendo una divisin en seis entidades: rinocerebral, cutnea, gastrointestinal, pulmonar, diseminada y formas miscelneas. Existe una fuerte asociacin entre la entidad subyacente y la forma de presentacin. La mucormicosis rinocerebral se presenta ms frecuentemente en pacientes diabticos con acidosis, y debido al advenimiento de quimioterapias cada vez ms potentes, se aprecia con ms frecuencia en pacientes leucmicos con neutropenia prolongada, en aquellos que reciben mltiples esquemas de antibiticos y corticoesteroides, as como en pacientes con trasplantes de rganos. En pocas ocasiones se ha documentado esta forma invasora en personas sin enfermedades subyacentes. La forma pulmonar puede presentarse en pacientes con leucemia y neutropenia; la mucormicosis gastrointestinal se observa ms comnmente en pacientes con desnutricin calrico-proteica y en prematuros, y la diseminada en pacientes con dficit inmunolgico grave como trasplantados, leucmicos y nefrpatas tratados con deferoxamina.1,2 La infeccin rinocerebral es la presentacin ms frecuente y caracterstica de mucormicosis, siendo la rinoorbitaria y la mucormicosis paranasal estadios tempranos de esta. La infeccin generalmente inicia en senos paranasales o paladar duro, y se extende a senos adyacentes con diseminacin a travs de senos etmoidales y zona retroorbitaria; puede tener acceso al cerebro a travs del pex orbitario, lmina cribosa y por va vascular. Una costra necrtica sangrante en paladar o en mucosa nasal y un drenaje ocular de pus negruzco orientan al diagnstico. Puede haber una progresin rpida y presentarse el deceso en pocos das o ser indolente si la enfermedad subyacente se logra controlar. Inicialmente puede haber dolor facial, cefalea, fiebre y algn grado de celulitis orbitaria, conforme progresa la invasin de la rbita la prdida de la funcin del II, III, IV y VI nervios craneales puede ocurrir, as como tambin puede haber prdida de la funcin de msculos extraoculares, proptosis, quemosis progresiva, congestin nasal, epistaxis y

letargia. La disfuncin de nervios craneales, especialmente el V y VII, ocurre de manera tarda manifestando ptosis y midriasis, lo cual es un factor pronstico grave, en caso de trombosis retiniana, hay prdida de la visin y puede haber afeccin intraocular. El compromiso cerebral es la complicacin ms seria y puede manifestarse como: infarto, absceso, trombosis del seno cavernoso, hematoma subdural y necrosis del lbulo frontal.2 Otras complicaciones son trombosis de arteria cartida interna y vena yugular, e incluso se ha reportado infarto de miocardio por oclusin de coronaria y aborto sptico. Puede haber presentaciones crnicas y secuelas tardas a pesar del tratamiento aparentemente satisfactorio por lo cual siempre debe realizarse un seguimiento del paciente a largo plazo

BIBLIOGRAFIA:

Mayo Clinic. Pulmonary Diseases Mandele?Douglas. Infections Diseases Pennington. Respiratory infections, Diagnosis and Management Ronald B. george., Richard W. Light, Michael A. Matthay. Chest Medicine 3era. edicin. Scott E Davis. Neumona Mictica. Clnicas mdicas de Norteamrica 1997; 5: 1092?1094.

33.- Paciente masculino de ao y medio de edad, refieren los padres que desde que inici a caminar aproximadamente a los 15 meses se ha presentado con cojera. Al realizar la exploracin llama la atencin que presenta marcha en Trendelenburg. Su sospecha principal de diagnstico es: a) Artritis sptica de cadera. . b) Enfermedad neurolgica que afecta al desarrollo psicomotor. c) Luxacin congnita de cadera. d) Enfermedad de Perthes.

La luxacin congnita de cadera (L.C.C.) llamada tambin Enfermedad Luxante de la Cadera, es la malformacin ortopdica de gravedad ms frecuente del ser humano. Corresponde a una displasia articular que se produce por una perturbacin en el desarrollo de la cadera en su etapa intrauterina antes del tercer mes de vida fetal. La sub-luxacin o la luxacin se produce despus del nacimiento, en los primeros meses de vida extra-uterina y como una consecuencia de la displasia. No hay que confundirla con la verdadera luxacin intrauterina de la cadera y que corresponde a una afeccin teratolgica y que se acompaa habitualmente con otras alteraciones congnitas cardacas, renales, de la columna vertebral, de los pies, etc. Esta luxacin intra-uterina es de muy difcil tratamiento, de mal pronstico y alcanza no ms del 5% del total de las luxaciones congnitas de la cadera. La L.C.C. es progresiva. Se nace con displasia y, si no es adecuadamente tratada, progresa a subluxacin y luxacin, representando el 95% de las L.C.C. (el otro 5% son las luxaciones teratolgicas ya mencionadas). Por esta razn, el diagnstico precoz (primer mes de vida) es de extraordinaria importancia, ya que tratada en este momento se logran caderas clnica, anatmica y radiolgicamente normales. LUXACIN En este perodo la sintomatologa es ms llamativa. Cuadro Clnico Hay retardo en la iniciacin de la marcha, que puede ser claudicante, describindose el signo de Trendelemburg, marcha de pato en la luxacin bilateral. La nia es incapaz de realizar la abduccin total de la cadera o caderas. Puede observarse asimetra de pliegues. Acortamiento aparente del miembro inferior afectado. Contractura de aductores que limita la abduccin de la cadera a 20 o menos (Normal 45 o ms). Contractura del psoasiliaco. El trocnter mayor es anormalmente prominente encontrndose por encima de la lnea de Nelaton Roser (lnea que se extiende desde la espina iliaca anterosuperior hasta la tuberosidad del isquion). La rotacin interna est aumentada a 60 o ms debido al aumento de la anteversin por encima de 25. El signo de Trendelemburg se da por debilidad del glteo medio al acercarse sus inserciones pelvitrocantricas; cuando la paciente se sostiene sobre una pierna, la imposibilidad de los abductores para elevar la pelvis del lado opuesto cayendo sta, produce el signo positivo.

Signo del pistn (Dupuytren). Radiologa Para el estudio de los cuadrantes de Ombredanne, el arco de Shenton. La anteversin del cuello. Hipoplasia del ctilo y de la cabeza femoral. La resonancia magntica para el estudio de la cpsula y sus alteraciones: Deformidad en reloj de arena, adherencias. El limbo, etc.

Ciruga: II ciruga ortopdica y traumatologa Autor: Universidad Nacional Mayor de San Marcos (Lima). Facultad de Medicina. Escuela Acadmico Profesional de Medicina Humana. Departamento Acadmico de Ciruga Publicacin: Lima: UNMSM, 2000 Descripcin: 407 Serie: (Ciruga; 2) ISBN: 9972-46-102-5 Otros autores: Salaverry Garca, Oswaldo, 1959-, ed.

Tema: Traumatologa; Ortopedia

34.- Femenino de 55 aos de edad, que acude a consulta externa refiere rubor, tumefaccin y rigidez de las articulaciones interfalngicas dstales desde hace tres meses, sin presencia de otras molestias articulares. Cul de los siguientes diagnsticos es ms probable? a) Osteoartritis erosiva b) Artritis reumatoide c) Espondilitis anquilosante d) Esclerodermia

Allen R. M. MMS Medicina Interna. 5. Edicin. National Medical Series. Mc. Graw Hill. 2006. (captulo 10 V E 1). Tpicamente, la osteoartritis erosiva afecta las articulaciones interfalngicas dstales en mujeres de edad madura. Es improbable que esos sntomas articulares dstales prominentes sucedan en pacientes con artritis reumatoide o con lupus eritematoso diseminado sin molestias articulares ms generalizadas. No hay pruebas que indiquen espondilitis anquilosante o esclerodermia.

35.- Masculino de 50 aos con dolor precordial relacionado al esfuerzo, de corta duracin, de 4 meses de evolucin y con ECG en reposo normal, el siguiente estudio de eleccin es:

a) b) c) d)

Ecocardiograma de reposo Prueba de Talio- Dipiridamol Ecocardiograma con estrs farmacolgico Prueba de esfuerzo con protocolo de Bruce

El propsito fundamental de la prueba es el de demostrar la existencia de isquemia miocrdica en los subgrupos de poblacin con mayor prevalencia de cardiopata isqumica o bien en los subgrupos en donde la prueba se efecta a manera de evaluacin del tratamiento en pacientes ya conocidos con cardiopata isqumica. Existe otro subgrupo de poblacin en quienes este estudio ayuda a determinar la clase funcional en la que se encuentran e incluye a pacientes con valvulopatas o con insuficiencia cardiaca de cualquier etiologa.

Protocolos de esfuerzo. El protocolo ms empleado es el de Bruce sobre treadmill, aunque existen otros protocolos y su eleccin depender de las condiciones del individuo. Los protocolos discontinuos son los que alternan periodos de esfuerzo que se intercalan con periodos de reposo de duracin similar, se emplean en escasas circunstancias.

Los protocolos continuos son los que no interrumpen el esfuerzo una vez iniciado hasta finalizada la prueba, permiten mejor adaptacin fsica y psicolgica y es posible adaptar la intensidad de forma individualizada para que la prueba tenga una duracin de 6 a 12 minutos. Los protocolos mximos son los que se suspenden debido a la sintomatologa del paciente, a los signos registrados durante la prueba o se alcanzan valores mximos de FC y VO2. Los protocolos submximos son los que se suspenden cuando el sujeto alcanza un nivel determinado de carga, habitualmente el 85% de la FC mxima terica (que se encuentra entorno a los 170 lpm). En la prctica diaria, el nivel de carga (VO2) se expresa en forma de trabajo externo (MET equivalentes metablicos) que corresponden a 3,5ml/kg/min de VO2, lo que permite comparar protocolos entre s (cada protocolo dispone de frmulas para realizar el clculo de los METS), el error que cometen en el clculo de los METS es mayor en protocolos discontinuos. Emplear la FC como nico criterio para determinar el esfuerzo mximo es errneo, por lo que deberan tenerse en cuenta otros criterios, como es la percepcin subjetiva por parte del paciente mediante la escala de Borg (tabla I). Esta dificultad en la prediccin del esfuerzo mximo es lo que limita la realizacin de pruebas submximas a la determinacin de la condicin fsica de sujetos aparentemente sanos. Tabla I. Escala de Percepcin del esfuerzo de Borg. Escala de 15 grados Valor Percepcin 6 7 8 9 10 11 12 13 14 15 16 17 18 19 20 No se siente nada Muy muy leve Muy leve Escala de 10 grados Valor Percepcin 0 0,5 1 2 Nada Muy muy leve Muy leve Leve Moderada Algo fuerte Fuerte o intensa Muy fuerte

Considerablemente leve 3 Moderadamente dura Dura Muy dura Muy muy dura Esfuerzo mximo 4 5 6 7 8 9 10

Muy (submxima)

muy

fuerte

* A la izquierda la escala original de esfuerzo percibido en 15 grados (de 6 a 20) y a la derecha la ms nueva de 10 categoras.

Tabla II. Indicaciones clsicas de la ergometra. I. Fines diagnsticos A. Pacientes sintomticos. 1. Dolor torcico: a) Tpico b) Atpico 2. Clnica de equivalentes isqumicos. A. Pacientes asintomticos. 1. Con alteraciones en el ECG sugestivas de isquemia. 2. Con alta probabilidad de padecer Cardiopata Isqumica (paciente con mltiples factores de riesgo) 3. Cuando convenga descartar con cierta seguridad CI. 4. Con sospecha de CI silente. 5. Sedentarios que inician programa de actividad fsica. 6. Para estudio funcional de ciertas arritmias. II. Con fines valorativos y pronsticos. 1. Seguimiento de paciente con CI conocida. 2. Tras IAM. 3. En exmenes prelaborales o laborales. 4. De la eficacia del tratamiento: Mdico. Cateterismo y angioplastia. Quirrgico. 5. Respuesta de la Tensin Arterial. 6. En valvulopatas o miocardiopatas. 7. Estudio de arritmias y trastornos de la conduccin aurculo-ventricular. 8. En cardiopatas congnitas. Bibliografa 1. Gibbons RJ (Edit.). ACC/AHA 2002 Guideline Update of Exercise Testing. 2002 American College of Cardiology Foundation and American Heart Association ACC/AHA; 2002 [Acceso 1-4-06]. Disponible en: 2. Guidelines for cardiac exercise testing. ESC Working Group on Exercise Physiology, Physiopathology and Electrocardiography Eur Heart J 1993; 14: 969-988. 3. Fernando Ars Aros F, Boraita A, Alegria E, Alonso AM, Bardaji A, Lamiel R el al. Guas de prctica clnica de la Sociedad Espaola de Cardiologa en pruebas de esfuerzo. Rev Esp Cardiol 2000; 53 (8): 1063-94 4. Chaitman B. Las pruebas de esfuerzo. En: Braunwald E, editor. Tratado de Cardiologa. Medicina Cardiovascular. 4 ed. Madrid Mc-Graw-Hill-Interamericana de Espaa; 1993. p. 177-197.

5.

6.

7. 8.

9. 10. 11. 12.

13.

Schlant RC, Friesinger GC 2nd, Leonard JJ. Clinical competence in exercise testing: a statement for physicians from the ACP/ACC/AHA Task Force on Clinical Privileges in Cardiology. J Am Coll Cardiol 1990; 16: 1061-5 Reyes Lopez de los M, Iiguez Romo A, Goicolea de Oro A, Funes Lopez B, Castro Beiras A. El consentimiento informado en cardiologa. Rev Esp Cardiol 1998; 51: 782796. Fletcher GF, Flipse T, Malouf J, Kligfield P. Current status of ECG stress testing. Curr Probl Cardiol. 1998 Jul; 23(7): 353-423. Alegra Ezquerra E, Alijarde Guimer M, Cordo Mollar JC, Chorro Gasc FJ, Pajarn Lpez A. Utilidad de la prueba de esfuerzo y de otros mtodos basados en el electrocardiograma en la cardiopata isqumica crnica. Rev Esp Cardiol 1997; 50: 6-14 Wasserman K, Hansen JE, Sue DY, Whipp BJ, Casaburi R. Principles of exercise testing and interpretation . 2 ed. Philadelphia: Lea & Febiger; 1994. p. 95-111. American college of Sports Medicine. Guideliness for exercise testing and prescription. 5 ed. Baltimore: Williams & Wilkins; 1995. Borg GA. Psychophysical bases of perceived exertion. Med Sci Sports Exerc 1982; 14: 377-381. Froelicher VF, Umann TM. Exercise testing: clinical applications. En: Pollock ML, Schmidt DH, editors. Heart disease and rehabilitation . 3 ed. Champaign, IL: Human Kinetics, 1995; p.57-79. Myers J, Froelicher VF. Exercise testing. Procedures and implementation. Cardiol Clin. 1993; 11(2): 199-213.

Weiner DA, McCabe C, Hueter DC, Ryan TJ, Hood WB Jr. The predictive value of anginal chest pain as an indicator of coronary disease during exercise testing. Am Heart J 1978; 96: 458-462.

36.- Masculino de 30 aos de edad adicto a las drogas intravenosas presenta debilidad del hemicuerpo derecho y cefalea en un periodo de dos das. La exploracin revela un individuo mal nutrido y afebril con hemiparesia derecha leve. Cul de los siguientes es el diagnstico ms probable?

a) b) c) d)

Absceso cerebral Endocarditis bacteriana Meningitis por virus de la inmunodeficiencia humana (VIH) Meningitis criptoccica.

Los adictos a drogas intravenosas estn propensos a sufrir bacteriemia, que a su vez puede producir absceso cerebral y disfuncin neurolgica progresiva. Los pacientes con aqul, por lo comn se encuentran afebriles a no ser que haya endocarditis acompaante u otro origen endovascular de infeccin. Los adictos a drogas intravenosas tienden a presentar endocarditis bacteriana y pueden cursar con dficit neurolgicos en forma apopltica debido a embolia sptica del cerebro. Sin embargo, por lo regular tienen fiebre. La meningitis por VIH produce cefalea y datos de irritacin menngea, pero no se presenta dficit neurolgico focal. La meningitis criptoccica se manifiesta con conducta alterada y cefalea y los pacientes estn afebriles. Sin embargo, es raro que haya datos de seudoapopleja. Por ltimo, el uso de drogas intravenosas puede provocar embolia de cuerpo extrao pero con problemas neurolgicos apoplticos. Un mbolo puede llegar al cerebro por un cortocircuito cardaco de derecha a izquierda o bien por una malformacin arteriovenosa pulmonar si la inyeccin es venosa. El mbolo puede entrar a la circulacin cerebral de manera directa en caso de inyeccin intracarotdea.

Referencias: Nath A. Brain abscess and parameningeal infections. In: Goldman L, Ausiello D, eds. Cecil

Medicine. 23rd ed. Philadelphia, Pa: Saunders Elsevier; 2007: chap 438

Allen R. M. MMS Medicina Interna. 5. Edicin. National Medical Series. Mc. Graw Hill. 2006. (Captulo 11 XVI B 1-2).

37.- Paciente de 52 aos de edad con diagnstico de neumona adquirida en la comunidad, quien recibi tratamiento antibitico a base de Ceftriaxona. No tuvo respuesta adecuada, y en el estudio diagnstico para determinar la causa, se encontr un derrame pleural del 60%, el cual se puncion. En el estudio citoqumico y bacteriolgico de este lquido, se reportan cocos grampositivos en la tincin de Gram, y un pH de 7. Con esto, usted considera como indispensable:

a) b) c) d)

Iniciar cobertura con vancomicina. Colocacin de sonda endopleural. Intubacin orotraqueal y ventilacin con volmenes altos. Ingreso a Unidad de Terapia Intensiva.

Los hallazgos del citoqumico y tincin son caractersticos de empiema. El drenaje del mismo es la maniobra teraputica base para el tratamiento y resolucin del mismo. Kasper DL, Braunwald E, Fauci AS, Hauser SL, Longo DL, Jameson JL. Harrisons Principles of Internal Medicine. McGraw Hill. 16 Ed. 1536 p.

38.- Un estudiante universitario de 20 aos de edad acude a consulta debido a tos seca, fiebre, cefalea y dolor muscular durante las ltimas 2 semanas. Refiere que sus compaeros de casa han desarrollado sntomas similares. Niega el uso de drogas ilcitas y no es homosexual. Su temperatura es de 38.2C, FC 90lpm, FR 18x. Se auscultan murmullo vesicular bilateral. Una RX de trax muestra opacidades intersticiales multifocales. Presenta leucocitosis y un test de aglutinina fro +. Cual de los siguientes es el patgeno ms probable?

a) b) c) d)

Bacterias anaerbicas Mycoplasma pneumoniae Pneumocystis carinii Streptococcus pneumoniae

En el caso antes mencionado, se trata de una neumona llamada atpica que se presenta por norma en pacientes jvenes y que han estado en lugares de convivencia con otras personas, la imagen radiogrfica de esta neumona es de una neuropata intersticial como se indica. 1.- Fishman AP, , Fishman JA, Grippi MA, Kaisser LR, Seor RM. Pulmonary Diseases and disorder. 3a. Edicin McGraw-Hill, EUA, 2006. 2.- Fraser, R ; Neil, C; Par, P; Diseases of the Chest, Third Edition, Editorial Elsevier, 2005. 3.- Murray and Nadels; Textbook Respiratory Medicine, Vol 1-2, Elsevier editorial, 2005.

39.- Masculino de 5 aos de edad, con antecedente de neumona por Pneumocystis, la profilaxis de sta patologa en ste paciente, se encuentra indicada cuando las cifras de linfocitos ajustados son de: a) b) c) d) < 1500 Clulas/l < 750 Clulas/l < 500 Clulas/l < 200 Clulas/l

La NPC en nios y en el embarazo La NPC es infeccin diagnstica de SIDA en un alto porcentaje de nios, sobre todo en el primer ao de vida. Los nios menores de 1 ao con CD4+ por debajo de 1500//l t ienen un 90% de riesgo de padecerla. Las manifestaciones clnicas, el diagnstico y el tratamiento no difieren de los del adulto. Para la prevencin deben seguirse las siguientes recomendaciones:

Se aconseja la profilaxis primaria en todos los nios menores de 1 ao hijos de madre VIH+ La profilaxis debera comenzar a las 4-6 semanas de vida y suspenderse en caso de que posteriormente no resulten estar infectados. Los nios infectados y aquellos cuyo estado de infeccin por VIH se desconozca debern seguir recibiendo tratamiento profilctico durante el primer ao de vida. En los mayores de esta edad se considerar continuar con la profilaxis atendiendo al recuento de linfocitos CD4+ segn la edad: de 1 a 5 aos, cuando el recuento sea < 500 clulas/l o el por cent aj e < 15%; en nios de 6 a 12 aos, cuando el r ecuent o sea inferior a 200 clulas/ l o el por cent aj e < 15%. No se ha estudiado la seguridad de interrumpir la profilaxis en nios infectados con el VIH y sometidos a tratamiento antirretroviral. Los nios con un historial que incluya episodios de NPC debern ser tratados de por vida con quimioprofilaxis para evitar casos de recurrencia.

NCP en el embarazo Como ya se ha dicho, la quimioprofilaxis anti-NPC debe administrarse a las mujeres embarazadas al igual que al resto de adultos y adolescentes. En este caso, el agente profilctico recomendado sigue siendo el cotrimoxazol, con dapsona como alternativa. Debido a la posibilidad terica de una posible teratogenicidad asociada a la exposicin a los frmacos durante el primer trimestre de gestacin, durante dicho perodo puede considerarse la alternativa de pentamidina en aerosol debido a que dicho agente no se absorbe sistmicamente por lo que el feto en desarrollo no sufre exposicin al frmaco. Lectura recomendada: Neumona por Pneumocystis carinii inmunodeficiencia humana (VIH). Gac Med Mex 2004; 140 (1): 59-70 en nios infectados por el virus de

40.- Masculino de 28 aos acude a su consultorio con reporte de exmenes de laboratorio con los siguientes resultados. Hemolisinas bifsicas + (o anticuerpos de DonathLandsteiner) productoras de hemoglobinuria paroxstica a frigore. La Entidad responsable de los resultados de este paciente es:

a) Sfilis. b) Leucemia linftica crnica. c) Mieloma mltiple. d) Lupus eritematoso diseminado.

Razones por las que se realiza el examen Este examen se realiza algunas veces cuando el mdico sospecha de un diagnstico de criohemoglobinuria paroxstica. Valores normales La ausencia de anticuerpos es lo normal. Significado de los resultados anormales Los resultados anormales indican la presencia de criohemoglobinuria paroxstica (PCH), un trastorno que ocurre cuando la exposicin a bajas temperaturas hace que el sistema inmunitario produzca anticuerpos que destruyen los glbulos rojos. Estos anticuerpos se denominan anticuerpos de Donath-Landsteiner. A medida que las clulas son destruidas, la parte de los glbulos rojos (hemoglobina) que transporta el oxgeno es eliminada en la orina. La criohemoglobinuria paroxstica es un sndrome adquirido. Algunas veces, los anticuerpos se presentan con una infeccin viral (por ejemplo, sarampin y paperas) o con sfilis; sin embargo, en algunos casos, el trastorno no est relacionado con una enfermedad y la causa se desconoce. K. Holmes, P. Mardh, P. Sparling et al (eds). Sexually Transmitted Diseases, 3rd Edition. New York: McGraw-Hill, 1999, chapters 33-37.

41.- Femenino de 28 aos G5 P2 A3 tiene una historia de abuso de sustancias prenatal. A las 37 sdg tiene una parto vaginal obtenindose una neonato pequeo para la edad gestacional, de gnero masculino con pqueas aperturas palpebrales, pliegues epicnticos, con cara aplanada, filtrum hipoplsico y borde del vermillon delgado. Estos hallazgos son caractersticos en neonatos cuyas madres tuvieron abuso prenatal de cual de las siguientes sustancias:

a) b) c) d)

Tabaquismo Marihuana Alcohol Narcticos

EFECTOS DEL ALCOHOL SOBRE EL RECIEN NACIDO 1 - Efectos somticos. En 1967 Lemoine y colaboradores describieron las anomalas observadas en hijos de madres alcohlicas; posteriormente, en 1973, Jones, Smith y colaboradores las denominaron sndrome alcohlico fetal (SAF). Las principales caractersticas observadas en los nios con

SAF son las siguientes: En el 50 a 80% hubo retraso del crecimiento intrauterino, microcefalia, apertura palpebral estrecha, nariz corta y respingona, mandbula hipoplsica y labio superior fino. Tambin se ha asociado a cardiopatas congnitas, anomalas en extremidades y en la columna vertebral.

BIBLIOGRAFIA: Bell GL, Lau K. Problemas perinatales y neonatales por abuso de sustancias. Clin Pediatr Nort America 1995, 2:247-266. Byrd RS, Howard CR. Childrens passive and prenatal exposure to cigarrette smoke. Pediatr Annals 1995: 24(12): 644-645 Cruz M, Bosch J. Sndromes Pediatricos. Barcelona, Espaxs Publicaciones Mdicas1998, 534-535

Eyler FD, Behnke M. Desarrollo temprano en lactantes con exposicin a drogas. Clin Perinatol 1999; 1: 105-149.

42.- Femenino de 28 aos de edad, acude a consulta prenatal de rutina G5 P4, con 28 SDG, refiere que no ha sentido movimiento fetal durante los ltimos 2 das. Su embarazo ha sido complicado debido a que padece hipertensin crnica, para lo cual le indicaron tabletas de alfa-metildopa 2 veces al da. Al examen, su FU es de 30cm, y las maniobras de Leopold demuestran que el feto se encuentra en situacin transversa. Su TA es 145/85mmHg. No se encuentra latido cardiaco con el Doppler. Cul de los siguientes es el paso ms apropiado a seguir en el manejo?

a) b) c) d)

Realizar un test sin estrs Amniocentesis USG Beta-HCG (cuantitativa)

Probable bito: Sintomatologa y diagnstico Signos funcionales: No se perciben movimientos fetales por 12-24 horas. Disminucin o ausencia de sntomas y/o signos como nauseas vmito, hipertensin, albuminuria) Paraclnicos: *ecografa: diagnstico precoz y exacto: Doppler. *Radiologa: hay 3 signos: +deformacin del crneo +curvatura y torsin de la columna +presencia de gas en el feto *lquido amnitico: puede estar meconiado, o sanguinolento Signos locales: en los senos hay secrecin calostral, sangrado leve y oscuro por vagina, el feto se vuelve blando a la palpacin, fetocardia (-), puede haber detencin y/o disminucin de la altura uterina, bajo peso corporal, entre otros.

Bibliografa: 1. OBSTETRICIA, Schwarcz R, editorial El ateneo, 2003. 2. Sociedad espaola de ginecologa y obstetricia, junio 2002. 3. OBSTETRICIA CLNICA, Llaca V, edicin 2000, captulo 24; Pg, 315-316.

43.- Masculino de 4 aos llevado a consultar por presentar exantema mculopapular de inicio en cara, posteriormente en tronco, extremidades y nalgas, con adenopata cervical y retroauricular, refiere la madre que ha presentado fiebre no cuantificada y catarro El diagnstico ms probable de este paciente es? : a) Varicela. b) Rubola c) Sarampin. d) Exantema sbito

La tambin llamada Tercera enfermedad, era conocida antiguamente como Sarampin alemn y considerada como una variante del Sarampin o la Escarlatina. Tambin se la conoce como Sarampin rojo. En 1914 se teoriz sobre su origen viral y recin en 1938 lo confirmaron Hiro y Tosaka. En 1940, durante una epidemia en Australia, un oftalmlogo report 78 casos de cataratas congnitas en bebs nacidos de madres infectadas en el primer trimestre del embarazo: fue el primer dato reconocido del Sindrome de Rubeola Congnita (SRC).

Agente etiolgico: Es un rubivirus (RNA) de la familia togavirus, aislado en 1962 por Parkman y Weller. Se clasifica de dos maneras: 1) Post-natal:

Es una enfermedad infectocontagiosa, comn en la infancia y juventud. Da sntomas generales leves: exantema mculopapular difuso, estado febril y adenopatas suboccipitales, postauriculares y cervicales posteriores (que son caractersticas de esta patologa, antecedindola entre 5 a 10 das). El 25 a 50% cursa en forma subclnica. En nios mayores y adultos (mujeres fundamentalmente), puede dar poliartralgias. Aunque en general no deja secuelas, puede complicarse con encefalitis o trombocitopenia.

Esta forma se transmite por contacto directo o por gotitas de Pflgge. Predomina a fines del invierno y comienzos de primavera. En comunidades pequeas o cerradas, durante un brote suelen padecerla todos los susceptibles. El perodo de incubacin vara entre 14 y 21 das. El contagio se producira desde pocos das antes del exantema y hasta 7 das despus del inicio del mismo (aunque estudios hechos en voluntarios, mostraron presencia del virus rubeola en secreciones nasofarngeas desde 7 das antes y hasta 14 das despus).

Manifestaciones clnicas: es una enfermedad infecciosa leve que evoluciona de manera subclnica u oligosintomtica en alrededor del 50% de los nios pequeos. Los sntomas prodrmicos, como aumento de volumen doloroso (adenomegalia) en ganglios retroauriculares, cervicales posteriores y occipitales son ms frecuentes entre los adolescentes y adultos jvenes. El exantema puede ser tambin el signo inicial, con mculas y ppulas rosadas plidas que aparecen inicialmente en la cara y cuello y se generalizan en 2 a 3 das. El exantema puede ser intensamente eritematoso y descamarse finamente. Los sntomas generales, como fiebre, cefalea, mialgias, artralgias y ocasionalmente artritis, son ms frecuentes entre las mujeres adolescentes y adultos jvenes. Las articulaciones ms afectadas son las muecas y las interfalngicas.

2) Congnita:

Ms del 25% de los fetos cuya madre padeci rubeola durante el primer trimestre del embarazo, son afectados. Puede producir muerte uterina o aborto espontneo. Las patologas asociadas con esta enfermedad son: oftalmolgicas (microoftalma, cataratas, coriorretinitis), cardiolgicas (conducto arterioso persistente, estenosis perifrica de la arteria pulmonar, etc.), auditivas (sordera neurosensorial) y neurolgicas (microcefalia, meningoencefalitis, retraso mental). Adems presentan retardo del crecimiento, hepatoesplenomegalia, ictericia y lesiones similares a las purpricas. Muchas de las rubeolas maternas pueden cursar en forma asintomtica y los casos leves del feto, se diagnostican varios meses o aos despus del nacimiento (sordera parcial o manifestaciones cardacas leves). Despus de la vigsima semana de gestacin, la infeccin fetal es casi nula. Los lactantes con SRC pueden eliminar el virus en sus secreciones y orina durante un ao o ms, afectando as a otras personas susceptibles. De todas maneras, esta susceptibilidad se ha mantenido antes y despus de la vacuna, en un 10 a 20%.

Bibliografa: 1. Report of the Committee on Infectious Diseases, 25h edition, Red Book 2000. American Academy of Pediatrics. 2. Fifth (human parvovirus) and sixth (herpesvirus 6) diseases. Koch WC. Curr Opin Infect Dis 2001, Jun; 14 (3): 343-356. 3. Primary human herpesvirus 8 in immunocompetent children. Andreoni M, Sarmati L, Nicastri E, El Sawaf G, El Zalabani M, Uccella I, et al. JAMA 2002 Mar 13; 287 (10): 1295-300.

Varicella vaccine update. AAP. Pediatrics 2000, Jan 105: 136-141.

44.- Se trata de masculino de 2 aos con otitis media aguda recurrente quien recibi tratamiento con amoxicilina en el mes previo, el tratamiento de eleccin en ste paciente es: a) Amoxacilina+ clavulanato a 40 mgkgda b) Amoxacilina+ clavulanato a 90 mgkgda c) Azitromicina v.o 10mg kg da d) Ceftriaxona IM 50 mgKgda

En nios mayores de 2 aos, con OMA moderad o leve y sin factores de riesgo, el tratamiento inicial debera ser sintomtico. Si a las 48-72 horas persiste o empeora la clnica se debe iniciar antibioterapia dirigida frente al S. pneumoniae, por su elevada frecuencia en nuestro medio y la baja tasa de curacin espontnea. Se administrar amoxicilina a las dosis estndar recomendadas (40-50 mg/kg/da), ya que tienen una buena actividad frente a S. pneumoniaesensible o con resistencia intermedia a la penicilina. La duracin del tratamiento antibitico no est claramente establecida, aunque se ha observado que con 5-7 das se logra la resolucin de la infeccin. Con una pauta corta se consigue, adems, disminuir el riesgo de resistencias bacterianas, del nmero total de antibiticos pautados y del coste econmico.

En caso de fracaso teraputico, es decir, reaparicin o persistencia de signos y sntomas de OMA, se debe administrar un antibitico no slo efectivo frente al S. pneumoniaeresistente a penicilina sino tambin frente a patgenos productores de betalactamasa: amoxicilina-clavulnico a dosis altas (80-90 mg/kg/dia de amoxicilina y 10 mg/kg/dia de cido clavulnico). Hasta que no se disponga de formas comerciales adecuadas (relacin amoxicilina/clavulnico= 8/1), esta dosificacin se puede alcanzar mezclando amoxicilina y amoxicilina-clavulnico

BIBLIOGRAFA 1. Rosenfeld RM. Mtodo basado en la evidencia para tratar la otitis media. Clin Ped Nort(ed esp) 1996; 6: 1075-1092. 2. Arnold JE. El odo. En: Behrman RE, Kliegman RM, Arvin AM, ed. Nelson. Tratado de Pediatra.l5 edicin. McGraw-Hill-Interamericana de Espaa, 1997; 2258-2270. 3. Pitkaranta A, Virolainen A, Jero J, Arruda E, Hayden F. Deteccin de infecciones por rinovirus, virus respiratorio sincitial y coronavirus en la otitis media aguda mediante la reaccin en cadena de la polimerasa de transcriptasa inversa. Pediatrics (ed esp.)1998; 6: 85-89. 4. Palva T, Pulkkinen K. Mastoiditis. J Laryngol Otol 1991; 105: 765-766.

45.- Femenino de 40 aos de edad acude al servicio de urgencias quejndose de calambres en las piernas y parestesias en los dedos de las manos. Un ao antes se le someti a una operacin del cuello, pero no est segura qu fue lo que se le hizo. El signo de Chevostek es positivo: la percusin sobre el nervio facial por delante de la oreja desencadena una contraccin espasmdica del labio superior. Cul de los siguientes trastornos sugiere un signo positivo? a) b) c) d) Hipercalciemia Hipocalciemia Acidosis Hiperpotasemia

SINTOMATOLOGA Neuromuscular: la hipocalcemia aguda se manifiesta por parestesia (hormigueo y adormecimiento de los dedos y regin peribucal) y calambres o contracturas musculares. La sintomatologa subclnica de tetania, evidenciada por el signo de Chvostek, (ocurrencia de espasmo facial, especialmente del orbicular de los labios, al percutir el nervio facial a mitad de distancia entre la comisura labial y el odo). El signo de Trousseau es un espasmo doloroso del carpo, que se presenta luego de mantener por tres minutos una presin >20 mmHg por encima de la sistlica, siendo un signo ms de tetania. La hipocalcemia crnica se presenta con irritabilidad, confusin, demencia e incluso, en infantes, como retardo mental. Tambin se reportan movimientos coreicos, distonas y

convulsiones. Se ha reportado calcificacin de ganglios basales en la radiografa de crneo, que no es reversible al tratamiento. Cardiovascular: prolongacin de la fase de potencial de accin y por lo tanto prolongacin del segmento ST en el ECG. En casos de severa deficiencia se presentan arritmias, hipotensin o falla cardiaca; la hipocalcemia aumenta la cardiotoxicidad de los digitlicos. Pulmonar: broncoespasmos y laringoespasmos vistos, sin embargo, con poca frecuencia.

Dermatolgica: piel seca, uas quebradizas y cada del cabello como signos no especficos en la hipocalcemia crnica

LECTURAS RECOMENDADAS: 1. Carlstedt F, Lind L. Hypocalcemic syndromes. Crit Care Clin 2001; 17:139-153. 2. Gibbs M, Wolfson A, Tayal V. Electrolyte disturbances. En: Rosens Emergency Medicine. Concepts and Critical Practice. J Marx, Hockberg. R, Walls R, et al (eds). Fith edition. Mosby. St Louis,1998. 3. Kapoor M, Chan GZ. Fluid and electrolyte abnormalities. Crit Care Clin 2001; 17:503-529. 4. Lind L, Carlstedt F, Rastad J, et al. Hypocalcemia and parathyroid hormone secretion in critically ill patients. Crit Care Med 2000; 28:93-99. 5. Lo CY. Postthyroidectomy hypocalcemia. J Am Coll Surg 2003; 196:497-498. 6. Marx SJ. Hyperparathyroid and hypoparathyroid disorders.N Engl J Med 2000; 343:1863-1875.

46.- Femenino de 26 aos, acude al servicio de consulta externa refiriendo presentar desde hace varias semanas temblor fino distal, sensacin de angustia, sudoracin palmar, ha perdido peso en los ltimos meses. Sin antecedentes de importancia. Exploracin fsica: T-A 130/86 mm hg, peso 54 kg., talla 160 cm., exoftalmos, sudoracin palmar, piel hmeda y caliente, FC 110 lpm, abdomen con ruidos peristlticos incrementados, con aumento en el nmero de evacuaciones. En este paciente va a encontrar una disminucin de:

a) b) c) d)

TSH. Cortisol en orina de 24 hrs T4 total. T3 r.

El bocio txico difuso (BTD) constituye la forma ms frecuente de hiperfuncin de la glndula tiroidea (70 % de los casos), que puede aparecer a cualquier edad, aunque por lo general aparece entre la tercera y cuarta dcada de la vida. Esta enfermedad es ms frecuente en la mujer, donde se observa un predominio de 7:1 en relacin con los hombres en regiones no bocigenas. Esta relacin se reduce en las zonas de bocio endmico. Los factores genticos desempean un papel esencial en la etiologa y existe una predisposicin familiar a esta enfermedad de Graves-Basedow. El BTD se caracteriza por la presencia de hipertiroidismo, bocio difuso y elstico, oftalmopata, dermopata, acropaquia tiroidea y onicolisis. Es importante el diagnstico y tratamiento precoz del hipertiroidismo para evitar complicaciones, principalmente las cardiovasculares.

TABLA I 1. Piel: Piel fina, caliente y sudorosa. Prurito. Pelo fino y frgil. Onicolisis. 2. Sistema cardiovascular: Taquicardia, palpitaciones y fibrilacin auricular. Insuficiencia cardiaca, angor pectoris, disnea de esfuerzo, vasodilatacin. Disminucin de la respuesta a la digital. 3. Aparato digestivo: Hiperdefecacin. Disfuncin heptica: hipertransaminasemia. 4. Aparato locomotor: Debilidad y atrofia de la musculatura proximal. Osteoporosis. Aumento de la maduracin sea en nios. Hiperreflexia, temblor distal, mioclonias. 5. Sistema nervioso: Irritabilidad, nerviosismo e insomnio. Psicosis, hipercinesia. 6. Otros: Prdida de peso a pesar de la polifagia. Intolerancia al calor. Alteraciones menstruales y disminucin de la fertilidad en mujeres.

Pruebas de funcin tiroidea: a) Determinacin de hormonas tiroideas: L-tiroxina (T4) que circula en plasma unida en su mayora a la protena transportadora (TBG), y menos del 0,1% libre. Aunque slo esta pequea porcin est libre, es la concentracin de T4 libre ms que la T4 total la que indica la actividad tiroidea; por lo

tanto, en la mayora de los casos slo es necesario determinar la T4 libre, que estar elevada en los casos de hipertiroidismo. L-triyodotironina (T3): se produce por la desyodacin perifrica de T4; es regulada por factores independientes de la funcin tiroidea. En algunos pacientes hipertiroideos la concentracin de T3 est elevada cuando no lo est la de T4. TSH, producida por clulas de la adenohipfisis, que controla la funcin tiroidea por accin directa positiva y que es controlada a su vez por la TRH hipotalmica con efecto positivo estimulador. En los casos de hipertiroidismo primario la TSH estar inhibida por el sistema de retroalimentacin debido al exceso de hormonas tiroideas perifricas (T4 y T3). La TSH tiene una mayor sensibilidad para el diagnstico del hipertiroidismo que la T4 libre. Sin embargo, no es del todo especfica, es decir, una TSH baja no siempre indica hipertiroidismo. TRH sintetizada a nivel hipotalmico. No se suele determinar en la prctica clnica. Existen algunos casos en los que la concentracin de T3 yT4 se encuentran en el lmite superior de la normalidad y persiste la sospecha de hipertiroidismo. En estos casos la administracin de TRH no produce ningn in cremento en los niveles de TSH en los hipertiroidismos primarios (test de TRH para TSH). As pues, para el diagnstico de un hipertiroidismo utilizaremos bsicamente los niveles de TSH y T4 libre.

Hipertiroidismo: clnica, diagnstico y tratamiento I. M. RECHE MOLINA, B. VALERA, C. HIDALGO, L. LEN, G. PIDROLA

Servicios de Medicina Interna y Endocrinologa. Hospital Universitario Virgen de las Nieves. Granada

47.- Masculino de 42 aos alcohlico con cirrosis y ascitis. Es hospitalizado por agitacin y comportamiento extrao. Cul de los siguientes hallazgos es el ms til para hacer el diagnstico de encefalopata heptica?

a) b) c) d)

Ictericia Asterixis de las manos Hemangiomas superficiales Signo de la ola positivo

ENCEFALOPATA Alteracin en el estado mental, en el comportamiento y en el sueo que progresa a la desorientacin y al coma. Indica una insuficiencia heptica severa. Fisiopatologia Las toxinas son inactivadas por el hgado pero entran a la circulacin portal por los cortos circuitos. Toxinas no son inactivadas y excretadas. Las toxinas se encuentran elevadas en el lquido cerebroespinal. TOXINAS. Amonio. Neuroexcitatorio y depresor del sistema nervioso central. Aminocidos aromticos. Son precursores de neurotransmisores. Si incrementan en sangre la encefalopata no progresa. Acido gammaaminobutrico (GABA). Incrementado significativamente en la encefalopata. Benzodiacepinas endgenas. Compiten por los receptores de GABA y barbitricos.

CUADRO CLINICO
El diagnstico de la encefalopata heptica depende de la existencia de una enfermedad heptica, desde la insuficiencia heptica aguda y la toxicidad por frmacos, hasta las enfermedades crnicas, tales como la hepatropata alcohlica o la cirrosis. En la historia clnica se hace nfasis especial en los antecedentes de enfermedades del hgado diagnosticadas con anterioridad, una historia de alcoholismo o de hepatitis. Transfusiones de sangre previas o abuso de drogas por va intravenosa pueden sugerir una hepatitis crnica B o C. La utilizacin de frmacos hepatotxicos, como la metildopa, la nitrofurantoina o la isonicida, puede ser causa de una hepatopata crnica, mientras que una dosis alta de paracetamol puede ser causante de una necrosis hepatocelular fulminante. El cuadro clnico incluye tres elementos: cambios en el estado mental, hedor heptico y asterixis. El hedor heptico se refiere al aliento ftido del paciente. La asterixis es un temblor por sacudidas, irregular y bilateral de las manos, debido a una interrupcin momentnea y brusca del tono muscular de los antebrazos. El examen fsico se centra en la bsqueda de los estigmas caractersticos de las enfermedades hepticas: ictericia, nevus en araa, ginecomastia, atrofia testicular, venas distendidas en la pared abdominal (cabeza de medusa) y ascitis. La exploracin neurolgica durante los estados precoces suele mostrar apraxia de construccin y dificultad para escribir. Otros hallazgos fsicos son la rigidez de las extremidades e hiperreflexia. En el estado de coma profundo suele haber prdida del tono muscular y disminucin de los reflejos tendinosos profundos.

RESUMEN: Petequias. Rinofima. Contractura de Dupuytren (aponeurosis palmar). Telangiectasias en el tronco. Ascitis. Asterixis. Eritema palmar. Atrofia testicular.

Blibliografa:
Shakelfords. Surgery of the alimentary tract. 5a. Ed. 2002. Tomo 3. Feldmans. Gastroeneterology. 2002. Perez. Anatoma y fisiologa del hgado. Univ. Catlica de Chile. 2005. Bratiz. Serum laboratory test in cirrhosis. Journal of Hepatology. Slovakia. 2005. Paradis. Glycomics. Journal of hepatology. Ireland. Agosto 2005.

48.- Masculino de 36 aos de edad acude a consulta por presentar nausea y vmito, aumento en el nmero de evacuaciones de consistencia aguada, (diarrea sin productos patolgicos), afebril, T.a 110.70 mmhg, refiere que su hijo present los misma sintomatologa casi al mismo tiempo que el, como antecedente refiere haber comido juntos en un restaurante hace aproximadamente 4 hrs De los siguientes microorganismos cual es el de mayor probabilidad de ocasionar ste cuadro?

a) Salmonella enteriditis. b) Shigella sonnei. c) Staphylococcus aureus. d) E. Coli

La contaminacin de alimentos por S. aureus, est asociada con una forma de gastroenteritis que se manifiesta clnicamente por un cuadro caracterizado por vmitos (76% de casos) y diarrea (77% de casos). El corto perodo de incubacin de 1-6 horas orienta a la sospecha de enfermedad producida por ingestin de una o ms enterotoxinas preformadas en el alimento que ha sido contaminado con cepas de S. aureus productor de la misma. Son raramente observados signos de toxicidad sistmica, tales como fiebre e hipotensin En general, es un cuadro autolimitado que tpicamente se resuelve en 2448 horas desde el inicio.

No est claro si se desarrolla en humanos inmunidad a largo plazo, pero anticuerpos frente a una SE no necesariamente confieren inmunidad frente a la intoxicacin por S. aureus, ya que existe mltiples SE capaces de producir enfermedad. En algunos casos, anticuerpos producidos frente a una SE confieren proteccin cruzada contra otra SE, ya que algunas comparten eptopes. Todas las SE son capaces experimentalmente en primates de producir emesis, y no se registra enterotoxemia, excepto en dosis muy altas, probablemente debido a su dificultad para atravesar mucosas. El 99% de casos de intoxicacin alimentaria por enterotoxinas Estafilocccicas est asociado a S. aureus y ocasionalmente se reportan casos por Las cepas estafilocccicas enterotoxignicas aisladas de alimentos implicados en brotes de infeccin son mas a menudo lisadas por fagos del grupo lll, y menos frecuentemente simultneamente por los grupos l y lll.

Staphylococcus epidermidis.

Tratamiento. Como para la mayora de enfermedades trasmitidas por alimentos autolimitadas, las medidas de sostn son la base del tratamiento. No est indicado tratamiento con antimicrobianos. Referencias bibliogrficas. - Manual of Clinical Microbiology. Murray, P. 1995 6th edition. - Principles and Practice of Infectious Diseases. Mandell, Douglas, Bennett. 1995. 4th edition. - Dinges M, Orwin P, Schlievert P. 2000. Exotoxins of Staphylococcus aureus. Clinical Microbiology Reviews, vol 13; 16-34

49.- Masculino de 55 aos que acude consulta por presentar dolor ocular y fotofobia intensa. En la exploracin ocular se observa ojo rojo principalmente alrededor del limbo corneal, pupila en miosis y depsitos blanquecinos en endotelio. Se realiza la toma de presin ocular la cual es de 10 mm Hg (normal >16). El diagnstico probable corresponde es?

a) b) c) d)

Queratitis. Conjuntivitis infecciosa. Uvetis. Glaucoma agudo.

La uvetis se define como la inflamacin de la vea, lmina intermedia del ojo que se encuentra entre la esclertica y la retina, la cual aporta la mayor parte del suministro sanguneo a la retina. La uvetis es una de las causas del ojo rojo.1 Suele acompaarse de disminucin de la agudeza visual, de curso lento y progresivo y dolor.

No produce secrecciones externas lo que la distingue de otras causas de ojo rojo, como la blefaritis, conjuntivitis y el chalazin. Por extensin se llama uvetis a cualquier tipo de inflamacin del interior del ojo. Suele cursar con cierta intolerancia a la luz y requiere de un exhaustivo oftalmolgico. Puede afectar a uno o ambos ojos.

Tipos:
Se suele clasificar en cuatro tipos: anterior, intermedia, posterior y panuvetica'. Anterior. Es la ms frecuente, entre el 70 y 90 por ciento de las uvetis. Es una inflamacin del iris del ojo, crnea y cuerpo ciliar,2 provocada a veces por una enfermedad autoinmune como la artritis reumatoide o la espondilitis anquilosante, aunque en la mayora de los casos su causa es desconocida.1 Cursa con ojo rojo, conjuntiva irritada, dolor y prdida de visin parcial. Intermedia o parsplanitis. Es la inflamacin de la pars plana, rea estrecha ubicada entre el iris y la coroides. Generalmente es un proceso leve que afecta a los hombres jvenes y no se asocia a ninguna otra enfermedad. Es posible que haya una asociacin con la enfermedad de Crohn y con la esclerosis mltiple. A veces se complican produciendo hemorragias o depsitos de material inflamado en la pars plana.1 Posterior. Es la inflamacin de la coroides o (coroiditis). Si se afecta tambin la retina se llama coriorretinitis. Puede producir prdida de visin de intensidad variable, dependiendo del tamao y la localizacin de la cicatrizacin. Si se afecta la parte central de la retina, denominada mcula, la visin central se deteriora. Panuvetis: se ve afectada toda la vea, es decir, los segmentos anteriores y posteriores del interior del ojo.

Olitsky SE, Hug D, Smith LP. Disorders of the Uveal Tract. In: Kliegman RM, Behrman RE, Jenson HB, Stanton BF, eds. Nelson Textbook of Pediatrics. 18th ed. Philadelphia, Pa: Saunders Elsevier; 2007: chap.628.

50.- Which one of the following conditions results in prologation of the partial thromboplastin time (PTT), but not the prothrombin time (PT)? a) b) c) d) Varicela hemorrhage as a result of cirrosis Menorrhagia resulting from von Willebrands disease Therapy with broad-spectrum antibiotics Therapy with coumarin for phlebitis

Enf. Von Willebrand

Manifestaciones

Epistaxis Hemorragia transvaginal Equimosis Gingivorragias Hematomas Hemartrosis

60 % 50 % 40 % 35 % 5% 3%

EvW Pruebas de escrutinio T. Hemorragia C. Plaquetaria TTPa TP Gpo AB0 Prolongado Normales, excepto 2B Normal o prolongado Normal 25 % bajo en 0

1.-Lee GR, Foerster J, Lukens J, Paraskevas F, Greer JP and Rodgers GM. Wintrobes clinical haematology; 10th Edition, Lippincott Williams & Wilkins, United States of America, 1999. 2.-Williams WJ. Manual Williams de hematologa 5a Edicin McGraw-Hill Interamericana, Mxico, 1997. 3.-Beutler E, Lichtman MA, Coller BS, Kipps T. Hematology. 5th International Edition. United Stated of America, 1995.

4.-Ruiz Argelles GJ. Fundamentos de hematologa; 2. Edicin, Editorial Mdica Panamericana, Mxico, 1998.

51.- Femenino de 19 aos que acude a consulta externa por presentar un exantema cutneo rojo difuso; fiebre de 39.4C y diarrea leve acuosa. Como antecedentes refiere infeccin de garganta por la que se le administr sulfametoxazol. Comenz sus menstruaciones hace tres das. En la exploracin fsica se encuentran cambios eritematosos difusos de la piel con descamacin temprana. La boca y las conjuntivas estn eritematosas. Cul de los siguientes explica todo el proceso? a) Bacteriemia por Salmonella b) Sndrome de choque txico (TSS) c) Tuberculosis d) Mononucleosis por virus de Epstein-Barr Allen R. M. MMS Medicina Interna. 5. Edicin. National Medical Series. Mc. Graw Hill. 2006. (Captulo 8 VII D 1 a, 2 b). La bacteriemia por Salmonella, el sndrome de choque txico (TSS), la tuberculosis y la mononucleosis de Epstein-Barr pueden acompaarse de fiebre, pero la presencia de exantema descamativo difuso sugiere TSS, reaccin farmacolgica grave (p. ej., sndrome de Stevens-John-son); enfermedad de Kawasaki, o escarlatina. El exantema cutneo relacionado con salmonelosis es muy sutil y evanescente (manchas de color rosa). La tuberculosis no se caracteriza por afeccin cutnea difusa y de mucosas o diarrea acuosa. Si bien la alergia al sulfametoxazol puede producir eritema cutneo y de mucosas, no causa diarrea.

52.- En la infeccin por virus de la inmunodeficiencia humana por lo comn la linfadenopata difusa en una persona clnicamente sana suele ser un signo de: a) b) c) d) Linfoma Sarcoma de Kaposi Tuberculosis No indica infeccin o tumor especficos

Allen R. M. MMS Medicina Interna. 5. Edicin. National Medical Series. Mc. Graw Hill. 2006. (captulo 8 VIII G 1 b). La linfadenopata difusa en una persona infectada por virus de la inmunodeficiencia humana (VIH) que se encuentra clnicamente bien suele ser signo de que no hay infeccin especfica o tumor. Aunque todas esas respuestas pueden ser ciertas, las personas con mltiples ganglios aumentados de tamao, tuberculosis o trastornos malignos tienden a encontrarse enfermas. Con mayor frecuencia tambin experimentan prdida de peso y fiebre. Es ms probable que el linfoma se presente con afeccin orgnica en pacientes infectados con VIH que en otros. El sarcoma de Kaposi puede producir afeccin linftica, pero en general slo se encuentra en etapa tarda de la enfermedad con lesiones cutneas y mucosas extensas. La linfadenopata moderada es un dato comn en infeccin por VIH en etapa media. Se desconoce su causa exacta, pero la desaparicin de la linfadenopata prolongada puede preceder al deterioro clnico. La sfilis produce adenopata local o difusa en pacientes con infeccin por VIH o sin ella. Sin embargo, esta adenopata siempre se acompaa de algn otro dato de sfilis.

53.- Acude a consulta un adolescente de 20 aos la cual refiere que desde hace varios meses presenta astenia, cansancio, prdida de apetito y dificultades para concentrarse en los estudios. Al interrogatorio refiere que a perdido inters en los estudios, frecuenta menos a sus amigos, con pesimismo en la mayora de sus actividades. Cul es el diagnstico ms probable?

a) Anorexia nerviosa b) Trastorno de ansiedad. c) Depresin mayor d) Distimia.

LA DISTIMIA La distimia es un estado de nimo crnicamente deprimido, menos grave que la depresin y que no cumple los criterios para una depresin mayor, o lo hace slo en perodos muy cortos. Su evolucin suele ser de ms de dos aos. Se caracteriza por un abatimiento prolongado del estado de nimo en que el sujeto distmico se describe a s mismo como triste o desanimado, perdiendo el inters por las cosas y vindose a menudo como intil y poco interesante. Posee sntomas persistentes o intermitentes, de intensidad ms leve comparacin a la depresin mayor. Aiskal (1983) la define como mal humor y se caracteriza porque el individuo est habitualmente triste, introvertido, melanclico, excesivamente consciente, incapaz de alegra y preocupado por su insuficiencia personal.

Los criterios de diagnstico de Distimia son los que a continuacin se detallan: A. Estado de nimo crnicamente depresivo la mayor parte del da de la mayora de los das, manifestado por el sujeto u observado por los dems, durante al menos 2 aos. Nota: En los nios y adolescentes el estado de nimo puede ser irritable y la duracin debe ser de al menos 1 ao. B. Presencia, mientras est deprimido, de dos (o ms) de los siguientes sntomas: 1. 2. 3. 4. 5. 6. Prdida o aumento de apetito Insomnio o hipersomnia Falta de energa o fatiga Baja autoestima Dificultades para concentrarse o para tomar decisiones Sentimientos de desesperanza

C. Durante el perodo de 2 aos (1 ao en nios y adolescentes) de la alteracin, el sujeto no ha estado sin sntomas de los Criterios A y B durante ms de 2 meses seguidos. D. No ha habido ningn episodio depresivo mayor durante los primeros 2 aos de la alteracin (1 ao para nios y adolescentes); por ejemplo, la alteracin no se explica mejor por la presencia de un trastorno depresivo mayor crnico o un trastorno depresivo mayor, en remisin parcial. Nota: Antes de la aparicin del episodio distmico pudo haber un episodio depresivo mayor previo que ha remitido totalmente (ningn signo o sntoma significativos durante 2 meses). Adems, tras los primeros 2 aos (1 ao en nios y adolescentes) de trastorno distmico, puede haber episodios de trastorno depresivo mayor superpuestos, en cuyo caso cabe realizar ambos diagnsticos si se cumplen los criterios para un episodio depresivo mayor. E. Nunca ha habido un episodio manaco, un episodio mixto o un episodio hipomanaco y nunca se han cumplido los criterios para el trastorno ciclotmco. F. La alteracin no aparece exclusivamente en el transcurso de un trastorno psictico crnico, como son la esquizofrenia o el trastorno delirante. G. Los sntomas no son debidos a los efectos fisiolgicos directos de una sustancia (p. ej., una droga, un medicamento) o a enfermedad mdica (p. ej., hipotiroidismo). H. Los sntomas causan un malestar clnicamente significativo o deterioro social, laboral o de otras reas importantes de la actividad del individuo. Como vemos, la Distimia presenta sntomas ms o menos similares a la Depresin mayor, pero se diferencian entre s respecto a:

Tipo de evolucin: el estado de nimo depresivo es crnico (no presenta intervalos libres de sntomas o mejoras significativas) y dura por lo menos 2 aos. Severidad de los sntomas: los mismos suelen ser leves o moderados, sin una alteracin significativa de las relaciones familiares, sociales y laborales del individuo.

Referencias Bibliogrficas
- Diagnostic and Statistical Manual of Mental Disorders, 4 edition (DSM-IV TR). American Psychiatric Association. American Psychiatric Press, 2000. - Kaplan and Sadock's Synopsis of Psychiatry, 9 edition. Lippincott Williams & Wilkins Press, 2003.

54.- Acuden a consulta los padres de un menor, que cuenta con cinco aos de edad, lleva varias noches despertndose agitado como si hubiera soado algo que le angustia. Cuando acuden a su lado por la noche, el nio les mira y dice palabras que no tienen ningn significado. Al cabo de un rato vuelve a dormirse y por la maana no recuerda nada de lo ocurrido. El diagnstico ms probable que presenta ste paciente es: a) b) c) d) Pesadillas Disomnia. Terrores nocturnos. Sonambulismo.

Criterios para el diagnstico de F51.5 Pesadillas (307.47) A. Despertares repetidos durante el perodo de sueo mayor o en las siestas diurnas, provocados por sueos extremadamente terrorficos y prolongados que dejan recuerdos vividos, y cuyo contenido suele centrarse en amenazas para la propia supervivencia, seguridad o autoestima. Los despertares suelen ocurrir durante la segunda mitad del perodo de sueo. B. Al despertarse del sueo terrorfico, la persona recupera rpidamente el estado orientado y despierto (a diferencia de la confusin y desorientacin que caracterizan los terrores nocturnos y algunas formas de epilepsia). C. Las pesadillas, o la alteracin del sueo determinada por los continuos despertares, provocan malestar clnicamente significativo o deterioro social, laboral o de otras reas importantes de la actividad del individuo. D. Las pesadillas no aparecen exclusivamente en el transcurso de otro trastorno mental (p. ej., delirium, trastorno por estrs postraumtico) y no se deben a los efectos fisiolgicos directos de una sustancia (p. ej., drogas, frmacos) o de una enfermedad mdica.

Criterios para el diagnstico de F51.4 Terrores nocturnos (307.46) A. Episodios recurrentes de despertares bruscos, que se producen generalmente durante el primer tercio del episodio de sueo mayor y que se inician con un grito de angustia. B. Aparicin durante el episodio de miedo y signos de activacin vegetativa de carcter intenso, por ejemplo, taquicardia, taquipnea y sudoracin. C. El individuo muestra una falta relativa de respuesta a los esfuerzos de los dems por tranquilizarle. D. Existe amnesia del episodio: el individuo no puede describir recuerdo alguno detallado de lo acontecido durante la noche. E. Estos episodios provocan malestar clnicamente significativo o deterioro social, laboral, o de otras reas importantes de la actividad del individuo. F. La alteracin no se debe a los efectos fisiolgicos directos de una sustancia (p. ej., drogas, frmacos) o de una enfermedad mdica.

55.- En la unidad de Medicina Familiar recibe a un paciente de 64 aos de edad que asegura ser hipertenso de ms de cinco aos de evolucin, sin agudizaciones adems de ser portador de una fibrilacin auricular, motivo por los cuales recibe propranolol en dosis de 100 MG cada maana. Al revisar su electrocardiograma usted espera encontrar:

a) Ausencia de onda P, complejo QRS normal e intervalos R-R diferentes. b) Ausencia de onda P, complejo QRS ensanchado, diferencia constante de los intervalos RR. c) Presencia de onda P, complejo QRS ensanchado y eje rotado a la derecha. d) Ritmo nodal con disociacin A-V

Guadalajara J. Cardiologa. Sexta Edicin 151 152 La fibrilacin auricular es la arritmia cardiaca ms frecuente. La despolarizacin catica y desordenada de las aurculas por mltiples ondas en simultnea, trae como consecuencia que se pierda la funcin mecnica de la contraccin auricular. Estos dipolos de activacin mltiple y desordenados alcanzan al nodo A-V y penetran en l, algunos pasan hacia el Haz de His mientras que otros no lo alcanzan, debido a la penetracin parcial del nodo por conduccin decreciente completa, esto se conoce como, conduccin oculta; as, la rpida penetracin de los estmulos auriculares favorece la aparicin de conduccin oculta, la cual afecta en forma impredecible el periodo refractario del nodo. El trazo electrocardiogrfico tpico es la ausencia de onda P, complejo QRS normal e intervalos R-R diferentes.

56.- Acatempa, Gro. Es una comunidad de 100,000 personas. Durante 2001 hubo 1,000 defunciones por todas las causas. Durante el mismo ao se registraron un total de 300 casos de Infartos Agudos al Miocardio y 60 defunciones por esta misma causa. La tasa de mortalidad especfica para Infarto Agudo al Miocardio en Acatempa Gro. es de : a) b) c) d) 20% 20 por 100,000 60 por 100, 000 10 por 100,000

La tasa de mortalidad para cualquier enfermedad especifica, puede expresarse para toda la poblacin o para cualquier subgrupo de edad, raza o sexo. Se calcular dividiendo el nmero de muertes causadas por la enfermedad especifica entre la poblacin total y se expresan como muertes por 100, 000 habitantes por ao. ( TMe= 60/ 100, 000 por 100, 000)

Morton R. F. Bioestadstica y Epidemiologa, Interamericana, 3. Ed. 1993; pg: 22.

57.- Se trata de un recin nacido al que se decide colocar una sonda orogstrica, al darnos cuenta de que dicha sonda no avanza y apreciar en una radiografa toracoabdomial la ausencia de aire en el intestino, Sospechamos de?: a) Atresia de esfago tipo II y V b) Atresia de esfago tipo III y IV c) Hernia diafragmtica congnita d) Atresia de esfago tipo I y II.

. Consiste en la interrupcin de la luz esofgica con o sin comunicacin con la va area. No existe an una definicin precisa del trmino "long gap". De acuerdo a nuestra experiencia se trata de aquellas AE, independientemente del tipo, caracterizadas por una distancia entre cabos esofgicos lo suficientemente grande como para imposibilitar la realizacin de una anastomosis primaria trmino-terminal.
De acuerdo a nuestro criterio, basado en la clasificacin de Ladd, las atresias de esfago pueden clasificarse en los tipos I, II, III, IV y V 1). La Tipo I es la atresia de esfago aislada sin fstula traqueoesofgica o atresia pura. No tiene comunicacin con la va area. Representa aproximadamente el 5% de los casos observados y es la segunda en frecuencia. Puede diagnosticarse con ms facilidad en el embarazo, que cursa tpicamente con polihidramnios y ausencia de imagen gstrica en las ecografas prenatales. Al nacer, los neonatos presentan el abdomen excavado por falta de pasaje de aire al intestino. En todos los casos ambos cabos esofgicos se encuentran muy separados entre s.

La Tipo II es una forma muy rara de atresia de esfago que representa del 1 al 3% de los casos. Existe una fstula traqueoesofgica desde el cabo superior del esfago a la trquea cervical. Al igual que en las Tipo I, no hay pasaje de aire al intestino distal y ambos extremos se encuentran muy alejados entre s. La Tipo III es la forma observada en el 90% de los pacientes. Comnmente se la conoce como sinnimo de atresia de esfago. Se caracteriza por una bolsa esofgica comunicacin entre el extremo distal esofgico y la trquea (fstula trqueoesofagica al cabo inferior), que puede nacer a cualquier altura de la trquea y excepcionalmente, tambin en los bronquios. En estos casos el embarazo no cursa tpicamente con polihidramnios y suele verse una imagen gstrica normal en las ecografas prenatales. Al nacer los neonatos suelen presentar el abdomen distendido por el gran pasaje de aire que existe desde la va area hacia el estmago e intestino. La distancia entre ambos cabos es por lo general inferior a dos o tres vrtebras. La coexistencia de atresia de esfago Tipo III con atresia duodenal, intestinal o malformaciones anorrectales constituye un cuadro de urgencia; el gran flujo de aire desviado hacia el sistema digestivo a travs de la fstula traqueoesofgica inferior provoca distensin abdominal severa y eventualmente, incluso, perforacin intestinal. La Tipo IV se caracteriza por una fstula trqueoesofagica en ambos cabos esofgicos. Su observacin es poco frecuente. Representa menos del 2% de los casos. La fstula superior suele encontrarse por accidente cuando se efecta la correccin quirrgica inicial. En ocasiones pasa inadvertida por el cirujano y su diagnstico es efectuado en forma tarda. La Tipo V no es en realidad una atresia de esfago. El esfago se encuentra permeable y es de buen calibre. En estos pacientes lo que se observa es una fstula traqueoesofgicaaislada, generalmente de ubicacin cervical o torcica alta. El diagnstico se realiza excepcionalmente en el periodo neonatal ya que los sntomas suelen aparecer en la infancia.

Bibliografa: Urgencias en Pediatra, Interamericana.McGraw Hill. Captulo: Urgencias Mdico Quirrgicas, Seccin XXIII, pg. 774-778. Operative Pediatric Surgery. Moritz M. Ziegler. International Edition, pg. 349- 354. Ciruga Peditrica, Ashcraft - Holder Interamericana.McGraw Hill pg. 257 a 277.

58.- Masculino recin nacido que a la exploracin fsica presenta peso 3.900kg, talla 55cm, ausencia de lanugo y vrmix gaseosa, la sospecha diagnstica en ste paciente es: a) Prematuro b) Trmino c) Postmaduro d) Hipotrfico Definicin: Se define como recin nacido postrmino a cualquier nio nacido ms all de 42 semanas de gestacin, cualquiera que sea su peso. El recin nacido postmaduro es el producto de una gestacin postrmino o gestacin prolongada que presenta signos de postmadurez.

Signos ms comunes: Peso normal o disminuido y talla aumentada. Piel seca y descamada, puede ser apergaminada. Uas largas y quebradizas. Manos tumefactas con descamacin precoz e intensa. Abundante cabello. Lanugo y vrmix caseoso ausentes. Depsitos de grasos disminuidos. Coloracin verde, marrn o amarilla de la piel y uas por impregnacin de meconio. Actitud vivas y facies despierta.

BIBLIOGRAFA: Garca-Monroy L., Embarazo Prolongado en: Nuez-Maciel E. et al. Tratado de Ginecologa y Obstetricia, Volumen II Cap.50. Mxico, Ed. Cullar, S.A de C.V; 2001. pp. 687-694. Matute GMM. Inicio del trabajo de parto. En: Nuez-Maciel E. et al. Tratado de Ginecologa y Obstetricia, Volumen I Cap.10. Mxico, Ed. Cullar, S.A de C.V; 2001. pp. 187194. Clinicas Obsttricas y Ginecologicas Vol 1:2002. McGraw-Interamericana. lvarez-Ponce VA., Lugo-Snchez AM., lvarez-Snchez AZ., Muiz-Rizo ME. Comportamiento del embarazo prolongado en el servicio de cuidados perinatales. Rev Cubana Obstet Ginecol 2004.

59.- Femenino de 34 aos la cual labora en lavandera, hace varios aos presenta enrojecimiento de los pliegues proximales de las uas de varios dedos de las manos, que ocasionalmente le supuran. Cul es el diagnstico ms probable? a) Dermatitis de contacto. b) Liquen plano. c) Dermatoficia. d) Paroniquia candidisica crnica.

Infeccin por Candida del lecho ungueal que se presenta con mayor frecuencia como una onicolisis asociada con paroniquia, aunque tambin se observa la destruccin completa del lecho ungueal y la erosin de la zona distal y lateral de las uas de los dedos, sin distrofia ungueal total. La perionixis candidisica se trata de la inflamacin con eritema, edema, dolor y salida ocasional de pus blanquecino escaso y espeso del reborde ungueal de uno o varios dedos de la mano. Con frecuencia, se acompaa de onicopata (discoloracin, onicolisis, distrofia total, etc.).

Predisponentes: Todas las labores manuales que generen humedad. (muy frecuente en amas de casas, trabajadores de restaurantes, lavanderas, etc.)

BIBLIOGRAFA: 1. Crespo Erchiga V. Protocolo diagnstico de contaminantes. En "Micologa para dermatlogos" Ed. Janssen, Madrid, 1994, pp:49-70. 2. Crespo V, De Lus B, Delgado V, Crespo A y Vera. Espectro etiolgico de las onicomicosis en nuestro medio. CO7. II Congreso Nacional de Micologa. Santiago de Compostela. Junio, 1994. 3. Crespo Erchiga V, Delgado Florencio V y Martnez Garca S. Micologa dermatolgica. Ed. M.R.A. Barcelona, 2006. 4. Daniel III CR. The Diagnosis of Nail Fungal Infection. Editorial Arch Dermatol 1991;127:1566-1567. 5. Delgado Florencio V. Protocolo de identificacin de dermatofitos. En "Micologa para dermatlogos". Ed. Janssen, Madrid, 1994, pp:27-41. 6. Delgado Florencio V. Estrategia en el diagnstico y tratamiento de las micosis superficiales. Ed. Aula Mdica, Madrid, 1994. 7. Delgado V, Abad Romero-Balmas J, Armijo Moreno M y Dulanto F. Scopulariopsis brevicaulis como agente de onicomicosis. Actas Deermo-Sif. 1976; 9-10:693-700. 8. English MP. Nails and fungi. Br J Dermatol 1976; 94:697-701. 9. Fevilhade de Chauvin M. Onicomicosis. Dermatologa prctica. 1994; 9:1-2.

60.- El siguiente diagnstico tiene como caracterstica la aparicin de cilindros hemticos en el sedimento urinario: a) Dao tubular. b) Cualquier lesin de la nefrona. c) Lesin a cualquier nivel de las vas urinarias. d) Dao glomerular severo.

La presencia de cilindros hemticos o sus derivados indica hemorragia dentro de la neurona que puede ser el resultado de una lesin glomerular como la que se ve en la nefritis hemorrgica aguda, o una enfermedad manifestada por necrosis vascular o del penacho vascular como ocurre en la Periarteritis nodosa prpura de Henoch-Schonlein, endocarditis bacteriana subaguda. Tambin la necrosis tubular renal acompaada de inflamacin intersticial puede producir hematuria y formacin de cilindros hemticos. Asimismo, la nefrosis hemoglobinrica resultante de la administracin de sangre incompatible y la hemlisis intravascular intensa pueden producir hallazgos similares

Bauer. J. D.; Toro. G... v Ackermann. P. G. (eds.): Brav's Clinical Laboratorv Methods. ed. 6, 'St. ouisC,. V: M O S ~C~o. Bibliografa nefrolgica: FARRERAS-ROZMAN. Medicina Interna; Harcourt Brace Espaa S.A. GUARDIA J, GRAU JM, NET A. Medicina Interna Fundamental; Springer Verlag Iberica S.A. ARTHUR GREENBERG. Tratado de enfermedades renales. Harcourt Brace.

61.- Posterior a un trabajo de parto con expulsin normal, y tras una hora aproximada en periodo de alumbramiento en el que se practic masaje uterino y se increment moderadamente la dosis de oxitocina, no aprecian signos de desprendimiento placentario, se indica una extraccin manual de placenta, que resulta imposible por no existir plano de separacin entre la placenta y la pared uterina. Cul es el diagnstico ms probable?

a) Engatillamiento placentario. b) Placenta succenturiata con cotiledn aberrante. c) Placenta circunvalata d) Placenta adherente por acretismo placentario.

Se denomina a la placenta como acreta cuando sta se implanta en zonas donde la decidua es deficiente o anormal y por tanto hay una infiltracin del miometrio por vellosidades coriales; esta infiltracin puede ser focal, parcial o total. A su vez esta condicin se subdivide en acreta, increta y percreta. La placenta increta y percreta infiltran todo el espesor de la pared miometrial, en la percreta adems las vellosidades, perforan la serosa y llegan en algunas ocasiones a infiltrar rganos vecinos, especialmente la vejiga. La PA est limitada a la superficie miometrial. La frecuencia de presentacin del AP vara entre 10 y 48 por 10.000 partos. (Oishi A 1999, Hung TH 1999, Zaki ZM, 1998).

En las mujeres con acretismo placentario se han visto factores de riesgo, dentro de los cuales se encuentran: 1. 2. 3. Edad y multiparidad: La presentacin AP, aumenta con la paridad de la paciente y la edad, siendo muy rara en primparas. Placenta previa: esta se ha encontrado en el 30% de los casos de PA. Igualmente se ha visto PA en el 9.3% de las pacientes con placenta previa (Miller DA, 1997). Cesrea anterior, o cirugas uterinas previas: Se ha visto este antecedente en el 25% de los casos. En el 29% de los casos la PA estaba implantada en la cicatriz uterina y solo en el 5% la placenta estaba implantada en otro sitio. (Miller Da, 1997). Dilatacin y legrado, en el 25% de los casos. Infeccin uterina previa, remocin manual de la placenta, leiomiomas y otras anomalas uterinas: La asociacin con estas entidades es inconstante. Niveles anormalmente elevados de feto-protena y de b-HCG, en el segundo trimestrre. (Hung TH, 1999).

4. 5. 6.

Una placenta adherente o penetrante no es fcil de diagnosticar antes del alumbramiento. Despus de ste, se manifiesta como retencin placentaria y sangrado uterino. El diagnstico generalmente se realiza, despus de intentar la extraccin manual de la placenta. Las manifestaciones clnicas propias de la placenta adherente, de la placenta acreta y de la placenta increta, consisten en una manifiesta dificultad o imposibilidad para la expulsin o extraccin de la placenta. Como consecuencia de la atona parcial y de la hemostasis insuficiente en las zonas de despegamiento placentario, se producir una hemorragia ms o menos grave que en nada se diferenciar de la hemorragia de la atena uterina. Y no ser solamente al intentar el alumbramiento artificial que se pondran de manifiesto las razones ntimas de la retencin placentaria; alumbramiento que ser engorroso en. la placenta ahderente e imposible en las variedades acreta e increta. En varias ocasiones puede no existir hemorragia y en estos casos la nica manifestacin de este estado morboso ser la prolongacin del perodo del alumbramiento. La placenta adherente, como toda placenta retenida, es pronto presa de un proceso infeccioso sin embargo se han sealado casos de placentas retenidas aspticamente durante muchos mese3. al cabo de los cuales han sido expulsadas sin causar trastorno alguno; pero hay que hacer observar que en estas enfermas se ha tratado de retensin de mebranas por abortes ovulares.

1.

Arredondo-Soberon F, Sabella V, Garza-Leal J, Valente PT. Placenta increta en primer trimestre de embarazo. Ginecol Obstet Mex 1995; 63: 279-81. 2. Cantanzarite V, Stanco L, Schrimmer S et al. Managing placenta previa/accreta. Contemp Obstet Gynecol 1996; 41: 66-95.

3. Ecker JL, Sorem KA, Soodak L, et al. Placenta Increta Complicating a FirsTrimester Abortion A case report. Journal Reproductive Medicine. 1992; 37-10. 4. Finberg G, William J. Placenta accreta: prospective sonographic diagnosis in patients with placenta previa / accreta. Contemp Obstet Gynecol 1996: 41: 66-95. 5. Gist RS, Voung V, Brody S, Rees P, Landry AD. Placenta increta occurring in a bligter ovum. South Med J. 1996; 89(5): 545-7. 6. Harden, MA, Walters MD, Valente PT Postabortal hemorahage due to placenta increta: A case report. Obstet Gynecol. 1990; 75: 523. 7. Hudon L, Belfort MA, Broome DR. Dosis and management of placenta percreta: A review. Obster Gynecol survey 1998; 53: 509-517. 8. Hung TH, Shau WY, Hsieh CC, et al. Risk factors for placenta accreta. Obstec Gynecol 1999; 93: 545-50. 9. Kinoshita T, Ogawa K, Yusumizu T, Kato J. Spontaneous rupture of the uterus due to placenta percreta at 25-weeks gestation: a case report J Obster Gynaecol Res 1996; 22: 125-8. Kirkinen P, Helin-Martikainen HL, Vanninen R, Patanen K. Placenta accreta: imaging by gray-scaleand contrast enhanced color Doppler somography and magnetic resonance imaging. J Clin Ultrasound 1998; 26: 90-4.

62.- Femenino de 19 aos que acude a consulta refiriendo presentar 6 meses sin regla. Se manifiesta preocupada por su sobrepeso, ha estado a dieta y ha perdido 6 kg en 8 meses. Actualmente pesa 46 Kg. con talla de 1,65 mts. Signos vitales dentro de sus parmetros normales. Se realiza prueba de embarazo con resultado negativa. Cul es la causa ms probable de su amenorrea?

a) b) c) d)

Disgenesia gonadal. Sndrome de ovario poliqustico. Adenoma hipofisario. Hipogonadismo hipogonadotropo.

El hipogonadismo femenino se caracteriza por un fallo gonadal debido a la alteracin del propio ovario o secundario a un fallo hipotlamo-hipofisario. Este fallo de la gnada puede ocurrir en distintos momentos de la vida y por causas diversas, lo que va a condicionar una presentacin clnica diferente. El ovario posee dos funciones relevantes: producir gametos femeninos (oognesis) y secretar hormonas esenciales en la regulacin de la funcin reproductora y que influyen en la diferenciacin y el desarrollo de los rganos sexuales (hormonognesis). Los mecanismos celulares y moleculares responsables de la produccin de oocitos y hormonas por la gnada femenina son parcialmente independientes entre s. Sin embargo, ambas funciones son llevadas a cabo de modo concertado gracias a la accin de un complejo

sistema de control que implica interacciones mltiples entre el hipotlamo, la hipfisis y el ovario. Para la adquisicin de la capacidad reproductora a partir de la pubertad se requiere una adecuada formacin e integracin funcional durante las etapas tempranas del desarrollo. Los elementos que componen este eje reproductor son: 1) el sistema neuronal hipotalmico responsable de la produccin del neuropptido GnRH; 2) las clulas gonadotropas de la hipfisis anterior que secretan LH (hormona luteinizante) y FSH (hormona folculo estimulante), y 3) el ovario. Hay que destacar que la formacin de la gnada y de los elementos hipotlamohipofisarios del eje gonadotropo se llevan a cabo de modo independiente en etapas tempranas del desarrollo embrionario, producindose posteriormente su integracin funcional en circuitos de retroalimentacin positivos y negativos, esenciales para una correcta funcin reproductora.

FALLO GONADAL PRIMARIO O HIPOGONADISMO HIPERGONADOTRFICO Incluye a aquellos pacientes en los que la produccin y accin de los esteroides estn reducidas. En los primeros 4 aos de vida y a partir de los 9-10 aos, la hipfisis, al faltarle la retroalimentacin negativa de estos esteroides, aumenta la produccin de gonadotropinas. Sin embargo, la elevacin de las gonadotropinas no puede normalizar la funcin gonadal. Las causas pueden ser congnitas y adquiridas.

Etiologa comn a ambos sexos

Alteraciones congnitas del receptor de las gonadotropinas

Recientemente se han clonado y mapeado los genes de los receptores de as gonadotropinas (LH-R y FSH-R) (20), estando ambos en 2p21, y se han identificado diversas mutaciones y deleciones que conducen a una disminucin de la funcin de estos receptores, independiente de las gonadotropinas (hipogonadismo hipergonadotropo); en otros casos se produce un aumento de la funcin de estos receptores.

Mutaciones que ocasionan prdida de la funcin del receptor de la LH

Hasta la fecha se han encontrado seis sustituciones de una base del gen del LH-R que conducen a dos mutaciones nonsense o sin sentido que ocasiona un codn de parada, y a cuatro mutaciones missense o mutaciones con error de sentido en los que cambia un aminocido en la protena. La mayora son homocigotos y se considera que el trastorno es A-R. Clnicamente, existe una forma grave y una menos grave, lo que tiene relacin con la actividad del receptor. En las mujeres hay amenorrea, falta de respuesta del ovario a la LH y respuesta normal a la FSH con desarrollo folicular normal.

Mutaciones que ocasionan prdida de la funcin del receptor de la FSH

Son ms raras. Clnicamente las mujeres homocigotas presentan disgenesia ovrica e hipogonadismo hipergonadotropo, mientras que las mujeres heterocigotas son fenotpicamente normales.

BIBLIOGRAFA: 1. Caete R, Jimnez L. Ontogenia y diferenciacin gonadal. Gnadas. Actualizaciones en Endocrinologa. McGraw-Hill Interamericana. Captulo 1, 2000. 2. Barrio R, Ezquieta B. Mecanismos genticos de la diferenciacin sexual: sus alteraciones. Gnadas. Actualizaciones en Endocrinologa. McGraw-Hill Interamericana. Captulo13, 2000. 3. De Roux N, Morel Y, Hardelin JP. Genetic anomalies of the gonadotropic axis. Rev Prat 1999; 49 (12): 1277-1282. 4. Tena-Sempere M, Aguilar E, Pinilla L. Fisiologa del eje hipotlamo-hipofiso-ovrico. Gnadas. Actualizaciones en Endocrinologa. McGraw-Hill Interamericana. Captulo 2, 2000. 5. Labarta JI y cols. Hipogonadismo hipergonadotrpico. En Pubertad normal y patolgica, 2. curso de postgrado. Valladolid, 1996. 6. Veldhuis JD. Neuroendocrine mechanisms mediating awakening of the human gonadotropic axis in puberty. Pediatr Nephrol 1996; 10 (3): 304-317. 7. Hopwood NJ. Pathogenesis and management of abnormal puberty. Spec Top Endocrinol Metab 1985; 7: 175-236. 8. Minagawa M, Yasuda T, Niimi H J. Spinal and femoral bone mass accumulation during normal adolescence: comparison with female patients with sexual precocity and with hypogonadism. Clin Endocrinol Metab 1996; 81 (3): 1248-1253. 9. Park KH, Lee SJ, Kim JY, Kim JY, Bai SW, Kim JW. A concomitant decrease in cortical and trabecular bone mass in isolated hypogonadotropic hypogonadism and gonadal dysgenesis. Yonsei Med J 1999; 40 (5): 444-9.

63.- En el cunero, se observa que un neonato presenta asimetra de pliegues glteos. A la EF la cadera, una de ellas puede ser fcilmente luxada posteriormente con un click y regresada a su posicin normal con un sonido parecido. La familia se encuentra preocupada porque el primer hijo tuvo el mismo problema. Cul es el diagnstico ms probable?

a) b) c) d)

Enfermedad de Legg-Perthes Artritis sptica Parto traumtico Displasia de cadera

La falta de relacin normal en las estructuras que forman una articulacin se conoce como luxacin. En el caso de la cadera, la cabeza del fmur no encaja debidamente en su cavidad (acetbulo). El desarrollo del acetbulo ser normal siempre que la cabeza del fmur permanezca en posicin correcta y el esfuerzo muscular sea adecuado. Siempre que esto no se cumpla, se producir una alteracin entre el ctilo y la cabeza femoral, dando lugar a una alteracin con el resultado de luxacin de la cadera. DIAGNSTICO: EXPLORACIN FSICA a) Maniobras de Ortolani y Barlow - click audible-. Ortolani: el click se produce cuando la fvea de la cabeza del fmur encuentra la cresta cotiloidea caminando sobre ella. La limitacin a la abduccin desaparece, esto es signo de reduccin. Barlow: se realiza la aduccin de la cadera, que tras una ligera presin longitudinal sobre el fmur, produce una luxacin. La prueba de Ortolani traduce una luxacin y la de Barlow una cadera luxable. b) Asimetra de pliegues -no valorable en luxaciones bilaterales-. c) Limitacin a la abduccin. d) Dismetra.

Garca Portabella, M. Luxacin congnita de cadera antes de los tres meses de edad. 2001. Garcia-Siso Pardo, J. M. Displasia del desarrollo de la cadera. (Parte I). Peditra Rural y Extrahospitalaria. Vol. 32. N 304 Pgs. 481-491. 2002. Graf, R. Classification of hip joint dysplasia by means of sonography. Arch Orthop Trauma Surg 102:248-255, 1984. Edeiken, J. Luxacin congnita de cadera. Diagnstico Radiolgico de las Enfermedades de los Huesos. Ed. Mdica Panamericana. Pgs. 388-392. 1977.

64.- Femenino de 2 aos, es llevada al servicio de urgencias por presentar dolor abdominal intermitente. Antecedentes: evacuaciones sanguinolentas en nmero de 6 en 24 horas de 48 hrs de evolucin. Exploracin fsica: T/A 110/65mmHg, FC 90 x, FR 28 x, Temp 36.5, irritabilidad, llanto fcil, se palpa masa dolorosa a nivel de colon ascendente. En este paciente el diagnstico ms probable es:

a) b) c) d)

Divertculo de Meckel. Invaginacin intestinal. Absceso apendicular. Apendicitis aguda

Invaginacin intestinal: Ocurre cuando una porcin del tracto alimentario se pliega dentro del segmento adyacente. Es la causa ms frecuente de obstruccin intestinal entre los 3 meses y los 6 aos de edad. El cociente varn: mujer es 4:1. Se produce de forma sbita con dolor clico paroxstico intenso recurrente a intervalos frecuentes, el 60% de los lactantes expulsan heces que contienen sangre roja y moco, las deposiciones en confitura de grosella. Nelson, Tratado de Pediatria, 17 Edicin, Ed. Elsevier, Pg.1242-1243

65.- Femenino de 27 aos se diagnostica con insuficiencia suprarrenal primaria en ste caso podemos encontrar las siguientes alteraciones de laboratorio: a) Hipercalcemia e hipernatremia b) Hipernatremia e hiperglucemia c) Hipocalcemia y leucopenia d) Hipercalemia e hiponatremia

En la insuficiencia suprarrenal primaria se observa disminucin en los niveles sricos de sodio, cloruro y bicarbonato, mientras que el potasio srico se eleva. Esta se debe a los efectos combinados de dficit de aldosterona, reduccin del filtrado glomerular y acidosis. Y sobre todo durante el estrs puede haber disminucin en los niveles de glucosa.

1.

Datos de laboratorio

Hiponatremia: los niveles sricos bajos de sodio se debe a su prdida por la orina por dficit de aldosterona y al desplazamiento del sodio hacia el compartimento intracelular. Esta prdida de sodio extravascular reduce el volumen plasmtico y acenta la hipotensin. 2. Hiperkalemia: aumento de los niveles sricos de potasio. Se debe a los efectos combinados del dficit de aldosterona, la reduccin del filtrado glomerular y la acidosis.

3. Hipocortisolemia: los niveles de cortisol y aldosterona son bajos y no aumentan con la administracin de ACTH. 4. Hipercalcemia: aumento de los niveles sricos de calcio. Ocurre en un 10-20% de los pacientes de causa desconocida. 5. Cambios electrocardiogrficos: suelen ser inespecficos, aunque con lentificacin generalizada del trazado. 6. Hemograma: puede haber anemia normoctica, linfocitosis relativa y eosinofilia moderada. 7. Prueba de estimulacin de ACTH: prueba principal que confirma el diagnstico de insuficiencia suprarrenal, al evaluar la capacidad de las suprarrenales para producir esteroides, que suelen estar ausentes o disminuidos tanto en sangre como en orina tras la estimulacin de ACTH. 8. Determinacin de la ACTH: en la insuficiencia suprarrenal primaria o Enfermedad de Addison, la ACTH y sus pptidos afines, estn elevados en plasma ante la prdida del mecanismo de retroalimentacin del eje hipotlamo-hipfisariosuprarrenal. 9. hipertermia: la hormona del hipotlamo no controla la homeostasis

Williams GH, Dluhy RG. Enfermedades de la corteza suprarrenal. En Jameson JL (ed): Harrison. Endocrinologa. 1a ed. Madrid. MacGraw-Hill Espaa, 2006: 137-138.

66.-Mujer de 42 aos. Acude a consulta por cefalea y acfenos. Tiene antecedente de DM en padre. EF: peso 78 kg, estatura 1.62, permetro abdominal 108 cm, TA: 140/100. Laboratorio: glucosa de ayuno 116, prueba de tolerancia a la glucosa, resultado a las 2 horas de 189. Los diagnsticos que se establecen en la paciente son: a) b) c) d) Obesidad y diabetes mellitus Sndrome metablico con intolerancia a la glucosa Obesidad e intolerancia a la glucosa Sndrome metablico y diabetes mellitus

GLUCOSA: <100 mg/dl = normal 100 y < 126 = glucosa de ayuno alterada (GAA) 126 mg/dl = diabetes mellitas

Glucosa a las 2 horas postcarga:

1 40 mg/dl = normal 1 40 a 1 99 mg/dl = intolerancia a la glucosa 200 mg/dl = diabetes mellitus

El sndrome metablico es la agrupacin de diferentes factores de riesgo asociados con el sndrome de resistencia a la insulina. En la literatura existen variaciones importantes en la prevalencia de este sndrome, dependiendo de los criterios o definicin que se empleen para su diagnstico. En la actualidad hay ms de cinco definiciones del sndrome metablico; la descrita en 2001 y su actualizacin en 2005 por el Programa Nacional de Educacin para el Colesterol, Panel de Tratamiento del Adulto III (NCEP-ATPIII), es la ms empleada en la prctica clnica.

OMS
DM o glucosa de ayuno anormal o intolerancia a la glucosa o resistencia a la insulina y 2 ms:

Para el diagnstico de Sndrome Metablico los criterios de la OMS (3) requieren el diagnstico de GAA ITG o DM2 ms dos de los siguientes elementos: Medicacin antihipertensiva y/o presin arterial alta (_ 140/ _90 mm Hg) Triglicridos: >/=150 mg/dL ( >/= 1,7 mmol/L) HDL col <35 mg/dL ( > 0,9 mmol/L) en hombres o < 39 mg/dL <1,0 mmol/L) en mujeres IMC > 30 kg/m2 y/o cintura/cadera > 0.9 en hombres, > 0.85 en mujeres Microalbuminuria >/= 20 g/min o albmina/creatinina _ 30 mg / g Por su parte, los criterios clnicos del NECP-ATPIII para el diagnstico de sndrome metablico son ligeramente distintos (1) Obesidad abdominal: CC: > 82 cm (mujer) y > 102 cm (hombre) Tiglicridos >150 mg/dL HDL col < 50 (mujer) o < 40 (hombre) Presin arterial >/= 130 / >/= 85 mm Hg Glucosa en ayunas >/= 110 mg/mL

Ntese que el NECP-ATPIII establece valores ms bajos para la presin arterial y la ADA para la glucosa en ayunas >/= 100 mg/dL)

67.- Se trata de femenino de 75 aos que desde hace dos semanas presenta ictericia. Hace dos das se aade confusin mental. A la EF FR 20, FC 110, TA 90/60, Temperatura 39, no responde a comandos verbales, pero se aleja del dolor que se provoca al palpar el hipocondrio derecho y epigastrio. Los datos clnicos en el caso anterior son sugestivos de:

a) b) c) d)

Coledocolitiasis Cncer de pncreas Cirrosis Colangitis

Diagnstico El diagnstico de colangitis se basa en la asociacin de signos y sntomas de infeccin con los propios de una obstruccin biliar. La presentacin clsica es la aparicin de dolor en hipocondrio derecho o epigastrio junto a fiebre, generalmente alta, con escalofros, e ictericia (trada de Charcot). Cuando se aade confusin mental y shock (sepsis) se denomina Pentada de Reynolds, que se observa con menor frecuencia, pero habitualmente se asocia con una colangitis supurada grave. Sin embargo, la correlacin entre la clnica tpica, las formas atpicas y la presencia de pus en la va biliar es pobre y en muchos casos de colangitis faltan algunos de estos rasgos. Algunos enfermos, sobre todo de edad avanzada, pueden tener confusin mental o shock sin fiebre, o existir una leucocitiosis con desviacin izquierda como nica manifestacin de la infeccin, por lo que se debe sospechar la existencia de una colangitis subyacente, sobre todo en enfermos de edad con algunas de estas manifestaciones. El laboratorio muestra hallazgos de obstruccin biliar con aumento variable de la bilirrubina y enzimas de colestasis. Es habitual encontrar leucocitosis con desviacin izquierda, siendo por lo general las cifras ms altas, en torno a 20.000 por mm3 , reflejo de las formas ms graves. La ecografa es la tcnica de eleccin para detectar la existencia de obstruccin biliar por su elevada eficacia y versatilidad, pudindose detectar tambin complicaciones de la colangitis, como el absceso heptico. Dentro de la colangitis se pueden establecer unas formas leves, generalmente de buen pronstico, y unas formas graves, donde se concentra la mayor parte de la mortalidad. Las primeras se manifiestan como cuadros febriles sin signos de afectacin sistmica que se autolimitan espontneamente o bajo tratamiento mdico en 24-48 horas. Las formas inicialmente graves son las que asocian confusin mental, hipotensin, shock o fracaso renal. La edad es un factor constante de aumento de la morbimortalidad.

Referencias bibliogrficas: 1. Bilhartz LE, Horton JD. Gallstone disease and its complications. En: Gastrointestinal and liver diseases. Sleisenger and Fordtran. Filadelfia: WB Saunders Co., 1998; 948-972. 2. Chung-Mau L, Chi-Leung L, Lai ECS, Sheuns-Tat F, Wong J. Early versus delayed laparoscopic cholecystectomy for treatment of acute cholecystitis. Ann Surg 1996; 223: 37-42. 3. Hamy A, Visset J, Likholatnikov D, Lerta F, Gibaud H, Savigny B et al. Percutaneus cholecystostomy for acute cholecystitis in critically ill patients. Surgery 1997; 121: 398401. 4. Harris A, Chong Hen Chang A, Torres-Viera C, Hammett R, Carr-Locke D. Meta-analysis of antibiotic prophylaxis in endoscopic retrograde cholangiopancreatography (ERCP). Endoscopy 1999; 31: 718-724. 5. Hermann RE. Surgery for acute and chronic cholecystitis. Surg Clin North Am 1990; 70: 1.263-1.275. 6. Koo Kp, Thirlby RC. Laparoscopic cholecystectomy in acute cholecystitis. What is the optimal timing for operation? Arch Surg 1996; 131: 540-545. 7. Lai ECS, Mok FPT, Tan ESY, Lo CM, Fan ST, You KT et al. Endoscopic biliary drainage for severe acute cholangitis. N Engl J Med 1992; 326: 1.582-1.586. 8. Marton KI, Doubilet P. How to image de gallbladder in suspected cholecystitis. Ann Int Med 1988; 109: 722-727. 9. Van den Hazel SJ, Speelman P, Tytgat GNJ, Dankert J, Van Leeuwen DJ. Role of antibiotics in the treatment and prevention of acute and recurrent cholangitis. Clin Infect Dis 1994; 19: 279-286. 10. Westphal J-F, Brogard J-M. Biliary tract infections. A guide to drug treatment. Drugs 1999; 57: 81-91.

68.- Masculino de 46 aos de edad con cirrosis heptica tuvo dolor abdominal generalizado durante 24 h sin nuseas ni vmitos. Su temperatura es de 38.3C y ha tenido distensin abdominal con onda de lquido claro. Hay hipersensibilidad difusa en la palpacin abdominal. En la paracentesis se obtuvo lquido transparente con 816 leucocitos/mm3 (85% polimorfonucleares, 15% linfocitos). La tincin de Gram no muestra bacterias. Cul de los siguientes diagnsticos es ms probable? a) b) c) d) Enfermedad ulcerosa pptica Pancreatitis Peritonitis primaria Colecistitis

El diagnstico ms probable es peritonitis primaria. Aunque es difcil diferenciar la peritonitis primaria (espontnea) por rotura de vscera hueca y contaminacin peritoneal, la presencia de fiebre y la leucocitosis la ascitis sugiere alguna clase de infeccin peritoneal. La pancreatitis se caracteriza por dolor localizado intenso (mesoepigstrico), que se irradia a la espalda. En general, las nuseas y vmitos no se relacionan con pancreatitis aguda. El dolor abdominal en caso de colecistitis se sita en el cuadrante superior derecho y suele haber nuseas y vmitos. El absceso heptico tiende a ser un trastorno subagudo sin datos peritoneales prominentes. En caso de ascitis crnica infectada, las enfermedades ulcerosas ppticas son causa poco probable. Allen R. M. MMS Medicina Interna. 5. Edicin. National Medical Series. Mc. Graw Hill. 2006. (captulo 8 V E 1 a).

69.- Masculino de 31 aos de edad que desde hace tiempo tiene episodios de ojo rojo con ausencia de secrecin. Refiere leve sensacin de cuerpo extrao y una carnosidad que crece y ya alcanz la crnea. A la exploracin se observa un tejido de neoformacin nasal que invade 1mm de la crnea. El diagnstico ms probable es:

a) b) c) d)

Carcinoma intraepitelial de conjuntiva Pterigion Pingecula Nevo conjuntival

El pterigin constituye una hiperplasia fibrovascular de carcter benigno de la conjuntiva bulbar que invade la crnea,1-4 est clasificado dentro de las degeneraciones no involutivas o tumoraciones epiteliales benignas corneales.1,3,5 Se localiza en la conjuntiva bulbar cerca del limbo corneal en el rea interpalpebral, a las 3 y 9 horas. Puede ser unipolar (solo afecta una parte) o bipolar, cuando afecta tanto la parte temporal como la nasal, es ms frecuente en el lado nasal. Tambin pueden ser unilaterales o bilaterales. Recibe su nombre por su aspecto de ala pequea. Presenta una forma triangular, cuyo vrtice mira hacia el rea pupilar y cuya base se orienta hacia la carncula en la localizacin nasal, y hacia el lado temporal en los localizados en el rea temporal. Habitualmente muestra un crecimiento horizontal que puede llegar a afectar el eje visual.2,3,7

Actualmente, en los estudios sobre alteraciones ultraestructurales del pterigin, se considera una alteracin inflamatoria y proliferativa de la superficie ocular.2 Se plantea que las clulas madre limbares se modifican con exposicin crnica a la luz ultravioleta, por lo que hay una rotura de la barrera limbar que causa la invasin conjuntival del epitelio corneal. Otro mecanismo que se expone relacionado con las reacciones inmunolgicas es una reaccin de hipersensibilidad tipo I a elementos irritantes exgenos (polvo, viento, etc.) asociada a una inflamacin local que causa un incremento en la produccin de IgE.3

Un estudio reciente muestra muestra una asociacin entre una pelcula lagrimal inestable y el inicio de un pterigin. Referencias bibliogrficas 1. Adamis AP, Stark T, Kenyon KR. The manegement of pterygium. Ophtamol Clin North Am. 1990;3(4):611 2. Klinworth GK. Chronic Actinic keratopathy, a condition associated with conjunctival elastosis (pingueculae) and typified by characteristic extracellular concretions. Am J Pathol. 1972;67:32. 3. Mac Kenzie FD, Hirst LW, Battistutta D. Risk analysis in the development of pterygia. Ophthalmology. 1992;99:1056-61. 4. Clear AS, Chirambo MC, Hutt MSR. Solar keratosis, ptert-gium, and squamous cell carcinoma of the conjunctiva in Malawi. Br J Ophthalmol. 1979;63:102-9.

70.- A 27-year-old woman has been sad for the last two weeks. She is fatigued a has a hard time concentrating at work. Just a few weeks earlier she was energetic an enthusiastic, and was able to work 10-12 hours a day with little sleep and go dancing at night. Her husband wants a divorce because he is tired of these constant ups and downs. The most accurate diagnosis is:

a) b) c) d)

Borderline personality disorder Seasonal mood disorder Dissociative identity disorder Cyclothymic disorder

Criterios para el diagnstico de

F34.0 Trastorno ciclotmico (301.13) A. Presencia, durante al menos 2 aos, de numerosos perodos de sntomas hipomanacos y numerosos perodos de sntomas depresivo que no cumplen los criterios para un episodio depresivo mayor. Nota: En los nios y adolescentes la duracin debe ser de al menos 1 ao. B. Durante el perodo de ms de 2 aos (1 ao en nios y adolescentes) la persona no ha dejado de presentar los sntomas del Criterio A durante un tiempo superior a los 2 meses.

C. Durante los primeros 2 aos de la alteracin no se ha presentado ningn episodio depresivo mayor, episodio manaco o episodio mixto. Nota: Despus de los 2 aos iniciales del trastorno ciclotmico (1 ao en los nios y adolescentes), puede haber episodios manacos o mixtos superpuestos al trastorno ciclotmico (en cuyo caso se diagnostican ambos trastornos, el ciclotmico y el trastorno bipolar I) o episodios depresivos mayores (en cuyo caso se diagnostican ambos trastornos, el ciclotmico y el trastorno bipolar II). D. Los sntomas del Criterio A no se explican mejor por la presencia de un trastorno esquizoafectivo y no estn superpuestos a una esquizofrenia, un trastorno esquizofreniforme, un trastorno delirante o un trastorno psictico no especificado. E. Los sntomas no son debidos a los efectos fisiolgicos directos de una sustancia (p. ej., una droga, un medicamento) o a una enfermedad mdica (p. ej., hipertiroidismo). F. Los sntomas provocan malestar clnicamente significativo o deterioro social, laboral o de otras reas importantes de la actividad del individuo.

71.- Masculino de 35 aos de edad que acude a consulta externa refiere que desde hace ms de 3 meses presenta una dermatosis diseminada a codos y rodillas con placas eritemato-escamosas. Sin antecedentes de importancia para el cuadro actual. Cul de los siguientes diagnsticos es el de mayor probabilidad? a) Dermatitis atpica b) Dermatitis seborreica c) Psoriasis d) Dermatitis de contacto

La psoriasis es una enfermedad crnica que evoluciona en brotes de causa desconocida, que se caracteriza por placas eritemato escamosas en diferentes partes de la piel. Se presenta por igual en hombres y en mujeres, en todas las edades, predominando en jvenes y ms frecuentemente en personas de piel blanca (parece que la presencia de melanina protege contra la enfermedad.

Tambin es ampliamente reconocido que la psoriasis es una enfermedad familiar y hereditaria (ditesis psorisica). Los sitios de predileccin para que aparezcan las lesiones, son los salientes seos como codos, rodillas, y la piel cabelluda as como la regin sacrocoxgea.

Christophers E, Krueger G G. Psoriasis. En: Fitzpatrick TB, EISEN AZ, Wolff K. Dermatologa en Medicina General Buenos Aires: Editorial Panamericana 1988. P. 585-591 Christophers E, Schubert C, Schrder J M. Psoriasis. Dermatologa. 1992; 45.

72.- Masculino de 27 aos que desde hace una semana inicia con erupcin maculopapular que afecta fundamentalmente a tronco, muslos y palmas, de forma simtrica. Las lesiones son rojas claras. Refiere Adinamia, astenia, con dolor de garganta y cefalea leve, no refiere fiebre. Tras realizarle, entre otras pruebas, RPR y FTA-Abs, ambas son positivas en suero, se le diagnostica de sfilis. Cul de las siguientes afirmaciones es correcta?

a) Si el paciente es VIH positivo la actitud no cambia. b) El tratamiento de eleccin es penicilina G benzatina. c) El tratamiento siempre debe incluir probenecid. d) Se trata de una sfilis primaria.

La penicilina es el antibitico de eleccin, con eficacia probada para cualquiera de los estados de la enfermedad. 1) Para la sfilis primaria se recomienda penicilina G benzatnica 2,4 MUI i/m en 1 sola dosis. Se prefiere repetir la dosis semanal de penicilina benzatnica (1 o 2 veces) cuando se presume que la personas no va a concurrir a los controles posteriores y en la embarazada. Con ello curan ms de 95% de los pacientes. Despus del tratamiento el VDRL se hace negativo en 1 ao en casi todos los casos.

2) El mismo tratamiento tambin cura a la gran mayora de los enfermos con sfilis secundaria, aunque algunos expertos recomiendan una segunda dosis con intervalo de 1 semana. El VDRL se negativiza en aproximadamente 2 aos, pero en algunos pacientes las pruebas serolgicas reagnicas siguen siendo reactivas, con niveles bajos de anticuerpos.

Para las personas alrgicas a la penicilina no embarazadas la alternativa es tetraciclina 2 g/d v/o fraccionados en 4 tomas diarias o doxiciclina 100 mg v/o c/12 horas, cualquera de ellas durante 14 das. Para personas que no toleran la tetraciclina, puede administrarse eritromicina 500 mg c/6 horas v/o por 14 das, aunque es menos eficaz y se han observado fracasos. Por eso cuando se usan drogas alternativas, el control posterior debe ser ms estrecho. Si se sospecha mala adherencia al tratamiento o que el paciente no va a concurrir a los controles, se prefiere desensibilizarlo y tratado con penicilina G benzatnica. Otra alternativa es la ceftriaxona 1 g/d i/m o i/v por 10 das. 3) La sfilis latente precoz se trata de igual manera. 4) Para los pacientes con sfilis latente tarda, latente de duracin no conocida y terciaria no neurosfilis, se recomiendan 7,2 MUI de penicilina G benzatnica i/m en 3 dosis con intervalo de 1 semana. En caso de alergia a la penicilina la alternativa es tetraciclina o doxiciclina en la forma dicha, por 4 semanas. Algunos expertos tratan a todos los que tienen sfilis cardiovascular con planes de neurosfilis. 5) No se recomienda penicilina G benzatnica para el tratamiento de la neurosfilis pues no alcanza niveles teraputicos en LCR. El plan para neurosfilis tanto sintomtica como asintomtica es de penicilina G cristalina 12 a 20 MUI/d i/v en 6 dosis diarias, por 10 a 14 das. Se menciona la alternativa de ceftriaxona 1 g/d i/m o i/v por 14 das, pero se han observado fallas teraputicas en pacientes infectados VIH con neurosfilis, sfilis latente o presunta sfilis latente. Aunque no se recomienda la ceftriaxona para el tratamiento de la neurosfilis, puede aceptarse como antibitico de alternativa para pacientes con neurosfilis y sfilis secundaria concomitante. Wardropper AG, Snow M. Neurosyphilis and HIV infection. Int J STD AIDS. 1994; 5(2):146-8. O'Mahony C, Rodgers C.A., Mendelsohn S.S., Sissons G., et al. Rapidly progressive syphilis in early HIV infection. Int J STD AIDS. 1997; 8(4): 275-78. Hook EW, Marra CM. Acquired syphilis in adults. N Eng J Med. 1992; 326 (16):1060-68. Guidelines for treatment of sexually transmitted disease. MMWR 1998; 47(RR-1):1-118. Hutchinsom CM, Hook EW. Sfilis en adultos. Clin Med N. A.1990; 6: 1451-76. Musher DM. Syphilis. Gorbach, Bartlett, Blacklow. Infectious diseases. Ed.Saunders. 1992: 822-28.

Charles D Ch. Sfilis. Sanford J P, Tyrrell D A J, Weller T H, Wolff S M. Infecciones obsttricas y perinatales. Doyma. 1994:252-69. Giovangrandi Y, Costa JM, Malka D, Belein V. Maladies infectieuses au cours de la grossesse (II). Rev Prat (Paris).1995; 45:2215-37.

73.- Al explorar a un paciente masculino de 40 aos de edad quien presenta una otoscopa sin alteraciones, una hipoacusia derecha, con Rinne negativo derecho y positivo izquierdo y Weber con lateralizacin al lado derecho. El diagnstico ms probable es:

a) Otitis serosa. b) Neuronitis vestibular. c) Hipoacusia sbita. d) Otosclerosis.

OTOSCLEROSIS

INTRODUCCION: OSTEODISTROFIA DE LA CAPSULA OTICA DEL LABERINTO. CLINICA: HIPOACUSIA: HIPOACUSIA PROGRESIVA TRANSMISIVA UNILATERAL , EN LA CUAL LA PACIENTE ENTIENDE MEJOR EN AMBIENTES RUIDOSOS.(PARACUSIA DE WILLIS) PROGRESIVA: AVANZA CON EL TIEMPO, EMBARAZOS. TRANSMISIVA EN INICIO. LUEGO PUEDE HACERSE NEUROSENSORIAL. UNILATERAL EN INICIO. LUEGO BILATERAL EN > DEL 75% PARACUSIA DE WEBER: NO OYE CON MASTICACION. VERTIGO : 10-20%. . SENSACION DE BORRACHERA OCASIONAL. ACUFENO: 75%. DE TONO GRAVE. MUY MOLESTOS. (ANSIOLITICOS) ALTS PSIQUICAS: IRRITABILIDAD, INTROVERSION, .HIPOCONDRIACOS,...NEUROSIS. EXPLORACION: OTOSCOPIA: NORMAL (MANCHA DE SCHWARTZE 10%) ACUMETRIA: RINNE NEGATIVO. WEBER LATERALIZADO AL LADO MAS AFECTO SCHWABACH : ALARGADO.(VIA OSEA PACIENTE Y EXPLOR) (TRIADA DE BEZOLD) PR.GELLE: NEGATIVO (DIAPASON EN FRENTEPRESION CON PERA DE POLITZER EN CAE.)

IMPEDANCIOMETRIA: CURVAS CENTRADAS Y EN OCASIONES ALGO APLANADAS. REFLEJO AUSENTE O INVERTIDO.(NO RECRUITMENT) TAC: SI VERTIGO. SE VE FOCO OTOESPONGIOTICO. AUDIOMETRIA: (+AUDIO VERBAL) ESTADIOS: FORMA TIPICA O DE POLITZER-SIEBENMANN: LA MAS FRECUENTE. PROGRESION LENTA.

1) HIPOACUSIA DE TRANSMISION CON UMBRAL DE 20-40 Dbs,PEOR EN FRECUENCIAS GRAVES. 2) HIPOACUSIA MIXTA: TRANSMISION DE 40-60 DBS Y PERCEPCION DE 20-30DBS (ESCOTOMA EN 2000Hzs-MUESCA DE CARHART). 3) SORDERA CON GRAN COMPONENTE NEUROSENSORIAL DE 6080DBS OTOSCLEROSIS DE LERMOYEZ: HIPOACUSIA TRANSMISIVA PURA. OTOSCLEROSIS COCLEAR DE MANASSE: DESDE INICIO COMPONENTE NEUROSENSORIAL.PURA

Bibliografa:
House J. Otosclerosis. Otolaryngol Clinics 1993;26(3):323-502. Jerger J. Clinical experience with impedance audiometry. Arch Otolaryngol 1970;92:311 Lempert J. Improvement in hearing in cases of otosclerosis: A new, one stage surgical technique. Arch Otolaryngol 1938; 28:42-97. Lippy WH, Schuring AG. Treatment of the inadvertently mobilized footplate. Otolaryngol Head Neck Surg 1973; 98:80-81. Meyer S. The effect of stapes surgery on high frequency hearing in patients with otosclerosis Am J Otol 1999; 20:36-40.

74.- Masculino de 4 aos de edad con antecedentes de otitis media a los doce meses de edad, actualmente cursa con otalgia, otorrea e hipoacusia, acompaado de malestar general. A la exploracin en conducto izquierdo se aprecia Hiperemia timpnica, hipervascularizacin y engrosamiento timpnico. El tratamiento de eleccin en esta patologa es:

a) Antihistamnicos de forma exclusiva b) Solo medidas generales c) Drenaje, AINES y esteroides locales d) Antibiticos, antihistamnicos y AINES.

Otitis media que se acompaa con secrecin y signos locales, sistmicos, o ambos. Instituto Nacional de la Nutricin. Salvador Zubiran Manual de teraputica mdica y procedimientos de urgencias. Cuarta edicin. Pag. 443 450. Mac Graw-Hill Interamericana. Mxico. La otitis media se define como la inflamacin del odo medio que se acompaa de secrecin y signos locales, sistmicos, o ambos, de enfermedad aguda. Esta enfermedad predomina en la infancia, sin embargo tambin existe en el adulto. Su distribucin tiene una clara periodicidad estacional, sobre todo en otoo e invierno. Los agentes de mayor prevalencia en nuestro pas son: Streptococcus pneumoniae, Haemophilus influenzae, Morexella catarrhalis estreptococo del grupo A y S. aureuse. Los antibiticos ms aceptados en estos casos son la amoxicilina, la amoxicilina con clavulanato, la calritromicina y el trimetropim con slfametoxazol. Se acepta el uso de antihistamnicos y antiinflamatorios no esteroideos para disminuir la congestin de la mucosa para resolver la obstruccin de la trompa farngotimpnica; en realidad no se ha demostrado efecto sobre la duracin de los sntomas. El drenaje del derrame solo esta indicado cuando ste persiste por ms de tres semanas.

75.- Masculino de 62 aos, refiere disnea de moderados esfuerzos y se le ausculta un primer tono fuerte, chasquido de apertura y soplo diastlico con refuerzo presistlico y en el ECG presenta ondas P con signos de crecimiento de la aurcula izquierda. El diagnstico ms probable es: a) b) c) d) Doble lesin mitral en ritmo sinusal. Estenosis mitral en ritmo sinusal. Insuficiencia artica en ritmo sinusal. Estenosis mitral en fibrilacin auricular, probablemente severa.

Estenosis mitral:

Los hallazgos se concentran en el distole:

- Despus del 2 ruido se ausculta un chasquido de apertura (momento en que se abre la vlvula mitral)

- Luego viene un soplo en decrescendo ("rodada mitral"): corresponde a la fase de llenado rpido.

- Acontinuacin, en pacientes en ritmo sinusal se puede escuchar un refuerzo presistlico, que es un soplo en crescendo y que se escucha inmediatamente antes del primer ruido (se debe a la contraccin de las aurculas).

- El primer ruido, tiende a ser de ayor intensidad (la explicacin sera que cuando se comienzan a contraer los ventrculos al comienzo del sstole, los velos de la vlvula mitral se encuentran en su mxima separacin ya que la aurcula estaba recin contrayndose, y esto, unido a las caractersticas de los velos, hara que el primer ruido sea ms intenso; sera equivalente a un "portazo"). Estos ruidos y soplos se auscultan mejor en el pex. La auscultacin puede resultar ms ntida si se pone a la persona en decbito semilateral izquierdo y se le pide que bote el aire y se mantenga un rato sin respirar. Convendra auscultar tanto con la membrana como con la campana del estetoscopio. Si la estenosis mitral ha generado hipertensin pulmonar secundaria, el cierre de la vlvula pulmonar (P2) es ms intenso y el segundo ruido se ausculta desdoblado porque se retrasa el cierre de la vlvula pulmonar. Tambin puede ocurrir que el ventrculo derecho se hipertrofie y sea palpable. Cuando el soplo de "estenosis mitral" se debe a la inflamacin de los velos por una enfermedad reumtica activa recibe la denominacin de soplo de Carey-Coombs. Manual de semiologa .2007 Universidad Catlica de Chile

76.- Al realizar un ensayo clnico controlado doble ciego para evaluar un nuevo antiinflamatorio, Qu condicin deben cumplir los participantes del estudio? a) Ni el observador ni los sujetos conocen cual grupo recibe el nuevo medicamento y cual el placebo b) Ni el grupo de estudio ni el grupo control conocen a los observadores c) Los sujetos del grupo control no conocen a los sujetos del grupo en estudio d) La asignacin del tratamiento no es conocida por los pacientes

Para considerar el efecto de placebo y reducir los sesgos debido a las concepciones de los pacientes y los investigadores el estudio puede conducirse bajo un patrn ciego. En un estudio doble ciego, la asignacin al tratamiento no es conocida por los pacientes ni por los mdicos.

Greenberg R. S; Epidemiologa mdica, Manual Moderno, 2. Ed. Pg. 116

77.- A 55-year-old woman presents to the Emergency Department with 8 hours of severe left lower quadrant abdominal pain, focal peritonitis, and guarding on physical examination. She has nonspecific abdominal radiographs and a leukocytosis. Which of the following is indicated at this point?

a) b) c) d)

IV antibiotics. CT scan. Colonoscopy. Surgery.

ABSCESO DIVERTICULAR: La formacin de un absceso diverticular complicado depende de la capacidad de los tejidos periclicos de controlar (localizar) la diseminacin del proceso inflamatorio. En general, los abscesos intra-abdominales se forman por: o Fuga anastomtica = 35% o Enfermedad diverticular = 23% Signos/Sntomas o fiebre+/- leucocitosis a pesar de antibiticos adecuados, tumoracin dolorosa Tratamiento o Absceso periclico pequeo - 90% responde a los antibiticos y manejo conservador. o Drenaje percutneo de los abscesos (DPA) es el tratamiento de eleccin para las colecciones simples, bien definidas. o 100% de los abscesos uniloculares simples se resolvieron con DPA y antibiticoterapia. Schauer P, Ramos P, Ghiatas A, Sirinek K. Virulent diverticular disease in young obese men.Am J Surg; 164:443-8. Pubmed-Medline

78.- Se trata de femenio de 28 aos con desarrollo de abceso puerperal en mama izquierda, si patolgicos sistmicos de importancia para su actual padecimiento, la medida inicial ms importante es: a) b) c) d) Antibiticos. Drenaje quirrgico del absceso Instituir una terapia con antiinflamatorios. Suspender la lactancia.

En el desarrollo del absceso puerperal hay inicialmente dolor, enrojecimiento, induracin; en las fases tempranas de la celulitis los antibiticos son muy efectivos, pero una vez formado el absceso, al igual que en el absceso no relacionado con la lactancia, se impone el drenaje quirrgico. El drenaje debe ser amplio, y en los abscesos no puerperales es necesario abrir los tabiques que con frecuencia se forman en la cavidad del absceso. Muchos autores preconizan el simple drenaje sin cubrimiento antibitico. Si se usan antibiticos, stos deben ser debidamente seleccionados de acuerdo con el germen, recordando que el ms frecuente es el Staphylococcus aureus.

Beller FK, Galask RP. Infections of the breast. En: Infectious Diseases. Edited by SL Gorbach, JG Bartlett, NR Blacklow. WB Saunders Co. Philadelphia, 1992 Benson EA. Management of breast abscesses. World J Surg 23:753, 1989 Bundred NJ, Dixon JMJ, Lumsden AB, et al. Are the lesions of duct ectasia sterile? Br J Surg 72:844, 1985 Ingham HR, Freeman R, Wilson RG. Anaerobic breast abscesses. Lancet 1:165, 1979 Scholefield JH, Duncan JL, Rogers K. Review of hospital experience of breast abscesses. Br J Surg 74:469, 1987

79.- Se realiza una endoscopia digestiva a un paciente de 57 aos, se reporta el diagnstico de sospecha de esfago de Barrett Cul es entre las siguientes, la conducta ms adecuada?

a) Esperar el resultado de la biopsia antes de tomar decisiones, para corroborar y tener seguridad del diagnstico b) Indicar tratamiento mdico con inhibidor de la bomba de protones 40 mg da en ayuno durante 1 ao y regresar a valoracin. c) Prepararlo para la realizacin de funduplicatura. d) Realizar esofaguectoma.

Esfago de Barret. La Enfermedad por Reflujo Gastroesofgico es una de las causas ms frecuentes de consulta mdica en la prctica diaria y esta se puede acompaar de Esofagitis por Reflujo (es la inflamacin del esfago causada por el reflujo del contenido gstrico al esfago), La Esofagitis por Reflujo se diagnostica mediante Endoscopa, y este procedimiento permite la biopsia, la cual nos permite confirmar la presencia de Esfago de Barret, una complicacin poco frecuente pero de gran relevancia clnica. El Esfago de Barret es una lesin premaligna, que predispone al Adenocarcinoma Esofgico, patologa neoplsica que ha aumentado su incidencia en los ltimos aos, incluso desplazando al Carcinoma Epidermoide.

DIAGNOSTICO. IPresentacin Clnica. La presencia de EB no provoca sntomas por s mismo. Ms bien, los sntomas son una consecuencia de la Enfermedad por Reflujo Gastro-Esofgico (ERGE) o sus complicaciones. La mayora de los pacientes tienen antecedentes prolongados de sntomas de RGE como pirosis y regurgitacin (2, 6, 10). Otros pacientes, aproximadamente la tercera parte, se presentan oligo asintomticos (2, 8, 13), lo que sugiere una disminucin de la sensibilidad al reflujo cido por parte del epitelio de Barret. De hecho, ms del 90% de los pacientes con EB no buscan atencin mdica y el trastorno pasa inadvertido hasta que el proceso se complica por el desarrollo de cncer, debutando con disfagia (por formacin de estenosis) o con hemorragia digestiva (por ulceraciones profundas de la mucosa lesionada). (10) IIFibroendoscopa Digestiva Alta con toma de Biopsia. La endoscopia con biopsias dirigidas constituye el patrn oro para el diagnstico del EB (2, 6). Sin embargo, debido a la gran cantidad de pacientes con RGE y la baja frecuencia de EB, no parece costo-efectivo el realizar endoscopia a todos ellos (13). Se ha sugerido que los siguientes pacientes con RGE deben ser sometidos a una endoscopia (9): RGE complicado (disfagia, estenosis, lcera, hemorragia); RGE con esofagograma que muestre patrn reticular o seudomembranas; RGE con sintomatologa persistente a pesar del tratamiento; RGE asociado a esclerodermia.

Se sospecha EB en la endoscopia por la presencia de epitelio color naranja a rojo, aterciopelado, de tipo gstrico; que contrasta con el color rosado blanquecino del epitelio esofgico normal (1, 2, 4, 8). La lnea de transicin entre ambos epitelios (cambio mucoso) puede ser regular (circunferencial) , ms frecuentemente, irregular (en forma de lengetas o islotes)(1, 2, 8,). La extensin de la metaplasia tambin es variable, pudiendo abarcar desde 2 cm de longitud a partir de la unin gastro-esofgica (UGE) o puede extenderse incluso hasta el esfago cervical (1). Para confirmar el diagnstico debe obtenerse la biopsia. Las muestras deben tomarse, fundamentalmente, del lmite de la UGE y hasta 1 2 cm por debajo de la misma, ya que esta es la zona de mayor riesgo de desarrollar un Adenocarcinoma, y es en esta parte ms proximal del segmento metaplsico donde se sita, principalmente, el epitelio columnar especializado (2). Por supuesto que deben tomarse muestras para biopsias de todo el segmento metaplsico y, especialmente, de aquellas zonas en las que se observe alguna alteracin macroscpica. Microscpicamente pueden identificarse tres tipos de epitelio de Barret (1, 2, 4, 8) Metaplasia gstrica fndica: es similar al epitelio del cuerpo o del fondo gstrico, y presenta clulas parietales y principales; Metaplasia gstrica cardiaca o transicional: como el epitelio del cardias gstrico, que exhibe acmulos profundos de glndulas mucosas y criptas; y Metaplasia tipo intestinal o tambin llamado epitelio columnar especializado: que tiene caractersticas de la mucosa gstrica y de la intestinal, que presenta clulas caliciformes dispersas entre las clulas cilndricas.

BIBLIOGRAFIA 1. Badaloni AE. Enfermedad por reflujo gastroesofgico. En: Ferraina P, Oria A, ed. Ciruga de Mitchans, Vol 1. 5 edicin. Buenos Aires, Argentina. Editorial El Ateneo. 392-399. 2. Cucarella JF. Esfago de Barret. Gaceta de la sociedad espaola de patologa digestiva [en lnea] 1999. [fecha de acceso 10 de noviembre de 2004] . URL. 3. Beers MH. Trastornos del esfago. Cncer esofgico. En: El Manual Merck, 17 edicin. Edicin electrnica en CD-ROM. Madrid, Espaa: Ediciones Harcourt: 1999; seccin 3, cap 20. 4. McGarrity TJ. Barret`s esophagus: the continuing conundrum. Surveillance should be confined to the surgically fit. BJM 2000; 321: 1238-1239 5. Heading R. Es el seguimiento endoscpico del esfago de Barret una prdida de tiempo y esfuerzo? XXX Congreso Chileno de Gastroenterologa. Medwave. Ao 4. N 2. [en lnea] Edicin Marzo 2004. [fecha de acceso 28 de diciembre de 2004]. URL. 6. Murra Saca JA. Esfago de Barret. Atlas de Video Endoscopia Gastrointestinal de El Salvador.[en lnea] 2004 [fecha de acceso 10 de noviembre de 2004] . URL. Disponible en: 7. Ayre AM, Benitez Fernandez A, Cocco JE, y col. Tratamiento del cncer de esfago: revisin. Revista de Postgrado de la VI Ctedra de Medicina [en lnea] 2003 [fecha de acceso: 15 de enero de 2005]; 126: 37-41. URL. Disponible en:

8.

Crawford JM. El tracto gastrointestinal. Esfago de Barret. En: Cotran, Kumar, Robbins, ed. Patologa Estructural y Funcional. 5 edicin. Madrid, Espaa: Ediciones McGraw-Hill-Interamericana. 845 9. Rodriguez A. Esfago de Barret. Boletn de la Escuela de Medicina. Universidad Catlica de Chile. [en lnea] 1998 [fecha de acceso: 28 de diciembre de 2004] Vol 27. N 1. 1998. URL. Disponible

80.- Lactante menor prximo a cumplir 2 meses, atendido en consulta para control de nio sano. Se refiere asintomtico. E.F.: dentro de lmites normales. Antecedente: esquema de vacunacin completo para la edad. De acuerdo a la cartilla nacional de vacunacin, que vacunas ya recibi: a) b) c) d) Polio y bcg. Hepatitis b y bcg. Solo bcg. Polio y hepatitis b.

81.- Femenino de 34 aos que inicia tratamiento con sulfato de magnesio por presentar eclampsia, se presentan signos de sobre dosificacin, el antdoto que debe emplearse en esta paciente es:

a) Gluconato clcico b) Nitroprusiato. c) Simpaticomimticos. d) Carbonato sdico.

NIVEL DE PRIMER CONTACTO (ATENCION PRIMARIA) Se debe instruir a todas las embarazadas que deben acudir inmediatamente a un centro de salud en cualquiera de los siguientes casos: Edema que se desarrolla rpidamente (en pocos das) Cefalea severa y persistente Dolor en la regin abdominal superior Visin borrosa

Se debe realizar la medicin de la presin arterial y un anlisis de orina para la deteccin de proteinuria a las mujeres que acudan a centros de salud presentando estos sntomas.

Convulsiones:
Si se asiste a una mujer con eclampsia en un centro de atencin primaria, 1. Deben mantenerse las vas respiratorias permeables. 2. Se debe colocar a la mujer de costado (posicin decbito lateral izquierda) para evitar la aspiracin del vmito u otras secreciones. 3. Si es posible, se debe establecer una va intravenosa. 4. Se debe administrar sulfato de magnesio. Monitoreo de la administracin de sulfato de magnesio: Durante el tratamiento con sulfato de magnesio, se recomienda realizar un control cada 4 horas, como mnimo, para detectar la presencia de: Reflejo rotuliano, frecuencia respiratoria superior a 16 por minuto, volumen de orina >100 ml en las 4 horas previas.

- Sobredosis de sulfato de magnesio: Todo centro de salud que utilice sulfato de magnesio debe disponer de ampollas de gluconato de calcio (1 g) como antdoto para la sobredosis de dicho frmaco. Se sugiere medir la presin arterial y administrar antihipertensivos segn corresponda. Convulsiones recurrentes: en caso de convulsiones recurrentes, se administran otros 2 a 4 g de sulfato de magnesio por va IV en el lapso de 5 minutos, tanto para el rgimen IM como el IV; la dosis se determina en funcin del peso de la paciente.

El sulfato de magnesio es un frmaco usado en el control de las convulsiones eclmpticas, para suprimir o controlar las contracciones uterinas sean estas espontneas o inducidas, y como broncodilatador luego del uso de beta agonistas y agentes anticolinergicos. Tambin tiene indicacin como terapia de reemplazo en la deficiencia de magnesio, como laxante para reducir la absorcin de txicos del tracto gastrointestinal. El sulfato de magnesio esta ganando popularidad como tratamiento de inicio en el manejo de algunas arritmias, particularmente en Torsades de Pointes, y en arritmias secundarias a sobredosis de antidepresivos tricclicos o toxicidad digitlica. Esta tambin considerado clase Ila (probable beneficio) para la fibrilacin ventricular refractaria y la taquicardia ventricular, luego de la administracin de dosis de lidocaina y bretilio.

FARMACODINAMIA El sulfato de magnesio tiene la capacidad de alterar la excitabilidad de la fibra miometrial, afecta el acoplamiento excitacin contraccin y el proceso mismo de contraccin, inhibe la entrada de calcio al sarcoplasma y reduce la frecuencia de los potenciales de accin. Inhibe tambin la liberacin de acetilcolina. Por ser estas acciones comunes en las fibras musculares se pueden ver afectadas tambin la musculatura voluntaria e incluso las fibras miocrdicas.(1)

Bibliografa:
Graves C. Frmacos que contraen o relajan el tero. En: Hardman J, Limbird L, Molinoff P, Ruddon R, Goodman A, eds. Goodman & Gilman. Las Bases Farmacolgicas de la Teraputica. 9 ed. Mxico DF: McGraw-Hill Interamericana; 1996. pp. 1012-3.

82.- Se trata de paciente de 34 aos que cursa con 39 SDG; a la exploracin fsica reflejos patelares hiperactivos, inquieta, se reportan cifras de TA 145/95, se realiza laboratorio que reporta proteinuria 2+,. El diagnstico ms probable es:

a) Glomerulonefritis aguda b) Hipertensin esencial c) feocromocitoma d) Preeclampsia

La hipertensin es la complicacin mdica ms comn del embarazo , aunque para algunos autores es la segunda complicacin mdica del embarazo slo despus de la anemia; es ms frecuente en jvenes durante el primer embarazo y en nulparas de mayor edad, hipertensas previas y diabticas. En Mxico, tambin es la complicacin ms frecuente del embarazo, la incidencia es de 47.3 por cada 1 000 nacimientos y es adems, la primera causa de ingreso de pacientes embarazadas a las unidades de terapia intensiva (debido a hemorragia masiva, para recibir soporte hemodinmico), segn la secretara de salud (2001) la mortalidad por complicaciones del embarazo ocupa el 15 lugar en la mortalidad hospitalaria en general. Adems, la tasa de preeclampsia se ha incrementado 40% en el periodo entre 1990 y 1999 y constituye hasta 40% de los partos prematuros iatrognicos.
.

Preeclampsia La preeclampsia es un sndrome clnico caracterizado por hipertensin con disfuncin orgnica mltiple, proteinuria, edemas. Es definida como un incremento de al menos 140/90 mmHg despus de la semana 20 de gestacin, un incremento en la presin sangunea diastlica de al menos 15 mmHg respecto a un nivel previo a la semana 20 combinado con proteinuria (> 300 mg en 24 horas). Las mediciones de la presin arterial citadas deben ser medidas al menos 2 ocasiones con por lo menos 6 horas de separacin. La proteinuria puede ser una toma simple de orina al azar que indique al menos 30 mg/dL 3 ++ en dos muestras de orina1 segn el tipo de prueba. El criterio del incremento de 30 mmHg en la presin sistlica y/o 15 mmHg en la presin diastlica respecto a valores previos a la semana 20 de gestacin ha sido eliminado por ser poco especfico15

1.Myers JE, Baker PN. Hupertensive diseases and eclampsia. Curr Opin Obstet Gynecol 2002; 14: 119-125. 2. Tierney, McPhee, Papadakis. Diagnstico clnico y tratamiento 2003. 38 ed, Mxico, Manual Moderno, 2003: 770-773. 3. Wilson MI, Goodwin TM, Pan VI, Ingles SA. Molecular epidemiology of preeclampsia. Obstet and Gynecol Survey 2003; 58(1):39-66.

4. Burrow GM. Complicaciones mdicas durante el embarazo. 4 ed, Mxico, McGraw-Hill panamericana: 1996: 1-25. 5. Guyton AC, Hall JE. Embarazo y lactancia en: Tratado de fisiologa mdica, 10 ed, Mxico, McGraw-Hill Interamericana 2001: 1135-45. 6. Vaticon D. Fisiologa de la fecundacin, embarazo, parto y lactancia, en: Tresguerres JAF. Fisiologa Humana. Mxico, Interamericana McGraw-Hill, 1992: 1086-1109. 7. Pridjian G, Puschett JB. Preeclampisa. Part 1: Clinical Considerations. Obstet and Gynecol Survey 2002; 57 (9): 598-618. and Pathophysiologic

8. Pridjian G, Puschett JB. Preeclampisa. Part I1: Experimental and Genetic Considerations. Obstet and Gynecol Survey 2002; 57 (9): 619-40. 9. IMSS. Embarazo de alto riesgo. Gua diagnstica teraputica. Rev Med IMSS 1998; 36(1):45-60.

83.- Masculino de 10 aos de edad que inici su padecimiento hace 24 hrs al presentar, posterior a la ingesta de pescado, evacuaciones diarreicas, dolor abdominal, nusea y vmito; acompandose de cefalea, parestesias periorales y bradicardia. El da de hoy se agreg la presencia de ataxia y refiere que se quema al tocar el agua fra. Cul es el agente etiolgico ms probable?

a) Vibrio cholerae
b) Ciguatera c) Agente Norwalk

d) StaphIlococcus aeurus

U.S. Food & Drug Administration - Center for Food Safety & Applied Nutrition Foodborne Pathogenic Microorganisms and Natural Toxins Handbook. La intoxicacin por ciguatera se produce por ingerir pescados como dorado, barracuda, cubera, etc. contaminados con macroalgas con ciguatoxina-1. la evolucin de la intoxicacin es la siguiente: en las primeras 12 horas se presentan manifestaciones gastrointestinales como dolor abdominal, nusea, vmito y diarrea; posteriormente se agregan datos neurolgicos como calambres, parestesias, mialgias, fatiga, ataxia, y disgeusia y finalmente alteraciones cardiovasculares (bradicardia, taquicardia o hipotensin.) Los datos principales son las parestesias periorales y la inversin de la sensacin trmica, es decir sentir fro al tocar objetos calientes y viceversa. Adems se puede presentar ceguera temporal, parlisis de los msculos faciales, oftalmoplejia, espasticidad, delirio, sensacin de prdida de los dientes, exacerbacin del acn, sialorrea, cada del cabello, uas y descamacin de las piel.

84.- Masculino de 6 aos acude al servicio de urgencias por salida de scaris por el ano. A la exploracin fsica presenta dificultad y sibilancia respiratoria, abdomen con hepatomegalia y distensin abdominal, se confirma el diagnstico por USG. El tratamiento de eleccin en este paciente es: a) b) c) d) Secnidazol va oral Metronidazol va oral Salbutamol Inhalado Albendazol va oral

Tratamiento Se realizar tratamiento farmacolgico por va oral, siendo las drogas de eleccin el albendazol, el pamoato de pirantel y oxantel y la clsica piperacina La ascaridiasis intestinal es una patologa ocasionada por un helminto que parasita el tubo digestivo del ser humano. Es la parasitosis ms frecuente y cosmopolita de todas las helmintiasis humanas, se calcula ms 1.5 billones de portadores en el mundo de los cuales 51 millones de afectados son nios. La gran mayora de los casos cursa en forma asintomtico o pueden producirse signos inespecficos como dolor abdominal difuso o ir acompaado en los casos crnicos por signos de desnutricin, dficit de crecimiento y retardo del aprendizaje. Los efectos patolgicos producidos por scaris en el organismo humano, se presentan en varios sitios de acuerdo a la localizacin de las diversas formas evolutivas. Las larvas al pasar por el pulmn producen ruptura de los capilares y de la pared alveolar. Cuando ocurre en forma masiva da origen al sndrome de Leffler que se caracteriza por lesiones mltiples de los alvolos donde producen granulomas de cuerpo extrao, el cual se observa a los rayos X como opacidades diseminadas. Cuando el nmero de parsitos es grande puede ocasionar complicaciones tales como obstruccin intestinal, vlvulos, y perforacin visceral con peritonitis, complicaciones que deben ser resueltas en forma quirrgica. Las mayores complicaciones se presentan por las migraciones de scaris adultos a diferentes sitios del organismo. La invasin del parsito al rbol biliar, puede causar ictericia obstructiva, colecistitis acalculosa, colangitis, pancreatitis, abscesos hepticos. Se han descrito diversas modalidades de tratamiento, La migracin masiva de parsitos hacia el rbol biliar se asocia con procesos infecciosos, para lo cual deben administrarse antibiticos aun en forma profilctica. La administracin de albendazol se considera el tratamiento antihelmntico de eleccin por su mayor excrecin biliar. En un bajo porcentaje (21.2%) se logra expulsar los parsitos de la vescula biliar.

Si no se logra expulsarlo, si persiste la sintomatologa o si el paciente empeora, es necesario someterlo a una laparotoma exploradora para realizar colecistectoma. Los pacientes pueden caer en una septicemia fatal, si no llevamos a cabo la teraputica en forma adecuada, lo que hace que la evolucin y el pronstico empeore. La parasitosis es uno de los grandes problemas de la salud pblica. La OMS la considera una de las principales causas de morbilidad; estrechamente ligada a la pobreza y relacionada con inadecuada higiene personal y de los alimentos crudos, falta de servicios sanitarios, falta de provisin de agua potable y contaminacin fecal del ambiente. Infecta a personas de todas las edades, pero la sufren principalmente los nios, a lo que les causa trastornos del crecimiento y desarrollo. Desde hace aos la OMS propone como solucin, aunque sea parcial, el uso de tratamientos antihelmnticos masivos y reiterados, sin diagnstico parasitolgico previo individual, en aquellas comunidades que tengan una elevada prevalencia de geohelmintos (>50%).

Bibliografa:
Cook GC, Zumla AI (edit). Mansons Tropical Diseases. 21th ed. London: Elsevier Science; 2002 Garca LS. Diagnostic Medical Parasitology. 4th. Ed. Washington: American Society for Microbiology; 2002 Gill G. Lectura Notes on Tropical Medicine. 5th ed. Blackwell Scientific Pub; 2004 Mensa JM, Gatell MT, Jimnez de Anta G, Prats A, Dominguez-Gil A. Gua terapetica antimicrobiana. 14 ed. Barcelona: MASSON; 2004. Murray PR, Baron EJ (Edit.)Manual of clinical microbiology. 8th edit. Washington: ASM Press; 2003 Villa Luis F. Gua de Terapia Farmacolgica. Medimecum. Barcelona: ADIS Internacional; 2005.

85.- Se trata de paciente femenino de 42 aos diagnosticado con sndrome de Cushing al encontrar una ACTH muy baja o suprimida, la primera posibilidad de diagnstico es:

a) b) c) d)

Enfermedad hipotlamo-hipofisiaria Hiperplasia suprarrenal congnita Adenoma suprarreanal Adenoma hipofisiario

Aproximadamente 20 a 25% de los pacientes con sndrome de Cushing tienen una neoplasia suprarrenal. En la mayora de estos casos existe una produccin autnoma de cortisol por la neoplasia, lo que conduce a descenso en los niveles de ACTH a rangos indetectables o menores a 2 pmol/L o 10 pg/ml, por retroalimentacin negativa de la secrecin de ACTH. Williams GH, Dluhy RG. Enfermedades de la corteza suprarrenal. En Jameson JL (ed): Harrison. Endocrinologa. 1a ed. Madrid. MacGraw-Hill Espaa, 2006: 126-127.

86.- Masculino de 68 aos con antecedentes de dolor lumbar de 1 mes meses de evolucin asi como malestar generalizado. Refiere cefaleas, visin borrosa, sed intensa y molestias digestivas. Con datos de deshidratacin, TA de 170/100 mmHg. Presenta una hemoglobina de 9.5 g/dl, VSG 90, acidosis metablica, creatinina 2.7 mg/dl, protenas totales 10.2 con un pico monoclonal de IgG tipo l, y una calcemia de 15 mg/dl. Se inicia tratamiento de urgencia el cual incluye el uso de esteroides, difosfonatos, expansin con suero salino y por sus condiciones el uso del siguiente diurtico. a) Furosemida. b) Clortalidona. c) Acetazolamida. d) Hidroclorotiazida.

DESCRIPCION La furosemida es un diurtico de asa de la familia de las sulfonamidas utilizado en el tratamiento del edema asociado a la insuficiencia cardaca congestiva, cirrosis y enfermedad renal, incluyendo el sndrome nefrtico. Tambin se utiliza en el tratamiento de la hipertensin ligera o moderada y como adyuvante en las crisis hipertensivas y edema pulmonar agudo. La furosemida es empleada, asimismo, para el tratamiento de la hipercalcemia. Pertenece al grupo de los diurticos de alto techo. Mecanismo de accin: el mecanismo de accin de la furosemida no es bien conocido. La furosemida no se une a los grupos sulhidrilo de las protenas renales como hace el cido etacrnico, sino que parece ejercer su efecto diurtico inhibiendo la resorcin del sodio y del cloro en la porcin ascendiente del asa de Henle. Estos efectos aumentan la excrecin renal de sodio, cloruros y agua, resultando una notable diuresis. Adicionalmente, la furosemida aumenta la excrecin de potasio, hidrgeno, calcio, magnesio, bicarbonato, amonio y fosfatos. In vitro, la furosemida inhibe la anhidrasa carbnica pudiendo ser este efecto el responsable de la eliminacin del bicarbonato. La furosemida no es un antagonista de la aldosterona. Despus de la administracin de furosemida disminuyen las resistencias vasculares renales aumentando el flujo renal, occurriendo lo mismo en las resistencias perifricas, lo que se traduce en una reduccin de la presin en el ventrculo izquierdo. Si inicialmente la furosemida tiene un efecto antihipertensivo debido a una reduccin de la volemia aumentando la velocidad de filtracin glomerular y reduciendo el gasto cardaco, ms tarde el gasto cardaco puede volver a su valor inicial pero las resistencias perifricas permanecen bajas, lo que resulta en una reduccin de la presin arterial.

Farmacocintica: La furosemida se administra por va oral y parenteral. La absorcin oral de este frmaco es bastante errtica y es afectada por la comida, si bien esta no altera la respuesta diurtica. La diuresis se inicia a los 30-60 minutos despus de la administracin oral y a los 5 minutos despus de la administracin intravenosa. El frmaco se une extensamente a las protenas del plasma (95%), atraviesa la barrera placentaria y se excreta en la leche materna. La furosemida experimenta un mnimo metabolismo en el hgado eliminndose en su mayor parte en la orina. Aproximadamente el 20% de la dosis se excreta en las heces, si bien este porcentaje puede aumentar hasta el 98% en los pacientes con insuficiencia renal. La semi-vida plasmtica es de 0.5 a 1 hora aunque aumenta significativamente en los neonatos y en los pacientes con insuficiencias renal o heptica en los que se deben reducir las dosis

CONTRAINDICACIONES: FUROSEMIDA est contraindicada en pacientes con anuria y en pacientes con antecedentes de hipersensibilidad al frmaco. La administracin de FUROSEMIDA se debe interrumpir durante el tratamiento de la enfermedad renal progresiva severa si ocurre azoemia creciente y oliguria. BIBLIOGRAFA 1. Reynolds J, Martindale W. The Extra Pharmacopoeia. 31 ed. London: Royal Pharmaceutical Society; 1996. pp. 871. 2. Kradjan W. Congestive Heart Failure. In: Young L, Koda-Kimble M, eds. Applied Therapeutics: The clinical use of drugs. 6 ed. Vancouver: Edit. Applied Therapeutics Inc.; 1999. pp. 15-4.

87.- Al encontrarse realizando exploracin fsica por dolor abdominal, usted identifica signo de Murphy positivo, esto se traduce en: a) Dolor a la palpacin profunda en el punto cstico b) Dolor a la palpacin en el punto cstico durante la inspiracin c) Dolor a la palpacin profunda en el punto cstico durante la espiracin d) Dolor que interrumpe la inspiracin a la palpacin profunda en el punto cstico

Maniobra de Murphy, el enfermo sentado y el examinador colocado atrs engancha el punto cistico mientras el enfermo trata de realizar una inspiracin profunda, Es positiva en la cocolecistitis aguda. Apuntes de Semiologa Semiologa mdica y tcnica exploratoria Escrito por Antonio Surs Batll, Juan Surs Batll

88.- Masculino de 39 aos, obeso, no fumador, con asma desde hace ao y medio, que empeora despus de las comidas y se despierta por las noches con tos irritativa. Sigue tratamiento con broncodilatadores y esteroides. Cul sera la mejor prueba diagnstica para aclarar el cuadro? a) b) c) d) Endoscopia bronquial. Pruebas de provocacin con alergenos. pH esofgico durante 24 horas. Radiografa esofagogastroduodenal

INDICACIONES DE LA pH-METRA La pHmetra intraesofgica de 24 horas es una prueba con una alta sensibilidad y especificidad para el diagnstico de RGE En general, la pHmetra estar indicada en tres tipos de situaciones: a) Cuando existen sntomas sugestivos de RGE y la evolucin no es favorable a pesar de instaurar el tratamiento correcto. b) Cuando quiera establecerse la relacin entre RGE y sntomas extradigestivos. c) Como control de la eficacia del tratamiento, ya sea mdico o quirrgico. Desde un punto de vista prctico pueden diferenciarse las situaciones en las que puede o no ser de utilidad la realizacin de una pHmetra. 1. pHmetra innecesaria En general, no ser necesario realizar una pHmetra para diagnosticar el RGE cuando el diagnstico ya est hecho mediante otra prueba o cuando sea evidente por la clnica.

1 Esofagitis pptica. La pHmetra no ser necesaria para hacer el diagnstico de RGE si la histologa o la endoscopia han demostrado la existencia de una esofagitis pptica. 2. pHmetra poco til. Existen una serie de situaciones en las que los datos ofrecidos por la pHmetra contribuyen poco para decidir el tratamiento o establecer el pronstico del paciente. 1.- Estudio del paciente con disfagia. El trnsito digestivo, la endoscopia con toma de biopsias y la manometra esofgica suelen proporcionar informacin mucho ms til en este tipo de patologa. 2.- Estudio de trastornos motores. Estara indicada la manometra. Si el trazado muestra alteraciones sugestivas de esofagitis, sta deber confirmarse mediante endoscopia. 3.- Estudio del reflujo alcalino. La pHmetra convencional de un solo canal no proporciona informacin suficiente para el estudio del reflujo alcalino. La colocacin de un electrodo gstrico amplia esta informacin, aunque hay que tener en cuenta que puede existir reflujo duodenogastroesofgico sin alcalinizacin gstrica ni esofgica.

3. pHmetra til 3.1 Pacientes con pausas de apnea. Los episodios de apnea de origen perifrico, producidos durante el sueo y de larga evolucin son los que con ms probabilidad pueden ser debidos a RGE y en los que ms til puede ser la pHmetra. No es suficiente con demostrar la existencia de reflujo, sino que debe establecerse la relacin entre ste y la apnea mediante la realizacin de un registro pneumocardiogrfico mltiple simultneo. No ser necesario realizar pHmetra en aquellos casos en los que la relacin entre las pausas de apnea y los episodios de reflujo sea clnicamente evidente. 3.2 Episodios aparentemente amenazadores para la vida. Al igual que en el grupo anterior, la pHmetra deber formar parte de un registro mltiple de frecuencia cardiaca, respiratoria y pulsioximetra para establecer la relacin del reflujo con estos episodios. 3.3 Asma refractario al tratamiento. Ms de la mitad de los asmticos presentan RGE concomitante. Sin embargo, el reflujo puede ser tanto causa como consecuencia de la patologa respiratoria, por lo que lo ideal es que la pHmetra pueda demostrar una relacin temporal entre los episodios de reflujo y la aparicin de las sibilancias. En los casos en que esto no sea posible, puede ser til detectar un patrn de RGE consistente en episodios prolongados durante el periodo nocturno. No est claramente demostrada la necesidad de reflujo a nivel proximal para la produccin de los sntomas respiratorios, por lo que puede ser suficiente con la realizacin de una pHmetra convencional con un solo electrodo esofgico.

3.4 Otros sntomas respiratorios. Puede utilizarse para el diagnstico del RGE oculto en pacientes con tos crnica, neumona recurrente o aspiraciones pulmonares, buscando la presencia de reflujos largos durante el sueo. Aunque lo ideal es demostrar la relacin temporal del reflujo con la patologa respiratoria, esto puede ser difcil debido al carcter intermitente de la misma. 3.5 Patologa ORL. El RGE puede ser la causa de diversa sintomatologa ORL, como estridor, laringitis e, incluso, sinusitis refractarias al tratamiento convencional. Es menos probable que otras patologas como la otitis recurrente, la disfona o la papilomatosis larngea sean debidas a reflujo. En estos casos s estara indicada la realizacin de pHmetra de doble canal, situndose el electrodo proximal inmediatamente por debajo del esfnter esofgico superior. 3.6 Control del tratamiento mdico. Indicada para valorar la eficacia del tratamiento del RGE moderado-severo previamente diagnosticado mediante pHmetra. En los casos con mala evolucin podr realizarse tras 4-8 semanas de tratamiento. Si la evolucin es favorable, la pHmetra de control puede diferirse 6-12 meses. En casos de RGE leve o de muy buena evolucin no ser imprescindible la realizacin de pHmetra de control previa al alta definitiva. 3.7 Control pre y postquirrgico. Aunque la pHmetra no es el nico criterio para indicar la ciruga, s que puede mostrar datos que la aconsejen, como la persistencia de un reflujo importante tras el tratamiento, la existencia de reflujos nocturnos prolongados, etc. Por otra parte, la mejora del registro puede aconsejar el retraso de la ciruga. Aunque no existe acuerdo unnime sobre la indicacin tras la ciruga, puede realizarse 3-6 meses despus para comprobar la competencia del mecanismo antirreflujo. En aquellos pacientes con persistencia de los sntomas es necesario realizar pHmetra antes de valorar la reintervencin.

89.- Masculino de 5 aos es llevado al servicio de urgencias por dolor sbito hiperemia, y aumento de volumen en regin interna de rbita derecha. Exploracin Fsica: Se palpa masa bien delimitada dolorosa en el borde interno de la rbita. La medida teraputica inicial en este paciente es: a) b) c) d) Slo drenaje. Tobramicina tpico y drenaje. Extirpacin de saco lagrimal. Dicloxacilina y drenaje.

El tratamiento debe iniciarse antes de que se identifique el microorganismo causal. Tan pronto se obtienen cultivos de nariz, conjuntivas y sangre deben administrarse antibiticos. La teraputica antibitica inicial debe cubrir estafilococos, H influenzae y anaerobios. La mayor parte de los casos responde a los antibiticos. Aquellos en los cuales esto no es as, pueden requerir drenaje. Riordan P,Witcher j, Oftalmologa general de Vaughan y Asbury, 17 Ediciin, Pg. 262

90.- In a male patient in determining non-pathological short stature, the most common cause is: a) b) c) d) Hypothyroidism Family Malnutrition Nephropathy

La talla constituye un indicador muy til del estado de salud de un nio, por lo cual una desviacin significativa del canal de crecimiento normal puede ser la primera manifestacin de una alteracin clnica de importancia. Junto con la talla de un nio, es importante evaluar su velocidad de crecimiento, parmetro ms sensible ya que permite detectar cambios en el canal de crecimiento que habitualmente preceden a los cambios en la talla absoluta. La velocidad de crecimiento debe establecerse sobre perodos no menores a 6 meses para ser confiable, restando particular atencin a las desviaciones significativas y no a aquellas menores, que pueden observarse en nios normales. Se define como talla baja aquella ubicada bajo el percentil 3 o bajo 22 desviaciones estndar de las curvas de NCHS. Estas curvas, de origen norteamericano, tienen utilidad para pacientes pertenecientes a estratos socioeconmicos medios y altos, pero tienen menos utilidad para evaluar a nios que pertenecen a estratos socioeconmicos bajos en nuestro pas. En general ameritan estudio aquellos pacientes cuya talla se encuentra bajo el percentil 3, o cuya velocidad de crecimiento se ha deteriorado en forma significativa incluso antes de ubicarse 22 desviaciones estndar bajo la media. Las causas de talla baja se clasifican en: 1. Variantes normales: talla baja familiar y retraso constitucional. 2. Trastornos primarios del crecimiento como displasias esquelticas, trastornos del desarrollo intrauterino y anormalidades cromosmicas.

3. Alteraciones del crecimiento secundarias a nutricin inadecuada, enfermedades crnicas (como sndrome de malabsorcin, insuficiencia renal, alteraciones pulmonares o cardacas), y enfermedades endocrinolgcas (como hipotiroidismo, alteraciones del eje somatotrfico, sndrome de Cushing, o raquitismo).

Causas de talla baja Familiar Retardo constitucional Retardo del crecimiento intrauterino Alteraciones nutricionales Enfermedades sistmicas Alteraciones cromosmicas Displasias seas Deprivacin emocional - maltrato infantil Trastornos endocrinos

REFERENCIAS 1. Mahoney CP: Evaluating the child with short stature. Pediatr Clin North Am 1987; 34: 825-48. 2. Mericq V, y Cassorla F: Sistema hormona del crecimiento-efector y su rol en el crecimiento infantil. Rev Chil Pediatr 1997; 68: 27-37. 3. Rosenfeld R, Albertsson- Wikland K, Cassorla F, et al: Diagnostic controversy, the diagnosis of childhood growth hormone deficiency revisited. J Clin End 4. Codner E, Mericq V, Ugarte F, et al: Utilidad de la determinacin del factor de crecimiento insulino smil tipo 1 y de su protena ligante tipo 3 en el diagnstico de la deficiencia de hormona de crecimiento en nios. Rev Md Chile 1999; 127: 807-13.

91.- Femenino de 1 ao y medio de edad, es llevada por sus padres a consulta, por que presenta lesiones rojas, con costras melicricas en la cara, cuero cabelludo, rea de paal y extremidades, desde aproximadamente los 3 meses de edad. El evitar sustancias irritantes no ha funcionado en el tratamiento. La nia se rasca constantemente las reas afectadas. Hay una historia familiar positiva para asma y fiebre. Cul es el diagnstico ms probable? a) b) c) d) Celulitis Dermatitis de contacto Dermatitis atpica Liquen simple crnico

La dermatitis atopica (DA) (Wise-Sulzberger, 1993), llamada neurodermatitis diseminada, por las escuelas Europea, es un estado reaccional de la piel, intensamente pruriginosa, ms frecuente en los nios, multifactorial, en la que combinan factores constitucionales y factores ambientales, por lo tanto de difcil tratamiento, muy frecuente en la consulta diaria del pediatra y del dermatlogo.

DATOS EPIDEMIOLOGICOS La enfermedad originalmente conocida como prrigo de Besnier y eccema constitucional, es ubicua, afecta a todas las razas y existe en todos los pases. Se seala su existencia hasta en el 2% de la poblacin general y en el 14% de los nios. La enfermedad se inicia antes del primer aos de la vida en ms del 60% de los casos, la curva desciende hacia los 12 aos de la cifra de inicio es apenas del 5%-y es excepcional que la enfermedad se inicie en la edad adulta. La enfermedad sufre exacerbaciones en las temporadas de calor o de fro cuando hay sequedad de la atmsfera.

CUADRO CLNICO Tradicionalmente se han descrito tres etapas que en la actualidad ya no se presentan pues se suman unas a otras debido a los tratamientos que reciben los pacientes desde su inicio. Etapa de lactante. La enfermedad suele iniciarse en los primeros meses de vida, a veces casi desde el nacimiento. Las lesiones afectan la cara: mejillas, frente (respetando su centro); afectan tambin la piel cabelluda, los pliegues, las nalgas y el dorso de los pies. Las lesiones son de tipo eccematosos (piel llorosa): eritema, vescular y costras melicricas con costras hemticas como signo del rascado. Estas lesiones altamente pruriginosas evolucionan por brotes y en general si el paciente no es yatrognicamente tratado, tienen tendencia a involucionar al ao de edad. Fase del escolar. Se inicia hacia los 3 a 7 aos, cuando el nio empieza a ir a la escuela. Las lesiones son preferentemente flexurales: cuello, pliegues de codo y huecos poplteos y la morfologa corresponde a la de una dermatitis crnica: zonas de eritema y liquenificacin (ndice de rascado crnico) y costras hemticas peridicamente sufren un proceso de eccematizacin sobre todo por los tratamientos indebidos. La enfermedad evoluciona por brotes, hay temporadas en que no existen lesiones aun cuando persiste a veces una piel xertica (seca) y pigmentada con intenso prurito. Etapa del adulto. Se vea con poca frecuencia y ahora es habitual en las consultas diarias debido a los malos tratamientos. En este caso adems de las zonas flexurales, se presenta lesiones periorbitarias; y peribucales y lesiones vesiculosas en las manos. Son lesiones tanto liquenificadas como eccematosas, muy pruriginosas, que alternan con periodos asintomticos. Con el uso inmoderado de los corticoesteroides no hay diferenciacin entre las etapas de la enfermedad y se ven casos que arrastran su enfermedad casi desde que nacen hasta la edad adulta.

DIAGNSTICO Es clnico y relativamente sencillo, aunque no toda dermatitis flexural es necesariamente de origen atpico. Deben tomarse en cuenta los antecedentes del propio enfermo y los familiares. En los lactantes debe diferenciarse de la dermatitis seborreica que afecta sobre todo la piel cabelluda y el centro de la cara. En ocasiones hay mezcla de las dos dermatitis: la atpica y la seborreica y es difcil diferenciarlas. La dermatitis del paal predomina en los lactante; en las zonas glteas y genitales, se presenta con eritema y lesiones vesculopapulosas muy pruriginosas. La pitiriasis alba corresponde a los clsicos jiotes de los nios: manchas hipocrmicas cubiertas de fina escama en la cara. Estos procesos pueden coincidir con manifestaciones de DA, pero nunca se ha demostrado la relacin de causa efecto.

TRATAMIENTO Medidas generales. Es conveniente una explicacin amplia al paciente y su familia sealando la naturaleza y evolucin de la enfermedad y lo que se espera del tratamiento que vamos a indicar. Evitar el sol excesivo, el agua clorada de las albercas, el uso de jabn (indicar jabones neutros o sustitutos de jabn), prohibir el uso de pomadas y remedios caseros as como los cosmticos que irritan a la piel. Las dietas restrictivas han sido y siguen siendo un tema muy controversial, pues mientras los alergistras siguen insistiendo sin muchas bases en el beneficio de estas dietas, los ms hemos comprobado su inutilidad. Existe un grupo muy limitado en que se puede demostrar que una dieta restrictiva de huevo, leche, fresas, etc. puede mejorar los brotes de DA y en tales casos (slo en esos casos) se aconsejara la supresin de tales medicamentos; en lo general se permite al paciente que coma de todo. En la actualidad se estn limitando por los pediatrias ciertos alimentos como el huevo, las fresas, el pltano y la leche de vaca en el primer ao de vida por ser muy alergnicos y se discute la utilidad de la leche materna en estos nios. Es necesario hacer ver que estos pacientes requieren de una atencin ms personal, que sienten la necesidad de cario por lo que es de recomendarse que el nio sea atendido personalemente por la madre. Tratamiento tpico. Depende del estado de la piel; si est eccematosa debe secarse antes de aplicar cualquier pomada que ser rechazada por la piel llorosa. El uso de fomentos con agua de manzanilla o suero fisiolgico es til. Los fometos con agua de vgeto (subacetato de plomo) al 20% son potentes antiexudativos, pero deben limitarse a reas restringidas y no usarse en nios. Estando la piel ya seca, se usan pasta inertes que llevan xido de zinc y calamina en una base de vaselina y lanolina que son protectoras a la vez que antiprutiginosas. Si la piel est muy seca y liquenificada, los fomentos y baos sern emolientes, con almidn y aceites seguidos de cremas o pomadas ms grasosas que llevan vaselina, coldcream y xido de zinc. En el comercio existen numerosos preparados humectantes que ayudan a mantener el manto cido grasa-agua, que se pierde en la DA. Si hay dermatitis por contacto o imptigo hay que tratar primero estas complicaciones con sulfato de cobre al 1: 1000 y pomadas con vioformo o mupirocn. En el caso de eritrodermia, los baos emolientes y el uso del petrolato (vaselina) ser lo indicado. Cuando hay mucha liquenificacin, pueden usarse cremas con alquitrn de hulla al 3% en base de coldcream por tiempo y zonas limitadas. Los corticoesteroides tpicos son los medicamentos ms usados en esta enfermedad y muchas veces causa de las complicaciones que se presentan. Estos medicamentos no curan nada, solo engaan al paciente y al mdico hacindoles creer que la enfermedad va curando cuando slo se oculta y modifica. Al pasar el efecto de estos medicamentos, invariablemente se presenta el rebote y ms tarde la corticodependencia, haciendo a la enfermedad incontrolable. Nunca deben usarse los corticoesteroides fluorinados en nios y en la cara y zonas genitales o en los pliegues por su posibilidad de absorcin.

La hidrocortisona es de baja potencia, hace menos dao, pero tambin es menos efectiva; algunos la recomiendan. Tratamiento sistmico. El uso de antihistamnicos sobre todo de la primera generacin que son sedantes como la clorfeniramina y la hidroxicina ayudan a mejorar el prurito; el ketotifeno y la oxotamida por su accin dual: inhiben la produccin de histamina por los mastocitos y bloquean los receptores Hl, tambin son de ayuda, al igual que los sedantes suaves tanto para el paciente como para la madre que est en perenne angustia que transmite al pequeo paciente. La talidomida ha mostrados ser de ayuda en casos de DA conticoestropeada, en su fase eritrodrmica, no tanto en los nios, a dosis de 100 mg al da. Los antibiticos tipo dicloxacilina, sern necesarios cuando haya infeccin o simplemente eccematizacin por el papel que tiene el estafilococo dorado.

1. Criterios Diagnstico para Dermatitis Atpica (DA), MedicalCriteria.com. Dr. Luciano Domnguez-Soto, Dr. Amado Sal Cano, Dermatitis atpica o neurodermatitis diseminada, Dermatologa, Parte C, Libro ) "

92.- El siguiente grupo de microorganismos patgenos son capaces de multiplicarse extracelularmente, pero que resisten la accin fagocitaria del macrfago, lo que ocaciona puedan persistir latentes, su erradicacin depende de la activacin de los macrfagos por los linfocitos T CD4 (T helper). A que grupo nos referimos?

a) Entamoeba histolytica. b) Mycobacterium tuberculosis. c) Pseudomonas aeruginosa. d) Virus de la hepatitis C.

La supervivencia del bacilo de Koch (BK) dentro del hospedador depende de la capacidad de los microorganismos para multiplicarse dentro de los macrfagos y monocitos. La inmunidad del hospedador a M. tuberculosis depende de que ocurra una respuesta celular antimicobacteriana que active los macrfagos para que produzcan la muerte o restrinjan el crecimiento de los microorganismos intracelulares. La tuberculosis es la enfermedad micobacteriana humana clsica. La va de infeccin por Mycobacterium tuberculosis consiste en la inhalacin de aire con partculas infectadas que alcanzan la va area terminal. Una vez deglutidos por los macrfagos alveolares, los bacilos comienzan a multiplicarse libremente y acaban destruyendo las clulas fagocitarias. De esta manera, se produce un ciclo posterior de fagocitosis por los macrfagos y linfocitos, que emigran hacia el foco de infeccin; finalmente, se produce la destruccin celular. Los macrfagos infectados diseminan el proceso hacia los ganglios linfticos locales en la fase inicial de la enfermedad, as como hacia el torrente circulatorio y otros tejidos (por ejemplo: mdula sea, bazo, riones, huesos y sistema nervioso central

93.- Femenino de 45 aos que tras 20 min. Posteriores al inicio de una transfusin presenta fiebre, ansiedad, dolor lumbar severo, nausea y enrojecimiento facial, por lo que usted sospecha: a) Reaccin febril b) Reaccin hemoltica aguda c) Toxicidad por citrato d) Hepatitis

R e a cc i n Hemoltica Definicin: Destruccin acelerada del eritrocito. De acuerdo a la causa puede ser inmune o no inmune, por el sitio de destruccin puede ser intra o extravascular y por el tiempo de aparicin puede ser aguda o retardada. Incidencia Reaccin hemoltica aguda: Las referencias internacionales reportan una incidencia de reaccin hemoltica aguda de 1 en 6 000 en 30 000 unidades transfundidas, con una tasa de mortalidad de 1 en 500 000 a 1 en 1 000 000 de unidades. Del total de las reacciones hemolticas agudas, el 6% resultan fatales. La FDA reporta que alrededor del 41% de las muertes por transfusin son causadas por incompatibilidad ABO, con una incidencia de mortalidad de 1 en 200 000 pacientes transfundidos. Reaccin hemoltica retardada. La incidencia es de 1 en 2 500 a 1 en 4 000 unidades transfundidas. La mortalidad es de 1 en 3.85 millones de unidades y de 1 en 1.15 millones de pacientes transfundidos.

Fisiopatogenia La reaccin hemoltica transfusional ms grave se presenta cuando interactan los eritrocitos transfundidos con los anticuerpos preformados en el receptor. La reaccin antgenoanticuerpo puede no activar complemento de acuerdo a la inmunoglobulina implicada, lo que conduce a hemlisis intra o extravascular. En la hemlisis extravascular algunas citocinas con actividad inflamatoria y vasoactiva intervienen en la reaccin como: Factor de necrosis tumoral alfa, Interleucina 1, 6, 8 y Protena quimioatractante de macrfago (MCP), as como la liberacin de sustancias tromboplsticas que explican el cuadro clnico caracterstico de la reaccin hemoltica transfusional. En la hemlisis extravascular el eritrocito sensibilizado es destruido por el sistema fagoctico mononuclear.

REACCION HEMOLITICA AGUDA INMEDIATA DE TIPO INMUNE: Es la reaccin ya descrita por incompatibilidad ABO y como consecuencia de ella al iniciar la transfusin en pocos minutos pueden ser lisados la mitad ms de los de los hemates transfundidos. La reaccin hemoltica aguda se produce de forma brusca, tras la infusin de varios mililitros de sangre aparece un cuadro de fiebre, escalofrios, dolor lumbar, dolor opresivo en torax , cefalea nauseas con sin vomitos e incluso shock,dependiendo de la cantidad y rapidez de la sangre transfundida. Los sintomas dependen de los pacientes pero la fiebre aparece en todos los casos. La reaccin es inmediata al inicio de la transfusin En los enfermos anestesiados estas reacciones quedan enmascaradas y ser mnimas,por lo que la hemlisis puede debutar con taquicardia e hipotensin brusca trs el inicio de la transfusin,si persiste la transfusin puede aparecer oliguria,hemoglobinuria,CID fracaso renal postoperatorio debido a algn efecto txico de la Hb a la situacin de shock.

Diagnstico Diferencial Contaminacin Bacteriana del componente sanguneo Hemlisis no inmune: - Mecnica - Trmica - Osmtica

Lecturas recomedadas: Rev Mex Med Tran, Vol. 3, Nm. 1, pp 18-21 Enero - Abril, 2010 Hospital de Pediatra CMN Siglo XXI Boletn La Calidad Abr May Jun 2004

94.- Femenino de 58 aos, que acude al mdico por cefalea y mareos. Exploracin fsica: rubicundez facial. Exmenes de Laboratorio: Hto 62%, leucocitos 13.000/microl con frmula normal; plaquetas 325.000/microl; saturacin arterial de oxgeno 95%; fosfatasa alcalina granuloctica 150 (sobre una puntuacin mxima de 200; valor de referencia: 30 a 80); B12 srico 593 pg/ml (valores de referencia: 170 a 920 pg/ml); Masa eritrocitaria: 35 ml/kg. La patologa responsable de los hallazgos de laboratorio en esta paciente es: a) Eritrocitosis espuria. b) Vrtigo de Mnire. c) Poliglobulia secundaria a hemangioma cerebeloso. d) Policitemia vera.

Trastorno mieloproliferativo de etiologa desconocida, que se caracteriza por proliferacin anormal de todos los elementos hematopoyticos de la mdula sea y por incremento absoluto de la masa de clulas rojas y del volumen total de sangre, y que se asocia frecuentemente con esplenomegalia, leucocitosis, y trombocitopenia. La hematopoyesis tambin es reactiva en sitios extramedulares (hgado y bazo). En ocasiones ocurre mielofibrosis. El curso clnico de la policitemia vera se complica con frecuencia con trombosis y posible transicin a metaplasia mieloide con mielofibrosis o leucemia mieloide aguda. Dejada a su curso natural la supervivencia media es de 18 meses. Las flebotomas y el tratamiento citoreductor han reducido las complicaciones tromboticas y mejorado sustancialemente la superviviencia. El dilema terpaetico que se plantea en esta entidad es la utilizacin aislada de la flebotomia para prevenir el riesgo de trombsosi o la combinacion con el tratamiento citorreductor, por ejemplo busulfan, con el riesgo, entonces de ocurrencia de mielofiboris o transformacin maligna.( Polycythemia Vera Study Group ) .

Bibliografa: 1. Berk PD, Goldberg JD, Silverstein MN, et al.: Increased incidence of acute leukemia in polycythemia vera associated with chlorambucil therapy. New England Journal of Medicine 304(8): 441-447. 2. Silverstein MN: Myeloproliferative disease. Current Hematology and Oncology 6: 163-184, 1988. 3. Demory JL, Dupriez B, Fenaux P, et al.: Cytogenetic studies and their prognostic significance in agnogenic myeloid metaplasia: a report on 47 cases. Blood 72(3): 855-859, 1988. 4. Barosi G: Myelofibrosis with myeloid metaplasia: diagnostic definition and prognostic classification for clinical studies and treatment guidelines. Journal of Clinical Oncology 17(9): 2954-2970, 1999. Tefferi A: Myelofibrosis with myeloid metaplasia. New England Journal of Medicine 342(17): 1255-1265, 2000.

95.- Masculino de 22 aos de edad, que acude al servicio de cardiologa, sin antecedentes patolgicos y sin hbitos txicos inicia con cuadro de 8 das de evolucin acompaado de fiebre y dolor centrotorcico intenso que aumenta con la inspiracin y los movimientos respiratorios. En el ecocardiograma se objetiva un derrame pericrdico importante, sin signos de compromiso hemodinmico. Cul sera su primer diagnstico?

a) b) c) d)

Pericarditis aguda idioptica. Pericarditis tuberculosa. Taponamiento cardaco. Pericarditis de origen autoinmune.

La pericarditis aguda (PA) es un sndrome clnico debido a una inflamacin del pericardio que se caracteriza por dolor torcico, roce pericrdico y alteraciones electrocardiogrficas evolutivas. Su incidencia y prevalencia son difciles de determinar. Estudios realizados en autopsias muestran una prevalencia de alrededor de 1% en la poblacin general, lo que sugiere que la presentacin en ocasiones se hace de manera subclnica. Representa un 5% de los dolores torcicos no isqumicos que acuden a urgencias 1.

La causa ms frecuente de PA es la idioptica y/o viral, que corresponden a casi el 80% de los casos, de hecho, los trminos idioptico y viral suelen utilizarse de forma indistinta 2-5. Otras posibles causas son: Infecciosa (7%): Bacteriana: neumococo, estreptococo, estafilococo, neisseria, legionella. Tuberculosa. Vrica: coxsackie, influenzae, VIH, hepatitis, adenovirus, echovirus. Fngica: histoplasmosis, coccidiomicosis, blastomicosis, candidiasis. Otras: sfilis, protozoos, parsitos (entamoeba histolytica, toxoplasma). Neoplasias (7%): primarias: mesotelioma, sarcoma, fibroma, lipoma secundarias (metstasis): pulmn, mama, linfoma, leucemia, carcinoides La triada diagnstica clsica es: dolor torcico, roce pericrdico y alteraciones electrocardiogrficas que suele ser precedida de fiebre, mal estado general y mialgias (aunque en pacientes ancianos no suele presentarse la fiebre). Los sntomas principales son: Dolor torcico: dolor retroesternal o localizado en hemitrax izquierdo en la zona precordial, de caractersticas pleurticas (aunque en ocasiones puede simular un cuadro isqumico), que se puede irradiar al cuello o al trapecio izquierdo. Puede acompaarse de sensacin de falta de aire. Ocasionalmente se localiza en regin epigstrica, simulando un abdomen agudo. Suele aumentar con la inspiracin profunda, la tos, la deglucin y la posicin supina y mejorar con la incorporacin a la posicin de sentado. Roce pericrdico: es el hallazgo patognomnico de la exploracin fsica en la PA. Corresponde al movimiento del corazn dentro del pericardio y se asemeja al ruido que produce la friccin de cuero, por roce de las hojas pericrdicas inflamadas. Se da en aproximadamente el 60-85% de los casos 7,8. Puede ser transitorio, monofsico, bifsico o trifsico, segn la relacin que tenga con los movimientos cardiacos durante la sstole auricular, sstole ventricular y el llenado ventricular rpido. Se ausculta mejor a nivel de mesocardio y en parte baja de borde esternal izquierdo, sobretodo al final de la espiracin con el paciente inclinado hacia delante. Es independiente de la existencia de derrame. Cuando slo tiene un componente se puede confundir con un soplo sistlico mitral o tricuspdeo. Es caracterstico del roce pericrdico su evanescencia (por lo que las auscultaciones deben ser repetidas en varias ocasiones) y los cambios en sus caractersticas segn la posicin en que se realice la exploracin.

Bibliografa:
LeWinter MM, Kabbani S. Pericardial diseases. En: Braunwalds heart disease. Douglas P. Zipes editor. 7th. Ed. Philadelphia: Elsevier; 2005. p.1757-1780 Friman G, Fohlman J. The epidemiology of viral heart disease. Scand J Infect Dis Suppl 1993; 88: 710. [Medline] Braunwald E. Enfermedades del pericardio. En: Harrison Principios de Medicina Interna. Dennos L. Koper editores. 16 ed. Mxico: Mc-Graw-Hill; 2005. p. 15541571 Maisch B, Seferovic PM, Ristic AD, Erbel R, Rienmller R, Adler Y, et al, Grupo de Trabajo para el Diagnstico y Tratamiento de las Enfermedades del Pericardio de la Sociedad Europea de Cardiologa. Gua de Prctica Clnica para el diagnstico y tratamiento de las enfermedades del pericardio. Versin resumida. Rev Esp Cardiol. 2004; 57:1090-114. [Medline] [Texto completo] Zayas R, Anguita M, Torres F, Gimenez D, Bergillos F, Ruiz M, Ciudad M, Gallardo A, Valles F. Incidence of specific etiology and role of methods for specific etiologic diagnosis of primary acute pericarditis. Am J Cardiol. 1995 Feb 15; 75(5):378-82.

96.- En nuestro pas, la Secretara de Salud ha lanzado una campaa para fomentar entre el personal de las instituciones gubernamentales, una rutina de ejercicios con duracin de cinco minutos diarios, esta es una accin especfica de: a) b) c) d) Restauracin de la salud Medicina preventiva Proteccin de la salud Promocin de la salud.

Las acciones de Promocin de la Salud pretenden fomentar la salud de los individuos y la colectividad, promoviendo que adopten estilos de vida saludables. Ello se consigue mediante intervenciones de educacin sanitaria.

Piedrola G. G; Medicina Preventiva y Salud Pblica. Masson, 9 ed. Pg. 15, 16.

97.- Al existir la sospecha de lquido libre en la cavidad peritoneal y despus de analizar los hallazgos de manera conjunta de las placas de abdomen y en el ultrasonido abdominoplvico, Dnde se debern de buscar los cambios de densidad de manera inicial?

a) b) c) d)

Infradiafragmtico Fondos de saco de Douglas y de Morrison Transcavidad de los epiplones Correderas parieto-clicas

Pedrosa C, Casanova R. Diagnstico por imagen. Mc Graw Hill 2001 p. 209, 210. La diseminacin del lquido intraperitoneal est producida por la gravedad y la presin hidrosttica secundaria al movimiento diafragmtico.la pelvis es la parte ms dependiente de la cavidad peritoneal, su capacidad es de 300 cc, a partir de lo cual el lquido asciende por las correderas parietoclicas. El lquido plvico desplaza las asas intestinales que rellenan los espacios plvicos, el leon del lado derecho y el colon sigmoides en el izquierdo, lo que resulta de la paricin de una densidad homognea en al pelvis menor, en contra del patrn poco homogneo de las asas intestinales que tienen gas y lquido. En la pelvis femenina el lquido se acumula en los recesos laterales de la vejiga y al tero, y en el hombre puede verse en la fosa vesicorrectal y los recesos plvicos laterales. En el ultrasonido el diagnstico de pequeas cantidades de lquido debe hacerse fundamentalmente en el saco de Douglas, en la bolsa de Morrison y en el receso yuxtaesplnico.

98.- Masculino de 12 aos, acude al centro de urgencias 30 mins. Despus de haber ingerido comprimidos de sulfato ferroso de 325 mg. La madre indica que faltan 20 comprimidos del frasco. Cada comprimido tiene 65 mg de hierro elemental. El nio ha vomitado una vez y su comportamiento parece totalmente normal. Los hallazgos de la exploracin fsica no arrojan nada importante. Pesa 13 kg. Cul de los siguientes enunciados sobre el estado del nio es el correcto?

a) b) c) d)

La dosis de hierro no debera causar secuelas clnicas. Ese episodio nico de vmito probablemente no esta relacionado con la ingestin. Aunque ha vomitado una vez debera administrarse ipecacuana. Los catrticos son ineficaces en intoxicaciones.

En situaciones en las que no hay testigos de la ingestin debe presuponerse inicialmente el peor escenario posible para estimar la toxicidad potencial. La dosis mxima de hierro ingerida por este nio es de 65 mg de hierro elemental x 20 pldoras / 13 kg = 100 mg /kg que es una dosis potencialmente grave. Entre los sntomas causados por la ingestin grave de hierro estn los vmitos por tanto en este caso el vmito debe considerarse relacionado con la ingestin. Ello implica un riesgo potencial de secuelas graves. En conjunto la ingestin debe valorarse como clnicamente importante. Como un solo episodio de vmitos no vaca el estmago de manera suficiente debe administrarse ipecacuana o alternativamente proceder a un lavado gstrico. El carbn activado no adsorbe el hierro y no es necesario. Los catrticos sirven de ayuda una vez que e ha vaciado el estmago. Los niveles de hierro srico y de capacidad de enlace del hierro deben estimarse segn las circunstancias. Tambin es til obtener una radiografa de abdomen por cuanto las pldoras son radioopacas y puede deducirse as algn indicio de su permanencia en el tracto GI despus del tratamiento inicial.

1.- Montoya-Cabrera MA. Intoxicaciones y envenenamientos en nios. Mxico, Intersistemas , 2000. 2.-Montoya CMA. Toxicologa clnica. 2. Ed, Mxico, Mndez Editores, 1997

99.- Femenino de 49 aos, con antecedentes de obesidad, IAM hace 5 aos as como tabaquismo positivo. Ingresa al servicio de traumatologa por fractura de cadera, ser intervenida quirrgicamente para reemplazo total de la misma, el riego que presenta esta paciente es: a) b) c) d) Paciente de riesgo bajo para trombosis venosa profunda Paciente de riesgo alto para trombosis venosa profunda Paciente con riesgo de trombosis arterial Paciente con riesgo muy bajo de TEP

La fisiopatologa de la TVP se resume mediante la trada de Virchow: estasis sanguneo, dao endotelial e hipercoagulabilidad. Estas 3 circunstancias aisladamente o en asociacin, intervienen en el desarrollo de un trombo. Los factores de riesgo enumerados a continuacin aumentan la probabilidad de desarrollar trombosis mediante uno o ms de los mecanismos de la trada (Kahn S, 1998): 1. Ciruga mayor, especialmente la ciruga ortopdica, pero tambin los pacientes sometidos a ciruga abdominal, neurociruga. 2. Neoplasias malignas, el riesgo aumenta en los pacientes que reciben quimioterapia activa. 3. Infarto Agudo de Miocardio. 4. Sndrome Nefrtico.

5. Ictus isqumico, preferentemente en el miembro hemipljico. 6. Inmovilizacin prolongada. A mayor tiempo de inmovilizacin mayor riesgo, aunque se ha documentado una incidencia de TVP del 13% en pacientes encamados durante 8 das. 7. TVP o Embolismos Pulmonares (EP) previos. El riesgo se debe probablemente a la persistencia de obstruccin al flujo y/o dao en las vlvulas venosas tras la TVP anterior. 8. Embarazo y postparto. 9. Anticoncepcin oral y Terapia Hormonal Sustitutiva (THS). 10. Alteraciones congnitas o adquiridas que produzcan hipercoagulabilidad: o Deficiencia congnita de Protena C, Protena S y Antitrombina III. o Resistencia a la Protena C activada: esta alteracin se encuentra en el 5% de la poblacin general y en el 20-40% de los enfermos con TVP. o Hiperhomocisteinemia o Disfibrinogenemia o Presencia de Anticuerpos Antifosfolpido Estos trastornos se asocian a TVP recurrentes, o en localizaciones atpicas o de aparicin a edades tempranas (antes de los 45 aos). Estos trastornos se asocian a TVP recurrentes, o en localizaciones atpicas o de aparicin a edades tempranas (antes de los 45 aos).

A pesar de la naturaleza inespecfica de los signos y sntomas de la TVP aguda Wells y cols desarrollaron un modelo clnico objetivo que incorpora 8 caractersticas clnicas o factores de riesgo asignndoles 1 punto a cada una si estn presentes y 2 si existe la probabilidad de un diagnstico alternativo, esto permite clasificar a los pacientes en baja probabilidad 0 puntos, probabilidad moderada 2 puntos y probabilidad alta 3 puntos con prevalencias de TVP de 5%, 33% y 85% respectivamente. Este modelo de probabilidad previa es validado en mltiples ensayos adems de que tiene una excelente reproducibilidad (Cuadro 1) (8,1,12).

Bibliografa:
Ebell MH. Evaluation of the patient with suspected deep vein thrombosis. J Fam Pract. 2001 Feb;50(2):167-71 [PubMed] [Texto completo] Gabriel Botella F, Labis Gmez M, Bras Aznar JV. Trombosis venosa profunda: presente y futuro. Med Clin 2000; 114: 584-596. [PubMed] Gorman WP, Davis KR, Donnelly R. ABC of arterial and venous disease. Swollen limb1: General assessment and deep vein thrombosis. BMJ 2000; 320: 1453-1456. [PubMed] [Texto completo] Green L, Fay W, Harrison V, Kleaveland M, Wahl R, Wakefield T, Weg J, Williams D. Venous thromboembolism (VTE) [Internet]. Ann Arbor (MI): University of Michigan Health System 2004 [acceso 18/3/2007] Disponible en: http://cme.med.umich.edu/iCME/vte04/ [NGC]

100.- A 6-year-old child presents with flesh-colored papules on the hand that are not pruritic. Examination reveals lesions that are approximately 4 mm in diameter with central umbilication. A halo is seen around those lesions undergoing regression. Which of the following is the most likely diagnosis?

a) b) c) d)

Verruca vulgaris Molluscum contagiosum Keratoacanthoma Herpetic whitlow

El Molusco Contagioso es el nombre de una infeccin viral del grupo de los Poxvirus. Es frecuente, transmisible, autoinoculable (uno mismo la puede trasmitir a diferentes partes del cuerpo), se puede curar sola y es benigna. Afecta principalmente a los nios menores de 10 aos, adultos sexualmente activos y a pacientes inmunosuprimidos (con defensas bajas) como los pacientes con SIDA, ocurriendo en un 5 18 por ciento. Afecta ms a hombres que a mujeres, y su frecuencia aumenta tambin en climas tropicales y durante el verano. CLINICA El periodo de incubacin de la infeccin es de 14 a 50 das, aunque hay datos de recin nacidos con lesiones al cabo de 7 das postparto. Las lesiones se inician como neoformaciones que miden generalmente de 2 a 6 mm, aunque pueden llegar a medir 3 cm(13,14), son hemisfricas, cupuliformes, lisas, del color de la piel o perladas, algunas (20%) tienen una umbilicacin central(15); la base es levemente eritematosa y son de consistencia firme(13,14). Se localizan en cualquier parte del cuerpo e incluso pueden afectar mucosas, generalmente se agrupan en un rea especfica, pero pueden estar diseminadas en personas infectadas con el virus de la inmunodeficiencia humana adquirida, siendo un marcador de enfermedad avanzada (16). En el caso de los nios las lesiones se localizan normalmente en cara, tronco, brazos y piernas a diferencia de los adultos jvenes en quienes el molusco contagioso se adquiere por transmisin sexual, las lesiones tienen predileccin por genitales, abdomen y cara interna de los muslos; en otras series no hay diferencia (17). Las lesiones del molusco suelen aparecer entre los 14 das a 6 meses despus de la exposicin, hay datos de recin nacidos con lesiones al cabo de 7 das postparto. Se pueden propagar por auto inoculacin, pero es tpico que se resuelvan espontneamente en pocos meses. Las lesiones son asintomticas en la mayora de los pacientes (13,18), aunque en el 10% de los casos puede haber prurito y desarrollarse una reaccin eccematosa (14).

CRITERIO DIAGNSTICO El diagnstico se hace clnicamente y en algunos casos dudosos puede efectuarse biopsia con tincin H-E donde se encuentran los cuerpos de molusco ( inclusiones intracitoplasmticas grandes) o de Hendersen-Paterson; el 90% de los pacientes posee Ac tipo Ig G. Puede realizarse microscopa electrnica, PCR, Elisa, e inmunohistoqumica.

REFERENCIAS 1. Schotz J, Rosen-Wolft A, Bugert J et al. Molecular epidemiology of molluscum contagiosum. J Infect Dis 1988; 158: 898-900.

2. Porter CD, Nlake NW, Archard LC et al. Molluscum contagiosum virus type in genital and non genital lesions. Br J Dermatol 1989; 120: 37-41. 3. Overfield TM, Briody JA. An epidemiologic study of molluscum contagiosum in Achorage, Alaska. J Pediatr 1966; 69: 640-42. 4. Gottlieb SL, Myskowwski PL. Moluscum contagiosum. Intern J Dermatol 1994; 33: 453-61. 5. Telner P, Solomon LM. Eruptive molluscum contagiosum in atopic dermatitis. Can Med Assoc J 1966; 95: 978-79. 6. Pauly CR, Artis WM, Jones HE. Atopic dermatitis, impaired cellular immunity and molluscum contagiosum. Arch Dermatol 1978; 114: 391-93. 7. Dohil, P. Lin, J. Lee, A. Lucky, A. Paller, L The epidemiology of molluscum contagiosum in children. J Am Acad Dermatol. 2006;54:47-54. 8 Rosenberg EW, Yusk JW. Molluscum contagiosum. Eruption following treatment with prednisone and methotrexate.Arch Dermatol 1970; 101: 43941. 9. Koopman JJ, Van Merrienboer FCJ, Vreden SGS, Dolmans WMV. Molluscum contagiosum: a marker for advanced HIV infection. Br J Dermatol 1992; 126: 528-29. 10. Schwartz JJ, Myskowski PL. HIV-related molluscum contagiosum presenting as a cutaneous hom. Int J Dermatol1992; 31: 142-44.

You might also like